Tổng hợp đề đề xuất kì thi duyên hải và đồng bằng bắc bộ năm 2016 môn Tiếng Anh khối 11

Page 1

ĐỀ THI HSG DUYÊN HẢI BẮC BỘ MÔN TIẾNG ANH

vectorstock.com/3687784

Ths Nguyễn Thanh Tú eBook Collection DẠY KÈM QUY NHƠN OLYMPIAD PHÁT TRIỂN NỘI DUNG

Tổng hợp đề đề xuất kì thi duyên hải và đồng bằng bắc bộ năm 2016 môn Tiếng Anh khối 11 có đáp án và phần nghe WORD VERSION | 2021 EDITION ORDER NOW / CHUYỂN GIAO QUA EMAIL TAILIEUCHUANTHAMKHAO@GMAIL.COM

Tài liệu chuẩn tham khảo Phát triển kênh bởi Ths Nguyễn Thanh Tú Đơn vị tài trợ / phát hành / chia sẻ học thuật : Nguyen Thanh Tu Group Hỗ trợ trực tuyến Fb www.facebook.com/DayKemQuyNhon Mobi/Zalo 0905779594


HỘI CÁC TRƯỜNG THPT CHUYÊN VÙNG DUYÊN HẢI VÀ ĐỒNG BẰNG BẮC BỘ

KÌ THI HỌC SINH GIỎI NĂM HỌC 2015-2016 MÔN THI: TIẾNG ANH LỚP 11 Ngày thi: 16 tháng 4 năm 2016

TRƯỜNG THPT CHUYÊN BẮC GIANG ĐỀ THI ĐỀ XUẤT

FF IC IA L

Thời gian làm bài: 180 phút (không kể thời gian giao đề) (Đề thi 18 gồm trang)

A. LISTENING (40 pts)

Part 1 (10 pts) You will hear an interview with Roland Brundy, the new chairman of the television channel GTV. For questions 1-5, choose the correct answer A, B, C or D.

ẠY

M

Q

U

Y

N

H

Ơ

N

O

1. According to Roland Brundy, what will be the result of competition within the media? A. a narrower range of programmes on GTV B. the development of new technology C. an increase in GTV’s staff numbers D. greater potential for creativity 2. Roland feels that in order to have ‘artistic’ standing, a channel needs A. an awareness of history. B. to build up expertise. C. accomplished actors. D. to buy expensive programmes. 3. According to Roland, the main problem in reacting to competition is that A. it is hard to avoid copying others. B. all solutions are open to criticism. C. viewers object to change. D. it is hard to interpret the market accurately. 4. How does Roland feel about the impact of technology? A. sceptical B. undecided C. negative D. fearful 5. Roland says that one problem with his type of work is that it is hard to A. adapt to change. B. operate objectively. C. judge its importance. D. measure your success. YOUR ANSWERS 1. 2. 3. 4. 5.

D

Part 2 (10 pts) For question 6-10, listen to a piece of BBC news Listen and decide whether the following sentences are true (T) or false (F). 1. 2. 3. 4. 5.

The Child Well-Being Index has been going for over 75 years. The child death rate has halved since 1975. Children's health has got worse in the past two decades. The report says obesity causes an abundance of fast food. The number of kids below the poverty line hasn't changed for 20 years. 1


YOUR ANSWERS 1. 2. 3. 4. PART 3. For question 11-20, Complete the notes below. Write NO MORE THAN TWO WORDS for each answer.

5.

Australian Aboriginal Art

M

Q

U

Y

N

H

Ơ

N

O

FF IC IA L

ANCIENT ART • Rock and bark painting • Sand drawings • (1) ..................................... • Decorations on weapons and tools Cave art • protected from (2) ......................... • styles include dot paintings (e.g. arrows, water holes and (3)............... ) and naturalistic art • main function: (4) ................................. Use of ochre Reason • readily available • soil or rock contains (5) ...................... • produces many colours and shades of red • artist's palette found that is (6)................... old Preparation • ochre collected • turned into a (7) .................... • fluid binder, e.g. tree sap or (8) ............................... added MODERN ART • Artists use acrylic colours and (9) .................... • Paint and decorate pottery and a range of (10) .........................

1.

2.

3.

4.

5.

6.

7.

8.

9.

10.

ẠY

YOUR ANSWERS

D

B. VOCABULARY AND GRAMMAR (60 pts) Part 1. Choose the word/ phrase that best completes each of the following sentences. (10 pts) 1. Jack has egg ______ because he couldn't remember how to spell "Batman"! A. on his teeth

B. on his face

C. on his shirt

D. on his fingers 2


2. As the maestro lifted his baton the theater was so still you could hear ______ . A. his heartbeat

B. a pin drop

C. bird wings

D. the drum beating

3. I don't agree with you, but your idea certainly gives me food _____. B. for consider

C. for thought

D. for thinking

FF IC IA L

A. for fun

4. I think I understand the nuts and ______ of the operation. A. screws

B. hammer

C. nails

D. bolts

5. I'll be back in the twinkling of ______ . A. an eye

B. a lighting bolt C. a smile

D. a laugh

B. tigers off

C. dogs off

D. apes off

N

A. hunters off

O

6. Look, I will pay you back. Would you please call the ______?!

B. lease

C. title

H

A. rent

Ơ

7. Ever since he married Jane, he's had a new ______ on life. D. loan

B. hand

Y

A. foot

N

8. Oh, I'm sorry. I shouldn't have said that. I guess I really put my ______ in my mouth. C. elbow

D. knee

U

9. You need to keep a stiff ______ no matter what! Don't get nervous! B. right arm

Q

A. nose

C. eyebrow

D. upperlip

A. real

M

10. I think you can take what he says at ______ value. B. face

C. true

D. straight

11. I wish you'd be honest with me. Don't lead me down the ______ .

ẠY

A. hiking path

B. garden path

C. hiking trail

D. garden trail

D

12. I'd like to start the meeting and get down to ______ tacks. A. silver

B. brass

C. copper

D. gold

13. I've been working here for two months and have got into the ______. A. swing of things

B. groove of things 3


C. rut of things

D. swing of routine

14. I really wasn't ready for the presentation, but was saved by the ______ when Jack took over for me. B. bang

C. bell

D. ring

FF IC IA L

A. tone

15. We need to tighten our ______ and start saving some money. A. shoes

B. tie

C. belt

D. shoelaces

16. Bob's so stupid. He doesn't know enough to come in out of the _____. A. rain

B. snow

C. wind

D. sleet

B. knife-feed

C. spoon feed

D. happily feed

N

A. fork-feed

O

17. I had to ______ the class all the answers to the test.

B. skin

C. blisters

H

A. bone

Ơ

18. She needs to take a vacation. She's been working her fingers to the _____. D. nails

B. eat

C. burn

D. cut

Y

A. throw

N

19. Ask Tom, he's got money to ______ .

B. mouse

Q

A. dog

U

20. I told you not to tell her! Now you've let the _____ out of the bag. C. fly

D. cat

1.

M

YOUR ANSWERS

3.

4.

5.

7.

8.

9.

10.

ẠY

6.

2.

Part 2. Read the text and find 5 mistakes and correct them. You should indicate

D

in which line the mistake is. (5 pts) A newspaper makes its money from the price people pay for it and also from the advertisings it carries. A popular newspaper with a circulation of over five million daily makes a lot of money. Less serious newspapers are probably read just for entertainment. They have big headings above the new stories, funny cartoons to 4


look at and sensational photos of violent. The gossip columns are full of stories of private live of famous people. No one takes the political views of such papers seriously. On the other hand, in a free country where there is no censorship, serious papers are read principle for their news, sent to them by their correspondents round

FF IC IA L

the world and by the big news agencies. People also read these papers for their

revisions of new books, films and plays, and for their editorials which represent the

opinion of the newspaper itself about the important events and issues of the moment.

Part 3. Complete each of the following sentences with a suitable preposition or

O

particle. (5 pts)

N

1. The car was going _______ full speed when the accident happened.

Ơ

2. He has already run _______ the money his father left him two years ago.

H

3. She averted her face _______ the sight of his suffering.

N

4. Well, the beautician did something to her face, and now she's _______ recognition.

2.

3.

4.

5.

M

Q

1.

U

Y

5. Apparently a number of army officers were implicated_______ the plot

Part 4 Complete the text by writing the correct form of the word in capitals. (10 pts) No matter how well-(1) (intend) _____ their bosses may be, many smokers

feel (2) (persecutor) _____ by their firms' (3) (smoke) _____, policies. “Just call me

ẠY

Sneaky Pete,” says a salesman of (4) (novel) _____ items who would face being

D

fired if his smoking habit was discovered. Says he: “It’s (5) _____ (credit) unfair. I was a smoker when they hired me, and then, out of the blue, I’m supposed to stop just because the boss says so”. Some employees fear their chances for (6) (advance) may be choked off by their smoking habit though (7) (favor) _____ toward (8) (smoke) _____ is rarely (9) (explicate) _____. Len Bell, director of human 5


resources at Pacific Northwest Bell, says a bias against smoking "could be in the back of a (10- management)'s mind when making decisions on a promotion. YOUR ANSWERS 2.

3.

4.

6.

7.

8.

9.

C. READING COMPREHENSION (60 pts)

5.

FF IC IA L

1.

10.

Part 1. For each gap, choose the correct answer A, B, C or D which best fits the

O

context. (15 pts)

One of the groups of consumers targeted by advertisers is, oddly enough, a

N

group with very little money of its own, but which has a huge (1) ______ the way

Ć

others spend their money. And in some ways, children and advertisers could have

H

been made for (2) ______. After all, it is easy to fool younger children into

N

believing (3) ______ anything and if an advertisement shows them how Biffo Breakfast Cereal will help them run faster than their friends, then (4) ______ they

Y

are concerned, that is the truth. They also have strong feelings of admiration for

U

action heroes or cartoon characters or sports stars, and will want to be (5)

Q

______their favourite star, and drink the same cola or eat the same sweets. And when children want something badly enough, they won't stop nagging their parents

M

until they (6) ______ it. Advertisements will even (7) ______them with the

KĂˆ

arguments they can use when they are told that a water-firing robot or a giant chocolate bar is not good (8) ______, or too expensive, or not available in the

áş Y

supermarket. This is why most EU countries place (9) ______upon television advertising aimed at children. Some countries have (10) ______ban on ads

D

promoting toys during children's programmes. Others restrict the advertising of unhealthy food, or ads involving anything dangerous. This seems to be a sensible way of preventing advertisers from taking (11) ______ of children, but in some ways it also helps to make a more damning (12) ______If it is generally agreed that children need (13) from some kinds of advertising, then this (14) ______to definite 6


proof that advertising strongly influences children's behaviour. In that case, why should any advertising aimed at children be allowed? After all, isn't it just another form of brain-washing? Adults may (or may not) resist the (15) ______ claims of advertisers, but children clearly have not yet learnt to do this. This is why an EUin some quarters.

FF IC IA L

wide ban on advertising targeting children, or depicting children, is being called for

1. A. cause of

B. reason why

C. influence upon D. outcome which

2. A. each other

B. the worst

C. ever

B. all in all

C. as good as

D. just about

D. supposing

B. whatsoever

C. as far as

5. A. just like

B. exactly the same

C. as is

6. A. stop

B. achieve

C. gain

D. get

7. A. offer

B. explain

C. fill

D. supply

8. A. at it

B. for them

C. with them

D. to it

9. A. handicaps

B. rules

C. restrictions

D. conditions

10. A. a total

B. an utter

C. as sheer

D. a thorough

11. A. advantage B. pity

C. an interest

D. responsibility

12. A. change

B. profit

C. point

D. contribution

13. A. care

B. preservation

C. safeguards

D. protection

14. A. comes

B. amounts

C. indicates

D. refers

15. A. sceptical

B. suspicious

C. doubtful

D. questioning

N

Ơ H N

Y

U

Q

M

ẠY

O

4. A. unless

3. A. more and more

D. the time being

D. in imitation

D

Part 2. Fill ONE suitable word in each blank. (15 pts) THE CULT OF CELEBRITY Once, children had ambitions to be doctors, explorers, sportsmen, artists or scientists. Now taking their lead from TV, they just "want to be famous". Fame is no longer a reward for gallant service or great, perhaps even selfless endeavour. It is an 7


end in (1) ______, and the sooner it can be achieved, the sooner the lonely bedroom mirror can be replaced by the TV camera and flash gun, the better. Celebrity is the profession (2 ______ the moment, a vainglorious vocation which, (3) ______ some 18th- century royal court, seem to exist largely so that the rest of us might watch and

FF IC IA L

be amazed (4) ______ its members live out their lives in public, like self-regarding members of some glittering soap opera.

Today, almost (5) ______ can be famous. Never has fame (6) ______ more democratic, more ordinary, more achievable. (7)______ wonder it's modern ambition. It's easy to see why people crave celebrity, (8) ______ generations reared

O

on the instant fame offered by television want to step out of the limousine (9)

______ the flashlights bouncing around them. Who doesn't want to be the centre of

N

attention at some time in their lives?

Ć

Modern celebrity, peopled by the largely vain and vacuous, fills a need in our

H

lives. It peoples talks shows, sells goods and newspapers and rewards the famous

N

for-well, (10) ______famous.

6.

7.

U

2.

Q

1.

Y

YOUR ANSWERS

3.

4.

5.

8.

9.

10.

KĂˆ

pts)

M

Part 3. Read the passage and choose the best answer to each of the questions. (15

In the course of its history, human inventions have dramatically

áş Y

increased the average amount of energy available for use per person.

D

Primitive peoples in cold regions burned wood and animal dung to heat their caves, cook food, and drive off animals by fire. The first step toward the developing of more efficient fuels was taken when people discovered that they could use vegetable oils and animal fats in lieu of gathered or cut wood. Charcoal gave off more intensive heat than wood and was more easily obtainable than organic fats. The Greeks first began to use coal for metal 8


smelting in the 4th century, but it did not come into extensive use until the Industrial Revolution. In the 1700s, at the beginning of the Industrial Revolution, most energy used in the United States and other nations undergoing industrialization was

FF IC IA L

obtained from perpetual and renewable sources, such as wood, water streams,

domesticated animal labor, and wind. These were predominantly locally

available supplies. By mid-1800s, 91 percent of all commercial energy consumed in the United States and European countries was obtained from wood.

However, at the beginning of the 20th century, coal became a major energy

O

source and replaced wood in industrializing countries. Although in most regions

and climate zones wood was more readily accessible than coal, the latter

N

represents a more concentrated source of energy. In 1910, natural gas and oil

Ć

firmly replaced coal as the main source of fuel because they are lighter and,

H

therefore, cheaper to transport. They burned more cleanly than coal and polluted

N

less. Unlike coal, oil could be refined to manufacture liquid fuels for vehicles, a very important consideration in the early 1900s, when the automobile arrived on

Y

the scene.

U

By 1984, non-renewable fossil fuels, such as oil, coal, and natural gas,

Q

provided over 82 percent of the commercial and industrial energy used in the

M

world. Small amounts of energy were derived from nuclear fission, and the remaining 16 percent came from burning direct perpetual and renewable fuels,

KĂˆ

such as biomass. Between 1700 and 1986, a large number of countries shifted from the use of energy from local sources to a centralized generation of

áş Y

hydropower and solar energy converted to electricity. The energy derived from

D

non-renewable fossil fuels has been increasingly produced in one location and transported to another, as is the case with most automobile fuels. In countries with private, rather than public transportation, the age of non-renewable fuels has created a dependency on a finite resource that will have to be replaced. Alternative fuel sources are numerous, and shale oil and hydrocarbons 9


are just two examples. The extraction of shale oil from large deposits in Asian and European regions has proven to be labor consuming and costly. The resulting product is sulfur-and nitrogen-rich, and large scale extractions are presently prohibitive. Similarly, the extraction of hydrocarbons from tar sands

FF IC IA L

in Alberta and Utah is complex. Semi-solid hydrocarbons cannot be easily separated from the sandstone and limestone that carry them, and modern

technology is not sufficiently versatile for a large-scale removal of the material. However, both sources of fuel may eventually be needed as

petroleum prices continue to rise and limitations in fossil fuel availability

O

make alternative deposits more attractive.

N

1.What is the main topic of the passage?

B. Natural resources and fossil fuels

Ơ

A. Application of various fuels

D. A historical review of energy rates

H

C. A history of energy use

B. per year

C. per family

D. per day

Y

A. per capita

N

2.The phrase “per persion” is close in meaning to

U

3.It can be inferred from the first paragraph that ______.

Q

A. coal mining was essential for primitive peoples

M

B. the Greeks used coal in industrial productions

C. the development of efficient fuel was a gradual process D. the discovery of efficient fuels was mostly accidental

D

ẠY

4. The phrase “in lieu” is closest in meaning to A. in spite

B. in place

C. in every way

D. in charge

5 .The author of the passage implies that in the 1700s, sources of energy were ______. A. used for commercial purposes B. used in various combinations C. not derived from mineral deposits 10


D. not always easy to locate 6 .According to the passage, what was the greatest advantage of oil as fuel? A. It was a concentrated source of energy.

FF IC IA L

B. It was lighter and cheaper than coal. C. It replaced wood and coal and reduced pollution. D. It could be converted to automobile fuel.

7. According to the passage, the sources of fossil fuels will have to be replaced because _____ .

B. they are not efficient

C. their use is centralized

D. their supply is limited.

N

O

A. they need to be transported

Ơ

8. It can be inferred from the passage that in the early 20th centurgy, energy was

H

obtained primarily from______.

B. nuclear fission

N

A. fossil fuels

D. burning biomass

Y

C. hydraulic and solar sources

U

9. The author of the passage implies that alternative sources of fuel are currently ______. B. being explored

C. available in few locations

D. examined on a lar

M

Q

A. being used for consumption

ẠY

10. The word “prohibitive” is closest in meaning to A. prohibited

B. provided

C. too expensive

D. too expedient

D

YOUR ANSWERS 1.

2.

3.

4.

5.

6.

7.

8.

9.

10.

11


Part 4. Read the following passage and do the tasks that follow. (15 pts) EATS, SHOOTS AND LEAVES -A book review-

FF IC IA L

The title of Eats, Shoots and Leaves refers to a famously misplace comma in wildlife manual that ended up suggesting a panda rather violently "eats, shoots and

leaves" instead of eating shoots and leaves. The author of this book, journalist Lynne Truss, is something akin to a militant linguist dedicating this “zero

tolerance” manifesto on grammar to the striking Bolshevik printers of St.

Petersburg who, in demanding the same remuneration for punctuation as they

O

received for letters, ended up setting in motion the first Russian Revolution.

N

Some of the book involves humorous attacks on erroneous punctuation. There is the

Ơ

confused Shakespearian thespian who inadvertently turns a frantic plea: “Go, get him surgeons!” into the cheerful encouragement of “ Go get him, surgeons ! ” Street and shop

H

signs have a ubiquitous presence. A bakery declares “FRESH DONUT’S SOLD HERE”

N

and a florist curiously announces that “Pansy’s here!” (Is she?). The shameless title of a

Y

Hollywood film Two Weeks Notice is reeled in for criticism—“Would they similarly call it

U

One Weeks Notice?", Truss enquires—and sometimes, as in the case of signs promoting

Q

“ANTIQUE’S” and “Potatoe’s”—one questions whether we are bearing witness to new depths of grammar ignorance, or a postmodern caricature of atrocious punctuation.

M

Eats, Shoots and Leaves is not just a piece of comedy and ridicule, however, and

Truss has plenty to offer on the question of proper grammar usage. If you have ever wondered whether it is acceptable to simply use an “em dash”1 in place of a comma—the

ẠY

verdict from Truss is that you can. “The dash is less formal than the semicolon, which makes it more attractive,” she suggests. “It enhances conversational tone; and...it is capable

D

of quite subtle effects.” The author concludes, with characteristic wry condescension, that the em dash’s popularity largely rests on people knowing it is almost impossible to use incorrectly. Truss is a personal champion of the semicolon, a historically contentious punctuation mark elsewhere maligned by novelist Kurt Vonnegut Jr., as a “transvestite

12


hermaphrodite representing absolutely nothing”. Coming to the semicolon’s defense, Truss suggests that, while it can certainly be over-used—she refers to the dying words of one 20th century writer: “I should have used fewer semicolons”—the semicolon can perform the role of a “a kind of Special Policeman in the event of comma fights”.

FF IC IA L

Truss has come under criticism on two broad points. The first argument criticises

the legitimacy of her authority as a punctuation autocrat. Louis Menand, writing in the New Yorker, details Eats, Shoots and Leaves’ numerous grammatical and punctuation sins: a comma-free non-restrictive clause; a superfluous ellipsis; a misplaced apostrophe; a misused parenthesis; two misused semicolons; an erroneous hyphen in the word “abuzz”,

O

and so on. In fact, as Menand notes, half the semicolons in the Truss book are spuriously deployed because they stem from the author’s open flouting of the rule that semicolons

N

must only connect two independent clauses. “Why would a person not just vague about the

Ơ

rules but disinclined to follow them bother to produce a guide to punctuation?” Menand

H

inquires. Ultimately, he holds Truss accused of producing a book that pleases those who

N

“just need to vent” and concludes that Eats, Shoots and Leaves is actually a tirade against the decline of language and print that disguises itself, thinly and poorly, as some kind of a style

Y

manual.

U

Linguist David Chrystal has criticised what he describes as a “linguistic purism”

Q

coursing through Truss’ book. Linguistic purism is the notion that one variety of language

M

is somehow more pure than others, with this sense of purity often based on an idealised historical point in the language’s development, but sometimes simply in reference to an

abstract ideal. In The Fight for English: How Language Pundits Ate, Shot and Left, Chrystal—a former colleague of Truss—condemns the no-holds-barred approach to punctuation and grammar.

ẠY

“Zero tolerance does not allow for flexibility,” he argues. “It is prescriptivism taken to

D

extremes. It suggests that language is in a state where all the rules are established with 100 per cent certainty. The suggestion is false. We do not know what all the rules of punctuation are. And no rule of punctuation is followed by all of the people all of the time.” Other detractors of Truss’ “prescriptivism” are careful to disassociate needless

purism from robust and sensible criticism, an oppositional stance they call descriptivism. 13


“Don’t ever imagine,” Geoffrey K. Pullum on the Language Log emphasises, “that I think all honest attempts at using English are just as good as any others. [Bad] writing needs to be fixed. But let’s make sure we fix the right things.” In other words, we do not require a dogmatic approach to clean up misused language. Charles Gaulke concurs, noting that his

FF IC IA L

opposition to “prescriptivism” does not require contending with the existence of standards themselves, but questioning whether our standards should determine what works, or whether what works should determine our standards.

Ultimately, it is unlikely the purists and pedagogues will ever make absolute peace with those who see language as a fluid, creative process within which everyone has a role

O

to play. Both sides can learn to live in a sort of contentious harmony, however. Creativity typically involves extending, adapting and critiquing the status quo, and revising and

N

reviving old traditions while constructing new ones. Rules must exist in order for this

Ơ

process to take place, if only for them to be broken. On the flip side, rules have an

H

important role to play in guiding our language into forms that can be accessed by people

N

across all manner of differences, so it is vital to acknowledge the extent to which they can be democratic, rather than merely autocratic in function. Nevertheless, all the regulations in

Y

the world cannot stem the natural spring of language, which bursts through rivets and

U

snakes around the dams that linguistic authorities may try to put in place. We should

Q

celebrate rather than curse these inevitable tensions.

M

Note: 1. Em dash = _____

Question 1-6

Look at the following statements and the list of people below. Match each statement with

ẠY

the correct person, A-E from the list.

D

NB: You may use any letter more than once 1. Mistakes should be corrected on the basis of common sense. 2. No one has legitimacy as an ultimate authority on punctuation use. 3. Eats, Shoots and Leaves is not the type of book it claims to be.

4. The idea that some forms of language can be better than others is wrong. 14


5. The semicolon has no real purpose. 6. We can ask whether rules are helpful without undermining the need for rules. List of People

FF IC IA L

A. Kurt Vonnegut Jr. B. Louis Menand C. David Chrystal D. Geoffrey K.Pullum

O

E. Charles Gaulke

N

Question 7-10. Complete the summary below.

Ơ

Choose NO MORE THAN THREE WORDS from the passage for each answer.

H

Eats, Shoots and Leaves is a book on punctuation by journalist Lynne Truss,

N

who could be described as a (7) ______ . She dedicates the book to the Bolshevik

Y

Printers who started the (8) ______ by protesting for better pay conditions. The

U

book is partly a humorous criticism of incorrect punctuation. Some of the examples

Q

are so bad it is possible that they are actually a (9) ______. Truss also guides the reader on correct punctuation usage. She likes the em

M

dash because it is not as formal as the semicolon, for example, but remains a (10)

______ of the latter due to its ability to discipline areas of text that are crowded with commas.

D

ẠY

YOUR ANSWERS 1.

2.

3.

4.

5.

6.

7.

8.

9.

10.

D. WRITING (40 pts) 15


Part 1. Rewrite the following sentences beginning as shown or using the word given. Do not change the meaning of the original sentences. (5 pts) 1. The journalists only heard about the changes to the wedding plans when they arrived at the venue.

FF IC IA L

It was only ___________________________________________________ 2. We only came to this restaurant because you insisted that we did so.

It was at _____________________________________________________ 3. Arguing with her won’t get you anywhere.

N

4. The thief must have come in through the window.

O

It won’t do ____________________________________________________

Ơ

The thief almost _______________________________________________

H

5. What put me off the idea was simply how expensive it was going to be.

N

The sheer _____________________________________________________

Y

Part 2. Graph description (20 pts)

U

The pie charts below give information about the household expenditure of two

Q

average U.S. families in different years. Summarise the information by selecting and reporting the main features, and make

M

comparisons where relevent.

D

ẠY

You should write at least 150 words.

16


............................................................................................................................ ............................................................................................................................ ............................................................................................................................

FF IC IA L

............................................................................................................................ ............................................................................................................................

............................................................................................................................

............................................................................................................................

............................................................................................................................

O

............................................................................................................................

N

............................................................................................................................

Ơ

............................................................................................................................

H

............................................................................................................................

N

............................................................................................................................

Y

............................................................................................................................

U

............................................................................................................................

Q

............................................................................................................................

M

Part 3. Essay writing (35 pts)

"Some people think that the detailed criminal description on newspaper

and TV has bad influences, so this kind of information should be restricted on

D

ẠY

the media."

Do you agree or disagree with this statement? Your essay should be about 250 words.

............................................................................................................................ ............................................................................................................................ ............................................................................................................................ 17


............................................................................................................................ ............................................................................................................................ ............................................................................................................................

FF IC IA L

............................................................................................................................ ............................................................................................................................

............................................................................................................................

...................................................................................................................................... .............................................................................................................................

O

............................................................................................................................

N

......................................................................................................................................

Ơ

......................................................................................................................................

H

..........................................................................................................................

N

...................................................................................................................................... ......................................................................................................................................

Y

..........................................................................................................................

U

......................................................................................................................................

Q

......................................................................................................................................

M

..........................................................................................................................

...................................................................................................................................... ......................................................................................................................................

ẠY

.......................................................................................................................... ......................................................................................................................................

D

...................................................................................................................................... GV ra đề: Đặng Thị Hương

ĐT: 0988 054 726 18


HỘI CÁC TRƯỜNG THPT CHUYÊN

ĐÁP ÁN KÌ THI HỌC SINH GIỎI

VÙNG DUYÊN HẢI VÀ ĐỒNG

NĂM HỌC 2015-2016

BẰNG BẮC BỘ

MÔN THI: TIẾNG ANH LỚP 11

FF IC IA L

Ngày thi: 16 tháng 4 năm 2016 TRƯỜNG THPT CHUYÊN BẮC

Thời gian làm bài: 180 phút

GIANG

(không kể thời gian giao đề)

ĐỀ THI ĐỀ XUẤT A. LISTENING (40 pts)

O

Part 1: 10pts

N

1. D 2. B 3. B 4. A 5. D

Ơ

Part 2: 10pts 1. F 2. F 3. T 4. F 5. T

H

Part 3: 20pts

Y

1. Body/ body art

N

complete ielt

U

4. story(-) telling/telling stories

2. (the) weather

3. animal tracks

5. iron oxide

Q

6. 18000/18,000/ 18 000 years 7. powder

8. bush honey

9. Canvas

M

10. (musical) instruments

B. VOCABULARY AND GRAMMAR (60 pts) PART 1: 10pts

ẠY

1. B 2. B 2. C 4. D 5. A 6. C 7. B 8. A 9. D 10. B 11. B 12. B 13. A 14. C 15. C 16. A 17. C 18. A 19. C 20. D

D

PART 2: 5pts 1. advertisings advertising 2. Violent violence 3. live lives 4. principle principally 19


5. revisions reviews PART 3: 5pts 1. at 2. Through 3. From

4. Beyond

5. in

1. intentioned

2. Persecuted

3. Antismoking

4. Novelty

5. Incredibly

6. Advancement

7. favouritism

8. Nonsmokers

9. Explicit

10. manager

C. READING COMPREHENSION (60 pts)

O

PART 1: 15pts

FF IC IA L

PART 4: 10pts

5. A 6. D 7. D 8. B 9. C 10. A 11. A 12. C 13. D

N

1. C 2. A 3. D 4. C PART 2: 15pts

2. Of

3. Like

H

1. Itself

Ơ

14. B 15. C

N

5. anyone/anyobody/everyone/everybody

6. seemed/been/appeared

8. Why

Y

7. No/small/little

4. As/while

10. being

Q

PART 3: 15pts

U

9. with

1. C 2. A 3. C 4. B 5. C 6. D 7. D 8. A 9. B 10. C 2C

3. B 4. C 5. A 6. E

1. D

M

PART 4: 15pts

7. militant linguist 8. First Russian Revolution

9. postmodern caricature

ẠY

10. personal champion D. WRITING (40 pts)

D

PART 1: 5pts 1. It was only when the journalists arrived at the venue that they heard about the changes to the wedding plans. 2. It was at your insistence that we came to this restaurant. 3. It won’t do you any good to argue with her. 20


4. The thief almost certainly came in through the window.

D

ẠY

M

Q

U

Y

N

H

Ơ

N

O

FF IC IA L

5. The sheer expense/ cost was what put me off the idea.

21


LISTENING TRANSCRIPT PART 1: CPE 4- TEST 4 You will hear an interview with Ronald Brundy, the new chairman of television channel GTV

FF IC IA L

Healther: The appointment of Roland Brundy as chairman of GTV has been widely welcomed. I spoke to him earlier this week. Roland, ten years is a relatively long

time in the life of a television channel. What trends do you think are going to have an impact on GTV during your reign as chairman?

Brundy: Well I think one of the most obvious is technology - the advent of digital radio and television and of course the increase in the number of channels in Britain.

O

And all that is closely linked to competition; more channels means more choice for

N

the consumer.

Ơ

Healther: Whom do you regard as the competition?

Brundy: Everybody. The other channels, independent radio companies, satellite.

H

It’s by no means all bad, it forces GTV to sharpen up its act and with all the extra

Y

one-way street if you like.

N

activity, it increases the pool of talent and ideas available, so competition isn’t, er, a

U

Healther: So how do you think the competition should be tackled? Is it a case of

Q

trying to do the same thing except more or better or should you compete by offering something different - or should you do both?

M

Brundy: I don’t think you can cover all angles. I think either you say you're going

to take the competition head on and here’s where we're going to find the resources to do it, this is where we’re going to match the exponentially rising costs of

ẠY

sporting events, say, and films. Or you say we're not going to take them head on; we don’t have the resources to do it, so we’ll do something different.

D

Healther: You’ve mentioned sport and films - is that because they’re areas where you're losing viewers fo other channels or... Brundy: Acquiring sporting events and movies is actually easy. You just write a bigger cheque than anyone else's. But it's not so easy to have the sort of standing you want in the arts - in drama - even if it’s mainstream stuff, er, or something like 22


current affairs. They require creativity, a corpus of people, a history - it all takes time, you can't just buy it. Healther: So, how you react to competition can be a difficult decision to make. Brundy: It can put you in a very invidious position. On the one hand, of course, you

FF IC IA L

can do nothing and then people turn around and say look at fuddy duddy old GTV, they haven’t even noticed what’s going on in the market - they haven’t changed

anything. But if you do change or adapt you can get accused of copying - but who’s copying whom?

Healther: Do you have a world view on technological change?

Brundy: A world view? Beware of broadcasters with world views! No, no I don't. I

O

think you have to take note of what - happening - all the time - but restrain any

N

fears you may have that it's ail going to happen tomorrow as we're led to believe.

Ơ

J'm dubious about the pace of technological change and the way it affects broadcasters and viewers and I've learnt through the experience of the last ten years

H

that technology doesn’t drive this industry - in the end it serves it.

N

Healther: What do you think the main differences are between running a television

Y

channel and running any other type of business?

U

Brundy: Well, the first that springs to mind always is how you gauge your

Q

achievement, ft’s much less precise in this field - not less important but much harder to quantify. GTV has actually done some quite sensible things in trying to

M

ascertain such things but where that hasn’t been possible it's tried to identify the

objectives it’s aiming to achieve. I think the other difference is that GTV is a more important place than anywhere I’ve worked, it touches every aspect of life in this

ẠY

country. I mean without being too pompous, it's far more significant than the

D

printing business, or a weekend commercial television company. PART 2 A report from the USA says American children are safer today than they were 20 years ago. The study is based on the annual Child Well-Being Index, which has been compiled since 1975. It measures 28 aspects about the quality of life among US children. Perhaps the most promising figure is that child death rates have gone 23


down by a third since 1975. Suicide rates among teens have declined and fewer youngsters are victims of violent crimes. Teen births are at their lowest level in decades. The report says US children are also better educated than they were 20 years ago.

Read

FF IC IA L

The report highlighted some negatives of the past 20 years. Child health has deteriorated. A major cause of this is the increasing levels of obesity because of poor diet, fast food, and less exercise. Teenagers are spending much longer indoors using technology. This may protect kids from physical dangers outdoors but can harm their health indoors. Child poverty still affects millions of children in the USA. This has not changed in the past 20 years. About 20 per cent of children lived below the poverty line in 2013. This is about the same as it was in 1995. more:http://www.breakingnewsenglish.com/1412/141225-childhood-

O

a.html#ixzz42MWCC8Nz

Ơ

N

PART 3- IELTS COMPLETE: BAND 6.5-7.5: UNIT 4

H

Were going to have a look today at Aboriginal art and paining, which actually dates

N

back 60.000 years, making it one of the oldest art traditions in the world. Now. as long as indigenous people have been living in Australia, they’ve been creating

Y

different types of art So let's start by having a look at some examples of ancient art.

U

It includes things like, as you can see here, rock paintings, bark paintings . even

Q

some sand drawings like this have been found. Then there's the whole area of body

M

art. which is so important for ceremonial practice, and lastly, here are some examples of decorative art on weapons and tools.

The oldest art examples today are the rock paintings because, obviously, rock is more durable than other materials and so the art has been preserved. In fact, most of

ẠY

this work is inside caves - largely because there, it's been sheltered, hasn't been

D

destroyed by the weather, while the paintings on outside rock surfaces have often been washed away over the years. Now. there are enormous variations in the style of Aboriginal rock art. depending on its age and location. Dot paintings are one of the best-known visual art forms of Aboriginal culture in which a surface is covered in small dots to reveal symbols Typical ones include arrows like this - here's a water hole, and these are animal tracks. You get to see both the abstract dot 24


paintings and more naturalistic art... you get both in rock art of various ages. As the ancient Aborigines didn’t have a written language, the key purpose of much of this rock art was storytelling, which has had a great significance for younger generations.

FF IC IA L

Let’s move on to look at the materials. Er. whatever they were painting, traditionally Aboriginal people all over Australia used pigment, such as ochre, to make paint. Ochre’s very finely textured natural rock and. um. well, they used this

because ochre is plentiful across most of Australia. It’s coloured by iron oxide,

which is the mineral that makes a lot of Australian outback soil - in places such as Ayers Rock - what is known as ’Uluru red’, Uluru being the Aboriginal name for

O

Ayers Rock. However, depending on the exact conditions under which it formed,

N

the shade can be anything from yellow to orange, red. purple or dark brown. Today,

Ơ

ochre occurs in many archaeological sites, and archaeologists at one site have discovered what appears to be an artist's palette of ochres, dating back 18,000

H

years.

N

Preparing the ochre paints was time-consuming work. First, the appropriate rocks

Y

had to be found and collected. Then the rock had to be broken up and ground into a

U

powder, and that had to be mixed with some sort of fluid to bind it into paint.

Q

Nowadays, the binder most commonly used is professional artist’s acrylic binder, but in the past. Aboriginal people used things like tree sap. or something similar

M

like bush honey.

Other fluids must also have been used but wouldn’t have held paint on rock or a piece of bark for thousands of years, so sadly those paintings would have been lost

ẠY

So. how have things changed? Well, modern Aboriginal is a mixture of the old and the new. Things changed in the 1970s really when Aboriginal people from many

D

different parts of Australia, particularly south Australia, central and northern Australia, took up acrylic painting and began to paint on canvas Taking a modern approach has had many advantages. It saves artists a great deal of time, and they can still choose to use the traditional yellowish-reddish-brownish colours if they wish to. But perhaps the most important fact is that, unlike bark and 25


rock paintings, the modern paintings are easy to sell. In fact, painting on canvas has given Aboriginal people an opportunity to showcase their art to the world and keep their ancient culture alive. Modern Aboriginal art. particularly dot painting, has taken off and started selling on a big scale internationally. Aboriginal art can also

FF IC IA L

be found on pottery and various musical instruments like didgeridoos and clapping sticks. Together, these have become some of the most popular souvenirs in

Australia. Their artists, like other artists in the world, are now able to earn a living

D

áş Y

KĂˆ

M

Q

U

Y

N

H

Ć

N

O

doing something they are passionate about.

26


KỲ THI CHỌN HSG

TRƯỜNG THPT CHUYÊN BẮC NINH

KHU VỰC DH-ĐBBB

Tổ Ngoại ngữ

Năm học: 2015 – 2016

ĐỀ ĐỀ NGHỊ

ĐỀ THI: TIẾNG ANH LỚP 11

-------------------

(Thời gian làm bài: 180 phút)

FF IC IA L

SỞ GD&ĐT BẮC NINH

PART A. LISTENING (50pts):

• Bài nghe gồm 3 phần, mỗi phần được nghe 2 lần, mỗi lần cách nhau 15 giây, mở đầu và kết thúc mỗi phần nghe có tín hiệu.

O

• Mở đầu và kết thúc có tín hiệu nhạc. Thí sinh có 3 phút để hoàn chỉnh bài trước tín hiệu kết thúc bài nghe.

Ơ

N

• Mọi hướng dẫn cho thí sinh (bằng tiếng Anh) đã có trong bài nghe.

H

I. For questions 1-5, you are going to hear part of a radio programme. Decide

N

which of the sentences are true (T), and which are false (F) according to the recording. Write your answers in the corresponding numbered boxes. The first

Y

(0) has been done as an example.

U

0. Some people in the Outback live over 100 kilometres away from their nearest

Q

neighbours. world.

M

1. Everybody in the Outback has at least a telephone to keep in touch with the

2. In an emergency, doctors use jeeps to get to the patient. 3. It doesn’t take the doctors more than ninety minutes to get to any place in the

ẠY

Outback.

4. Children living in the Outback do not go to school at all.

D

5. The children in the Outback can communicate with their teachers by two- way radio and post. Your answers: 0. T

1.

2.

3.

4.

5.

1


II. For questions 6-15, listen to a piece of news from the BBC about the “bloodbath” in Northern Sri Lanka and fill in the missing information. Write NO MORE THAN THREE WORDS taken from the recording for each answer in the

FF IC IA L

spaces provided. 6. According to the UN's spokesman Gordon Weiss, more than 100 children died during what he called the large-scale _______________over the weekend.

7. The UN had warned against ______________ and the weekend's events showed that 'that bloodbath has become a reality'.

8. A doctor working in the war zone said that the bodies of 378 people had been

O

registered at his hospital and that many other _______________were lying outside.

N

9-10. He said _______________ appeared to have been fired from ______________

Ơ

into a mainly civilian area under Tamil Tiger rebel control.

H

11. The government said the issue of ______________is highly sensitive here and the state-owned Daily News on Monday makes no mention of the incident at all.

N

12. In New York, the British Foreign Secretary David Miliband is to

Y

_______________informal discussions.

U

13. The discussions with ministers and charities are on _______________situation.

Q

14. The Sri Lankan government is dismissive of calls from Secretary David Miliband and other diplomats _________________in the North.

M

15. And the Sri Lankan government says it's about to defeat the rebels permanently

and _________________would not help civilians.

D

ẠY

Your answers:

6. _______________________

7._______________________

8.__________________________

9.________________________

10._________________________

11.______________________

12._________________________

13.______________________

14. ________________________

15._____________________

2


III. For questions 16-20, listen to a radio interview with a chef about the process of eating and choose the best answer (A, B, C, or D) according to what you hear. Write your answers in the corresponding numbered boxes. A. because it is something listeners may have done.

FF IC IA L

16. Heston mentions eating fish from a paper plate with a plastic knife and fork B. because doing so made him think about the process of eating. C. as an example of an unpleasant eating experience.

D. as an example of what influences the eating experience 17. What does Heston say about taste?

C. The sense of smell is involved in it.

N

B. Taste and flavour are separate from each other.

O

A. Fat should be considered a taste.

Ơ

D. The number of taste buds gradually decreases.

H

18. The experiment involving salt and other food shows that A. it is possible to taste something that you can’t smell.

N

B. the sense of smell is not as powerful as other senses.

Y

C. food can taste better when you can’t smell it.

U

D. the flavour of food can change as you eat it.

Q

19. The story about the trainee waiters illustrates that A. certain colours are more appealing than others.

M

B. something can seem to taste good because of its appearance.

C. one sense can strongly influence another. D. some people can perceive taste better than others.

ẠY

20. What does Heston say about bitterness? A. It can give a false impression that something is harmful.

D

B. It can become the main reason why people like something. C. Reactions to it can change over time. D. Its function is widely misunderstood.

Your answers: 16.

17.

18.

19.

20. 3


PART B. LEXICO-GRAMMAR (30 pts) I. Choose the best answer (A, B, C or D) to complete each of the sentences. (10 pts) 1. Many supporters were _______ from the stadium as it was already full. B. turned away

C. put off

D. played off

C. drag

D. pluck

FF IC IA L

A. sent off

2. Why do I always ________ the short straw? A. draw

B. pull

3. Without my glasses, my vision is very ________. A. fogged

B. blurred

C. misted

D. faint

4. It was decided that the cost of the project would be __________ and so it was B. prohibitive

C. restrictive

N

A. repressive

O

abandoned.

D. exclusive

B. launched

C. cropped

H

A. struck

Ơ

5. We _________ up a friendship the very first time we met. D. settled

6. Look, will you stop __________ in and let me finish my sentence! B. pushing

N

A. moving

C. butting

D. plugging

Y

7. The man gave a series of ___________ answers which told them nothing more.

Q

C. evasive

U

A. tricky

B. uncertain D. elusive

8. The announcement about job losses has _______ consequences for the firm. B. far-reaching

C. never-ending

D. wide-ranging

M

A. hard-hitting

9. The decision was _________ to a later meeting. B. arranged

C. deferred

D. delayed

ẠY

A. cancelled

10. “Don’t worry about your sister. I’m sure she’s okay”.

D

“But it’s unlike her ______ to me every week.”

A. if she didn’t write

B. if not writing

C. not writing

D. not to write

11. The ______ listened attentively to every word the vicar said in his sermon. A. sightseers

B. congregation

C. spectators

D. audience 4


12. In today’s paper it ________ that we shall have an election this year. A. says

B. admits

C. expresses

D. proposes

13. _____ calculations have shown that the earth’s resources may run out before the end of the next century. B. Rude

C. Crude

D. Blunt

FF IC IA L

A. Raw

14. The closets are filled with things we don’t need. We should have a _____soon. A. garage sale

B. wholesale

C. housekeeper

D. doggy bag

15. Hello! Aviation Electronics? Could I speak to Mr. John on _____ 8183, please? B. extension

C. system

D. Exchange

O

A. branch

N

16. Now’s a __________ time to tell me you’re going out this evening- I’ve spent B. reasonable

C. right

D. fine

H

A. suitable

Ơ

the whole day preparing supper for you.

17. He often forgets to do what he has been told and is scolded for being

N

__________.

B. impertinent

Y

A. disobedient

C. malicious

D. insolent

U

18. The peasant’s many __________ resulting from ill-treatment by their landlords A. grieves

Q

led finally to rebellion.

B. grumbles

C. grievances

D. complaints

M

19. I’d like to buy that glass vase you have in the window - the one with the roses

__________ on the side.

A. inscribed

B. etched

C. designed

D. tattooed

ẠY

20. You’ve lived in the city for most of your life, so __________ you’re used to the noise. B. presumably

C. allegedly

D. predictably

D

A. apparently

Your answers 1.

2.

3.

4.

5.

6.

7.

8.

9.

10.

11.

12.

13.

14.

15.

16.

17.

18.

19.

20. 5


II. The passage below contains 5 mistakes. Underline the mistakes and correct them. (5 pts) Humans have struggled with weeds since the beginnings of agriculture. The global need for weed control has been answered mainly by the chemical industry.

FF IC IA L

Its herbicides are effective and sometimes necessary, but some pose serious

problems, particularly if disuse. Toxic compounds threaten animal and public health when they accumulate in food plants, ground water, and drinking water. They also harm workers who apply them.

In recent years, the chemical industry has introduced several herbicides that

are more ecologically sound. Yet new chemicals alone cannot solve the world

O

weed problems. Therefore, scientists are exploring the innate weed-killing

N

powers of living organisms, primarily insects and microorganisms.

Ơ

The biological agents now in use are harmless to humans and are

H

environmentally benign. They can be chosen for their ability of attacking selected targets and leave crops and other plants untouch. In contrast, some of the most

N

effective chemicals kill virtually all the plants they come in contact with, sparing

Y

only those that are naturally resistant or have been genetically modified for

Q

U

resistance.

III. Fill in each of the blanks a suitable preposition. (5 pts)

M

1. John and George were both vying ________ her attention at the party.

2. He was passed _______ in the first time he applied for promotion, but he seems to stand better chance this time.

ẠY

3. The sun broke _____ at last in the afternoon. 4. It’s a huge risk, and we can’t afford to slip _____ or everything will go wrong.

D

5. Someone who robs an old lady of all her savings is _____ contempt and deserves to be punished. Your answers 1.

2.

3.

4.

5.

6


IV. Write the correct form of the word given in brackets. (10 pts) The entire ecological system on Earth can be thought as one huge living (1) .................. (ORGAN). It is composed infinitesimal number of (2) ......................... (DEPENDENCE) units that all play their part in contributing to the well–being and

FF IC IA L

functioning of the whole. We human beings are, of course, a part of this (3)

................... (INTRICACY) web of life. Unfortunately, we often forget we are (4) .................. (EXTRICABLE) linked to nature, and by doing so, (5) ......................

(ADVERT) contribute to its slow destruction. Survival will depend on our (6) ................... (WILL) to reorganize our political thinking.

No longer can those who ignore nature’s (7) ..................... (WARN) continue

O

to bury their heads in the sand. Unless these politicians (who, in (8) ..................

N

(DEMOCRACY) countries, are supposed to listen to the people as well as to

Ơ

corporations) do something about the enormous environmental problems facing the

H

Earth, they will cease to be (9) ...................... (RESPECT), and this will mean our old system of government will (10) ................... (EVITABLE) change and collapse.

6.

7.

Y

2.

Q

1.

U

Your answers

N

Nothing can save them.

3.

4.

5.

8.

9.

10.

M

PART C. READING (60 pts)

I. Read the text below and decide which answer (A, B, C or D) best fits each space. (15 pts)

ẠY

Reports that the government is about to (1)______ the go ahead to plans for the

building of a new runway at London's Gatwick airport have angered (2)_____

D

residents and raised (3)_____ of increased noise and exhaust pollution. The (4)_____ plans also include permission for additional night flights and will (5)_____ the compulsory purchase of farmland, as well as the demolition of a number of private homes. (6)______ to sources close to the Ministry of Transport, the government is known to be (7)_____ by the increasing volume of traffic at London 7


Heathrow, where there are no plans for further runways in the foreseeable (8)______. Gatwick is widely regarded (9)______ a better prospect for expansion than London's third airport, Stansted, which still suffers from poor transport links. A spokesperson for the Keep Gatwick Quiet association, (10)______ up of local

FF IC IA L

people, accused the government of going back (11)______ promises made before

the General Election. “We were told then that the airport authority had no intention of building another runway, and we believe that the government has a duty to

(12)______ by its pledges.” (13)______ figures in the government are also believed to be concerned at the news, although the Prime Minister, interviewed last night, is (14)_____ as saying that reports were 'misleading'. However, he would not give an B. make

C. give

D. perform

2. A. airline

B. local

C. particular

3. A. money

B. views

C. percentages

N

1. A. have

Ơ

D. with

H

4. A. controversial B. debatable

D. fears

C. notorious

D. undecided

C. assume

D. need

C. According

D. Used

B. request

6. A. Next

B. Up

7. A. pleased

B. concerned

C. divided

D. important

8. A. years

B. events

C. time

D. future

B. for

C. because

D. as

10. A. made

B. woken

C. taken

D. formed

11. A. from

B. to

C. on

D. with

12. A. perform

B. act

C. vote

D. stand

C. Prevalent

D. Petulant

M

Q

U

Y

N

5. A. involve

9. A. and

ẠY

13. A. Protuberant B. Prominent

D

O

(15)______ that plans for building a runway had definitely been rejected.

14. A. known

B. believed

C. quoted

D. written

15. A. estimate

B. objection

C. assurance

D. inquiry

Your answer: 1.

2.

3.

4.

5.

11.

12.

13.

14.

15.

6.

7.

8.

9.

10.

8


II. Fill each of the numbered blanks in the passage with one suitable word. (15 pts) Agriculture is the world’s most important industry. It provides us with 1

_______ all our food. It also supplies 2_______ for two other basic human needs -

FF IC IA L

clothing and shelter. In addition, agriculture provides materials 3_______ in making many industrial products, such as paints and medicines. About half the world’s workers are employed in agriculture far more than 4_______ any other industry.

Agriculture is one of the world’s oldest industries. It began about 10,000

years ago in the Middle East. 5_______ that time, certain Middle Eastern tribes had discovered how to grow plants from seeds and how to raise animals in captivity. 6

O

________ mastered these skills, they could begin to practice agriculture.

N

Before the development of agriculture, people got all their food by gathering

Ơ

wild plants, hunting, and fishing. They had to search for food continually, 7_______ left them little time for other activities. But as agriculture developed and farm

H

8

________ increased, fewer people were needed to produce food. The non-farmers

N

could then develop the arts, crafts, trades and other activities of civilized life.

1.

2.

3.

4.

5.

7.

8.

9.

10.

________

M

6.

10

U

Your answers

Q

civilization possible.

Y

Agriculture 9_______ not only greatly most the food supply but also

III. Read the following passage and choose the best answer (A, B, C or D) to

ẠY

each question. (15pts) MUSICAL TALENT

D

Among all the abilities with which an individual may be endowed, musical

talent appears earliest in life. Very young children can exhibit musical precocity for different reasons. Some develop exceptional skill as a result of a well- designed instructional regime, such as the Suzuki method for the violin. Some have a good fortune to be born into a musical family in a household filled with music. In a 9


number of interesting cases, musical talent is part of an otherwise disabling condition such as autism or mental retardation. A musical gifted child has an inborn talent; however, the extent to which the talent is expressed publicly will depend upon the environment in which the child lives.

FF IC IA L

Musically gifted children master at an early age the principal elements of

music, including pitch and rhythm. Pitch- or- melody - is more central cultures, for example, in Eastern societies that make use of tiny quarter- tone interval. Rhythm,

sounds produced at certain auditory frequencies and grouped according to a prescribed system, is emphasized in sub- Saharan Africa, where the rhythmic ratios can be very complex.

O

All children have some aptitude for making music. During infancy, normal

N

children sing as well as babble, and they can produce individual sounds and sound

Ơ

patterns. Infants as young as two months can match their mother’s song in pitch,

H

loudness, and melodic shape, and infants at four months can match rhythmic structure as well. Infants are especially predisposed to acquire these core aspects of

N

music, and they can also engage in sound play that clearly exhibits creativity.

Y

Individual differences begin to merge in young children as they learn to sing.

U

Some children can match large segments of a song by the age of two or three. Many

Q

others can only approximate pitch at this age and may still have difficulty in producing accurate melodies by the age of five or six. However, by the time they

M

reach school age, most children in any culture have a schema of what a song should

be like and can produce a reasonably accurate imitation of the songs commonly heard in their environment.

ẠY

The early appearance of superior musical ability in some children shows that

musical talent may be a separate and unique form of intelligence. There are

D

numerous tales of young artists who have a remarkable “ear” or extraordinary memory for music and a natural understanding of musical structure. In many of these cases, the child is average in every other way but displays an exceptional ability in music. Even the most gifted child, however, takes about ten years to

10


achieve the levels of performance or composition that would constitute mastery of the musical sphere. Energy generation in music history has its famous prodigies - individuals with exceptional musical powers that emerge at a young age. In the eighteenth century,

FF IC IA L

Wolfgang Amadeus Mozart began composing and performing at the age of six. As a

child, Mozart could play the piano like an adult. He had perfect pitch, and at the age of nine he was also a master of the art of modulation - transitions from one key to

another - which became one of the hallmarks of his style. By the age of eleven, he had composed three symphonies and 30 other major works. Mozart’s welldeveloped talent was preserved into adulthood.

O

Unusual musical ability is a regular characteristic of certain anomalies such as

N

autism. In one case, an autistic girl was able to play “Happy Birthday” in the style of

Ơ

various composers including Mozart, Beethoven, Verdi, and Schubert. For the

H

autistic child, music may be the primary mode of communication, and the child may cling to music because it represents as a haven in the world that is largely confusing

N

and frightening.

U

A. strong interest

Y

1. The word precocity in paragraph 1 is closest in meaning to

Q

C. advanced skill

B. good luck D. personal style.

2. Which sentence below best expresses the essential information in the highlighted

M

sentence in paragraph 1?

A. Children may be born with superior musical ability, but their environment

will determine how this ability is developed.

ẠY

B. Every child is naturally gifted, and it is the responsibility of the public

schools to recognize and develop these talents.

D

C. Children with exceptional musical talent will look for the best way to

express themselves through music - making. D. Some musically talented children live in an environment surrounded by music, while others have little exposure to music.

11


3. The author makes the point that musical elements such as pitch and rhythm ……. A. distinguish music from other art forms B. vary in emphasis in different cultures D. express different human emotions

FF IC IA L

C. make music difficult to learn.

4. The word predisposed in paragraph 3 is closest in meaning to ……. A. inclined

B. gifted

C. pushed

D. amused

5. According to the passage, when does musical talent usually begin to appear? A. When infants start to babble and produce sound patterns B. Between the ages of two and four months

N

D. Between ten years old and adolescence

O

C. When children learn to sing at two or three years old

Ơ

6. Why does the author discuss Mozart in paragraph 6?

H

A. To compare past and present views of musical talent. B. To give an example of a well- known musical prodigy

N

C. To list musical accomplishments of the eighteenth century

Y

D. To describe the development of individual musical skill

Q

A. rhythm

U

7. In music, the change from one key to another is known as …… B. prodigy

C. perfect pitch

D. modulation

8. All of the following are given as example of exceptional musical talent EXCEPT

M

……………..

A. a remarkable “ear” or perfect memory for music B. ability to compose major works at a young age

ẠY

C. appreciation for a wide variety of musical styles D. playing a single song in the style of various composers

D

9. The word haven in paragraph 7 is closest in meaning to …….. A. beautiful art

B. safe place

C. personal goal

D. simple problem

10. Which of the following can be inferred from the passage about exceptional musical ability? 12


A. It occurs more frequently in some cultures than in others. B. It is evidence of a superior lever of intelligence in other areas. C. It has been documented and studied but is little understood. Your answers 1.

2.

3.

4.

6.

7.

8.

9.

FF IC IA L

D. It is the result of natural talent and a supportive environment. 5.

10.

IV. Read the following passage and do the tasks that follow. (15 pts)

O

THE LITTLE ICE AGE A ………

N

This book will provide a detailed examination of the Little Ice Age and other

Ơ

climatic shifts, but, before I embark on that, let me provide a historical context. We

H

tend to think of climate - as opposed to weather- as something unchanging, yet

N

humanity has been at the mercy of climate change for its entire existence, with at least eight glacial episodes in the past 730,000 years. Our ancestors adapted to the

Y

universal but irregular global warming since the end of the last great Ice Age,

U

around 10,000 years ago, with dazzling opportunism. They developed strategies for

Q

surviving harsh drought cycles, decades of heavy rainfall or unaccustomed cold; adopted agriculture and stock-raising, which revolutionised human life; and

M

founded the world's first pre-industrial civilisations in Egypt, Mesopotamia and the

Americas. But the price of sudden climate change, in famine, disease and suffering, was often high.

ẠY

B ………

The Little Ice Age lasted from roughly 1300 until the middle of the nineteenth

D

century. Only two centuries ago, Europe experienced a cycle of bitterly cold winters; mountain glaciers in the Swiss Alps were the lowest in recorded memory, and pack ice surrounded Iceland for much of the year. The climatic events of the Little Ice Age did more than help shape the modern world. They are the deeply important context for the current unprecedented global warming. The Little Ice Age 13


was far from a deep freeze, however; rather an irregular seesaw of rapid climatic shifts, few lasting more than a quarter-century, driven by complex and still little understood interactions between the atmosphere and the ocean. The seesaw brought cycles of intensely cold winters and easterly winds, then switched abruptly to years

FF IC IA L

of heavy spring and early summer rains, mild winters, and frequent Atlantic storms, or to periods of droughts, light northeasterly winds, and summer heat waves. C ………

Reconstructing the climate changes of the past is extremely difficult, because systematic weather observations began only a few centuries ago, in Europe and North America. Records from India and tropical Africa are even more recent. For

O

the time before records began, we have only ‘proxy records' reconstructed largely

N

from tree rings and ice cores, supplemented by a few incomplete written accounts.

Ơ

We now have hundreds of tree-ring records from throughout the northern

H

hemisphere, and many from south of the equator, too, amplified with a growing body of temperature data from ice cores drilled in Antarctica, Greenland, the

N

Peruvian Andes, and other locations. We are close to a knowledge of annual

Q

D ………

U

going back 600 years.

Y

summer and winter temperature variations over much of the northern hemisphere

This book is a narrative history of climatic shifts during the past ten centuries, and

M

some of the ways in which people in Europe adapted to them. Part One describes the

Medieval Warm Period, roughly 900 to 1200. During these three centuries, Norse voyagers from Northern Europe explored northern seas, settled Greenland, and

ẠY

visited North America. It was not a time of uniform warmth, for then, as always since the Great Ice Age, there were constant shifts in rainfall and temperature. Mean

D

European temperatures were about the same as today, perhaps slightly cooler. E ……… It is known that the Little Ice Age cooling began in Greenland and the Arctic in about 1200. As the Arctic ice pack spread southward, Norse voyages to the west were rerouted into the open Atlantic, then ended altogether. Storminess increased in 14


the North Atlantic and North Sea. Colder, much wetter weather descended on Europe between 1315 and 1319, when thousands perished in a continent-wide famine. By 1400, the weather had become decidedly more unpredictable and stormier, with sudden shifts and lower temperatures that culminated in the cold

FF IC IA L

decades of the late sixteenth century. Fish were a vital commodity in growing towns

and cities, where food supplies were a constant concern. Dried cod and herring were already the staples of the European fish trade, but changes in water temperatures

forced fishing fleets to work further offshore. The Basques, Dutch, and English developed the first offshore fishing boats adapted to a colder and stormier Atlantic.

A gradual agricultural revolution in northern Europe stemmed from concerns over

O

food supplies at a time of rising populations. The revolution involved intensive

N

commercial farming and the growing of animal fodder on land not previously used

Ơ

for crops. The increased productivity from farmland made some countries self-

H

sufficient in grain and livestock and offered effective protection against famine. F ………

N

Global temperatures began to rise slowly after 1850, with the beginning of the

Y

Modern Warm Period. There was a vast migration from Europe by land-hungry

U

farmers and others, to which the famine caused by the Irish potato blight

Q

contributed, to North America, Australia, New Zealand, and southern Africa. Millions of hectares of forest and woodland fell before the newcomers’ axes

M

between 1850 and 1890, as intensive European farming methods expanded across

the world. The unprecedented land clearance released vast quantities of carbon dioxide into the atmosphere, triggering for the first time humanly caused global

ẠY

warming. Temperatures climbed more rapidly in the twentieth century as the use of fossil fuels proliferated and greenhouse gas levels continued to soar. The rise has

D

been even steeper since the early 1980s. The Little Ice Age has given way to a new climatic regime, marked by prolonged and steady warming. At the same time, extreme weather events like Category 5 hurricanes are becoming more frequent.

15


The Reading has six paragraphs, A-F. Choose the correct heading for each paragraph from the list of headings below. LIST OF HEADINGS Predicting climatic changes

ii

The relevance of the Little Ice Age today

iii

How cities contribute to climate change

iv

Human impact on the climate

v

How past climatic conditions can be determined

vi

A growing need for weather records

vii

A study covering a thousand years

viii

People have always responded to climate change

……………

3. Paragraph C

…… ……

4. Paragraph D

……………

5. Paragraph E

……………

6. Paragraph F

……………

O

U

Q

Questions 7-10

H

2. Paragraph B

N

……………

Y

1. Paragraph A

Ơ

Enough food at last

N

ix

FF IC IA L

i

M

Complete the summary using the list of words below. Weather during the Little Ice Age

Documentation of past weather conditions is limited: our main sources of knowledge of

ẠY

conditions in the distant past are (7) _______ and (8) _______.We can deduce that the Little Ice Age was a time of (9) _______ rather than of consistent freezing. Within it there

D

were some periods of very cold winters, others of storms and heavy rain, and yet others that saw (10) ________ with no rain at all. climatic shifts ice cores interactions

storms weather observations glaciers

tree rings heat waves written accounts 16


PART D: WRITING (60pts) Part 1: (5 pts) Use the word given in brackets and make any necessary additions to write a new

sentence. Do NOT change the form of the given word.

FF IC IA L

sentence in such a way that it is as similar as possible in meaning to the original 1. I’d be grateful if you could have a look at these figures. (CAST)

_____________________________________________________________ 2. You have said exactly the right thing. (NAIL)

_____________________________________________________________

3. They’ve been having discussions on the issue for over two weeks. (PROGRESS)

O

=> Discussions _________________________for over two weeks.

N

4. It won’t harm us to see what special offers are on at the moment. (AS)

Ơ

=> We might ___________________________on at the moment. 5. Would you mind if I asked you to sign this petition? (RAISE)

N

H

=> Would you _____________________asking you to sign this petition?

Y

Part 2: (20 pts)

U

The graph below shows the differences in wheat exports over three different

Q

areas. Write a report for a university lecturer describing the information shown

D

ẠY

M

below. Write at least 150 words.

17


Part 3: Writing an essay (35 pts) In the process of integration, the need for closing the distance between Vietnam and developed countries has made demands for quality of education more urgent than ever before. However, we are focusing too much on

FF IC IA L

providing human resources to meet with economic goals and underestimating education’s role in personal development. To what extent do you agree or disagree?

Give reasons for your answer and include any relevant examples from your own experience or knowledge. Do not include your personal information. You should

Nguoi gui de: Nguyen Thuy Nhung : 01688121859

D

ẠY

M

Q

U

Y

N

H

Ơ

N

O

write about 250 words.

18


HƯỚNG DẪN CHẤM ĐỀ ĐỀ XUẤT

SỞ GD&ĐT BẮC NINH TRƯỜNG THPT CHUYÊN BẮC NINH

HSG DH&ĐBBB –K11 2016

I. 1. F

2. F

3. T

4. F

5.T

II.

FF IC IA L

PART A. LISTENING (50pts):

6. civilian casualties

2. bloodbath scenario

7. co-sponsor

3. corpses

8. the island's humanitarian

4. heavy arms

9. for a ceasefire

N

O

1. killing of civilians

10. that a ceasefire

Ơ

5. government-run territory III. 18. A

19. C

H

17. B

20. C

N

16. D

Y

PART B. LEXICO-GRAMMAR (30 pts)

U

I. Choose the best answer (A, B, C or D) to complete each of the sentences.(10

Q

pts) 5. A

9. C

13. C

17. A

2. A

6. C

10. A

14. A

18. C

7. C

11. B

15. B

19. B

12. A

16. D

20. B

3. B

M

1. B

4. B

8. B

ẠY

II. The passage below contains 5 mistakes. Underline the mistakes and correct

D

them. (5 pts) Humans have struggled with weeds since the beginnings of agriculture. The global need for weed control has been answered mainly by the chemical industry. Its herbicides are effective and sometimes necessary, but some pose serious problems, particularly if disuse. Toxic compounds threaten animal and public health when they accumulate in food plants, ground water, and drinking water. They also harm workers who apply them.


1. with

against

2. disuse

misused

3. world

world’s

4. of attacking

to attack untouched

N

5.untouch

O

FF IC IA L

In recent years, the chemical industry has introduced several herbicides that are more ecologically sound. Yet new chemicals alone cannot solve the world weed problems. Therefore, scientists are exploring the innate weed-killing powers of living organisms, primarily insects and microorganisms. The biological agents now in use are harmless to humans and are environmentally benign. They can be chosen for their ability of attacking selected targets and leave crops and other plants untouch. In contrast, some of the most effective chemicals kill virually all the plants they come in contact with, sparing only those that are naturally resistant or have been genetically modified for resistance.

Ơ

III. Fill in each of the blanks a suitable preposition (5 pts)

H

1. for

N

2. over 3. through

Y

4. up

U

5. beneath

2. interdependent

M

1. organism

Q

IV. Write the correct form of the word given in brackets. (10 pts) 6. willingness

9. respected

10. inevitably

5. inadvertently

3. intricate

4. inextricably

7. warnings

8. democratic

ẠY

PART C. READING (60 pts)

D

I. Read the text below and decide which answer (A, B, C or D) best fits each space. (15 pts) 1. C

2. B

3. D

4. A

5. A

6. C

7. B

9. D

10. A

11. C

12. D

13. B

14. C

15. C

8. D

II. Fill each of the numbered blanks in the passage with one suitable word. (15 pts)


1. almost

2. materials

3. used

6. Having

7. which

8. output

4. in

5. By

9. affected

10. made

each question. (15pts) 1. C

2. A

3. B

4. A

6. B

7. D

8. C

9. B

FF IC IA L

III. Read the following passage and choose the best answer (A, B, C or D) to 5. C

10. D

3. Paragraph C

……v ……

4. Paragraph D

……vii………

5. Paragraph E

……ix………

6. Paragraph F

……iv………

Questions 7-10 8/7. ice cores

M

Q

9. climatic shifts

U

Y

7/8. tree rings

10. heat waves

N

……ii………

Ơ

2. Paragraph B

H

……viii………

N

1. Paragraph A

O

IV. Read the following passage and do the tasks that follow. (15 pts)

PART D: WRITING (60pts) Part 1: (5 pts)

ẠY

1. I’d be grateful if you could cast an eye /your eyes over these figures.

2. You have hit the nail on the head

D

3. Discussions on the issue have been in progress for over two weeks. 4. We might as well see what special offers are on at the moment. 5. Would you raise any objections to my asking you to sign this petition? Part 2: (20 pts)


Part 3: WRITING AN ESSAY ( 35 pts) Notes: The mark given to part 3 is based on the following criteria:

H

Ơ

N

O

FF IC IA L

1. Content: (35% of total mark) a. Providing all main ideas and details as required b. Communicating intentions sufficiently and effectively 2. Organization & Presentation: (30% of total mark) a. Ideas are well organized and presented with coherence, cohesion, and clarity b. The essay is well-structured 3. Language: (30% of total mark) a. Demonstration of a variety of vocabulary and structures appropriate to the level of English language gifted upper-secondary school students b. Good use and control of grammatical structures 4. Punctuation, and spelling and handwriting (5% of total mark) a. Good punctuation and no spelling mistakes b. Legible handwriting

N

Audio scripts: Part 1:

Y

Welcome to ‘Our Neighbourhood’, the programme that looks at the different places

U

and conditions that people live in. Today we’re visiting the Outback, a distant part

Q

of Australia far from the main population centres. Imagine you’re living on a farm and your nearest neighbours are over 100 kilometres away. It certainly is very

M

lonely and isolated. And this is how it feels for many people living on farms in the

Outback of Australia, far away from the cities and towns. Some people are lucky enough to have email and the Internet but many do not even have a telephone line.

ẠY

Everyone has a two-way radio. You can listen as well as talk into it, and it is the most important means of contact with other people. When someone needs a doctor

D

urgently, they use their two-way radio to contact The Flying Doctors. That’s a special medical service that covers six million square kilometres of Australia. The doctors fly to an emergency in a small aeroplane or helicopter, and they say they can arrive at any place within ninety minutes. There are thirteen flying doctor bases in Australia with thirty-eight aeroplanes and helicopters. They visit 150,000 people


every year. Another problem to solve is education. How do you go to school when you’re living five hundred kilometres from the nearest one? Some kids go to boarding schools, others have their lessons sent by post. But most outback children use their two-way radios twice a day to listen to their teachers and to communicate

FF IC IA L

with them and with other Outback students

(Destination: Australia, Surviving in the Outback, Crown 1, 1999.

© Mary Glasgow Magazines/ Scholastic. Reprinted by permission of Mary Glasgow Magazines.) Now you will hear part 1 again Part 2:

O

The UN's spokesman here, Gordon Weiss, told the BBC that more than 100

N

children died during what he called the large-scale killing of civilians over the

Ơ

weekend. He said the UN had 'warned against the bloodbath scenario and the

H

weekend's events showed that 'that bloodbath has become a reality'. A doctor working in the war zone said on Sunday that the bodies of 378 people had

N

been registered at his hospital and that many other corpses were lying outside. He

Y

said heavy arms appeared to have been fired from government-run territory into a

U

mainly civilian area under Tamil Tiger rebel control. But the government said the

Q

Tigers had done the firing. The issue of civilian casualties is highly sensitive here and the state-owned Daily News on Monday makes no mention of the incident at all.

M

In New York the British Foreign Secretary David Miliband is to co-sponsor

informal discussions with ministers and charities on the island's humanitarian situation. The Sri Lankan government is dismissive of calls from him and other

ẠY

diplomats for a ceasefire in the North. It says it's about to defeat the rebels permanently and that a ceasefire would not help civilians.

D

Now you will hear part 2 again PART 3: You will hear a radio interview with a chef about the process of eating. For question 16-20, choose the answer (A, B, C or D) which fits best according to what you hear.


Interviewer: I’m talking to chef Heston Blumenthal. Now, Heston, most of us think that the business of eating is pretty simple, don’t we? We eat things and we like the taste of them or we don’t, but you reckon it’s more complicated than that, don’t you?

FF IC IA L

Chef: Yes, eating is a process that involves all the senses. Any notion that food is simply about taste is misguided.Try eating a beautifully cooked piece of fish off a paper plate with a plastic knife and fork it is not the same. Interviewer: So how does taste operate then?

Chef: The sense of taste can be broken down into five basic categories, all of which happen in the mouth and nowhere else. These categories are: salt, sweet, sour, bitter

O

and umami- the most recently identified taste. There is a current theory that fat is a

N

taste but this has yet to be proved. We have up to 10,000 taste buds on the tongue

Ơ

and in the mouth. These regenerate, so the receptors we use today will not be the

H

same as were used a couple of days ago. When we eat, taste buds on our tongue pick up taste but not flavour. The molecules in food that provide flavour pass up into the

N

olfactory bulb situated between the eyes at the front of the brain. It contains

Y

hundreds of receptors that register molecules contained in everything that we eat

U

and smell. This is where the flavour of the food is registered.

Q

Interviewer: OK, so our sense of smell is connected with flavour rather than taste? Is that what you’re saying?

M

Chef: That's right. Smell and taste are registered in different parts of the head. There

is a simple but effective and enjoyable way of demonstrating this. Have ready some table salt and biscuits, fruits or anything easy to eat. Squeeze your nostrils tightly

ẠY

enough to prevent breathing through them, but not so tight as to hurt. Take a good bite of biscuit or fruit and start chomping, making sure the nostrils remain clenched.

D

You’ll notice that it is impossible to perceive the flavour or smell of the food being eaten. Now, with nostrils still squeezed and food still in the mouth, lick some salt. Although it was impossible to detect the flavour of the food that was being eaten with clenched nostrils, the taste of the salt is unhindered. Finally, let go of your nostrils and notice the flavour of the food come rushing into your headspace.


Interviewer: I’ll definitely try that some time. So what you’re saying is that all the senses can affect your experience when you eat? Chef: Yes, the brain has to process information given to it by other senses while we are eating, sometimes with surprising results. Here’s another example. A few years

FF IC IA L

ago at a sommelier school in France, trainee wine waiters were put through a routine

wine tasting. Unknown to them, a white wine that they had just tasted had been dyed red with a non-flavoured food dye, then brought back out to taste and evaluate.

Something very interesting happened. They all made notes on the assumption that

the wine was what it looked like - red. In this case, the eyes totally influenced taste perception.

O

Interviewer: OK, so it’s not just about taste, all the senses are coming into play in

N

different ways.

Ơ

Chef: Yes, and as well as allowing us to enjoy food, the senses act as warning

H

systems, taste being the last of the sensory barriers, and bitterness the last of the taste barriers. A natural aversion to bitterness can prevent us from eating foods that

N

could be harmful, although it appears that we have the ability to modify such basic

Y

likes and dislikes. For example, we generally grow to like bitter foods such as tea,

U

coffee and beer as we grow older.

Q

Interviewer: What got you interested in this business of the role played by various senses in the experience of eating? Was it just professional curiosity?

M

Chef: Well, I began thinking about this whole subject a couple of years ago when I

noticed that more and more customers at my restaurant were commenting on the fact that the red cabbage with grain-mustard ice cream served as an appetiser just got

ẠY

better each time they ate it. This was the only dish on the menu whose recipe had not changed over the past year. It seemed that the barrier being presented with this

D

dish was the vivid purple colour of the cabbage, a colour not normally associated with food. To some diners, the initial difficulty of accepting this colour interfered with the appreciation of the dish, but as they got used to it, they lost their inhibition and simply enjoyed its flavour. Interviewer: I see. Now, of course the sense of smell must come...


D

ẠY

M

Q

U

Y

N

H

Ơ

N

O

FF IC IA L

Now you will hear part 3 again.


ĐỀ XUẤT ĐỀ THI HỌC SINH GIỎI KHU VỰC DUYÊN HẢI – ĐBBB 2016 Môn: Tiếng Anh – Lớp 11 ----------------------------

SỞ GIÁO DỤC VÀ ĐÀO TẠO HÀ NỘI TRƯỜNG THPT CHU VĂN AN

(Có Video kèm theo) I. LISTENING Rented properties customer’s requirements

FF IC IA L

Part 1. Complete the form below. Write ONE WORD AND/ OR A NUMBER for each answer. Steven Godfrey

Example:

Answer:

No. of bedrooms:

Four

Preferred location:

In the ……………………..(1) area of town

Maximum monthly rent:

£ ……………………..(2)

Length of let required:

……………………….(3)

Starting:

September 1st

N

O

Name:

Ơ

Complete the table below. Write ONE WORD AND/ OR A NUMBER for each answer. Rooms

Monthly rent

Problem

Oakington Avenue

Living/dining room, separate kitchen

£ 550

No ……………(4)

Mead Street

Large living room and £ 580 kitchen, bathroom and a cloakroom

The ……………(5) is too large

Hamilton Road

Living room, kitchendiner, and a

£ 550

Too ……………(7)

£ ………………(8)

None

U

Y

N

H

Address

Q

…………………(6) Living room, dining room, small kitchen

M

Devon Close

Which two facilities in the district of Devon Close are open to the public at the moment?

…………………(9) hall

and …………………(10) pool.

Part 2. Complete the notes below. Write NO MORE THAN THREE WORDS AND/ OR A NUMBER for each answer.

ẠY

THE NATIONAL ARTS CENTRE Well known for:

D

Complex consists of:

…………………(1) Concert rooms Theatres Cinemas Art galleries Public library Restaurants

1


A …………………(2) Historical background

1940 – area destroyed by bombs 1960s-1970s – Centre was …………………(3) and built In …………………(4) – opened to public the …………………(5)\

Open:

FF IC IA L

Managed by:

363 days per year

Part 3. Choose the correct answer LATIN AMERICAN STUDIES

1. Paul decided to get work experience in South America because he wanted A to teach English there B to improve his Spanish

O

C to learn about Latin American life D to improve his English

N

2. What project work did Paul originally intend to get involved in?

Ơ

A construction B agriculture

H

C tourism

N

D architecture

3. Why did Paul change from one project to another?

Y

A his first job was not well organized

U

B he found doing the routine work very boring C the work was too physically demanding

Q

D his first job was boring

M

4. In the village community, he learnt how important it was to A respect family life

B develop trust

C use money wisely

D spend time with neighbors

ẠY

5. What does Paul say about his project manager? A he let Paul do most of the work

D

B his plans were too ambitious C he was very supportive of Paul D he was too ambitious II. VOCABULARY AND GRAMMAR Part 1. Choose the best answer:

2


1. That knife's no good. It's _____ . A flat B worn

C blunt

D loose

2. Don't eat that sandwich. the bread's _____ . A off

B stale

C rancid

D jammed

C jaded

Drown

3. We need a new carpet for the bedroom. This one's ____ . A mouldy

B dog-eared C smudged

D worn A flat B mouldy

4. You shouldn't eat that cheese. It's ______ .

FF IC IA L

5. I've washed these jeans many times so they're a bit _____ . A faded B scratched C worn D filthy 6. Have you got a new battery for the radio? This one's ____ . A stale B worn C flat D off

7. Careful when you drink from that cup. The rim is _______. A split B dented C scratched D chipped 8.I had a small accident with the car. One of the wings is a little ____ . A stained B dented C jammed D cracked

O

9.Someone spilt red wine on the carpet and now it's _______ . A worn B scratched C cracked D stained 10.I can't open the window. I think it's _____ . A run out B stained C stuck D loose

N

11. She's read this book twice already. That's why it's so ______ . A smudged B dusty C dented D dog-eared

H

Ơ

12. Charlie doesn't have as much hair as he used to. He's got a receding hairline at the front and it's ________ on top. A shrinking B thinning C lightening D weakening

N

13. Many companies use sponsorship of sporting events to gain increased ________ of their name or brand. A sighting B openness C viewing D exposure

A postal adverts

U

Y

14. New restaurants often try to attract customers by using _________ to homes in the surrounding area. B mailshots

C mail flyers D mailboxing

Q

15. When I got back from my holiday, I opened the door to find a huge pile of ____ mail on the doormat. A lot of it was advertising holidays. A ad B bin C junk D promo

B engage

A mark

M

16. It was costing us a thousand pounds a week to _____ a medium-sized ad in a daily paper. So we decided to spend the money on other forms of advertising. C place

D draw up

17. The current BMW _____ line is 'Freude am Fahren', which apparently translates into English as 'The Joy of Driving'. A life B strap C bottom D jingle

ẠY

18. In 2004, it became illegal in the UK to advertise cigarettes in magazines and outdoors on __________. Since then cigarettes have been advertised in other ways. A billboards B wall boards C signboards D sideboards

D

19. Many companies use _______ placement to put items that they sell into famous movies. For example in 'movie land' everybody uses Apple Computers whereas in real life most people use PCs. A merchandising

B hardware

C subliminal D product

20. Our company has spent millions trying to give our brand of toilet cleaners a fashionable image in the hope that younger households will buy ____ that image. A up B into C for D with

3


Part 2 - Read the text carefully and check each line for an error. If you find an error, underline it. Then write the correction in the box at the end of the line. Lines

Correction

1. Though plastic is essential on modern civilization, it is a very new

3. at an exhibition in London less than 150 years away. 4. Four years later celluloid was invented. Today we think of plastic as

FF IC IA L

2. material - the inventor Alexander Parkes demonstrated the first plastic

5. environmentally unfriendly, because it does not decay. But celluloid helped 6. to stop the slaughter of thousands of African elephants whose tusks were

O

7. being used to make ivory billiard balls. Later, thin transparent sheets of

N

8. celluloid have pictures printed onto them, and these reels of celluloid projected

Ơ

9. movies in the cinemas of the 1930s.

H

10. In 1913, a Swiss inventor invented cellophane while searching for a material

N

11. which would resist dirt and water and what could be easily cleaned. He was 12. looking for a superior tablecloth, but instead invented the twentieth century’s

U

Y

13. favourite packaging material. There was no stopping the plastics revolution.

Q

14. Plastic has replaced the fibers in toothbrushes, and the silk in women's stockings. 15. Vinyl is used for flooring, and Teflon for kitchenwear. Polythene is the most

M

16. common plastic - it was used for soft drink bottles, food containers and thousands of other everyday objects. Part 3. Fill in each blank with a correct preposition.

ẠY

1. I'm tired and stressed and the kids have been acting _________ all day. They don't seem to know how to behave properly.

D

2. I met Harold in the supermarket. He was asking _________ you and I said you were fine and really happy about this new job you've got. 3. I'll ask your boss __________ for dinner tonight if he isn't busy. It will make a good impression, don't you think? 4. If your boss accuses you of missing yesterday's meeting on purpose, I will back you _________ by saying you had to do something for me. 5. I can hardly read this document. Go and blow it ________ so it is at least twice the size.

4


Part 4. Use the word given in capitals at the end of some of the lines to form a word that fits in the gap in the same line. Use the word given in capitals at the end of some of the lines to form a word that fits in the space in the same line. Photography has many forms and ……………….(1). Of all of them, the

APPLY

practice of taking pictures of earth from the air has had the most value for WORLD

FF IC IA L

geographers……………….(2).

Photography was still in its……………….(3) when, in 1858, French artist

INFANT

Felix Tournachon took the first aerial photograph from a hot-air balloon. From these crude ……………….(4), aerial photographs have been taken from

BEGIN

…………….(5) higher vantage points and now cameras on satellites can

INCEASE

capture images of earth from space.

O

This would have been…………….(6) in 1858. Today, such images

provide a wealth of detailed information and ……………….…….(7) they have

IMAGINE

CONSEQUENCE

N

made an enormous contribution to our understanding of the Earth’s atmosphere and weather patterns. They are also invaluable to

H

those monitoring urban……………….(9).

Ơ

……………………………(8) and

ARCHAEOLOGY EXPAND

N

Although aerial photographs and maps both present a bird’s eye view of the Earth’s surface, they are valuable in different ways. In terms of ACCURATE

Y

…………………..…………….(10)

III. READING

Q

U

maps will always provide a more reliable representation of the surface of the Earth. However, aerial photography has become the more influential as a means of documenting the world’s changing landscape and ecosystems.

M

Part 1: Chose the words that best complete the sentences in the text

A Great Composer

D

ẠY

The classical composer Ernst Hoffsberger, who passed away earlier this week, truly 1.___ the world of contemporary classical music and was a great 2.___of inspiration to a whole generation of 3.____young artists in various fields. In many ways his three symphonies completely 4.___ the achievements of all other composers of the late twentieth century and by 5.___ the classical genre with jazz, rock and latterly hip-hop, his work at times bore little 6.__ to what is commonly considered to be a classical sound. Born in California just after the Second World War, Hoffsberger had a strict religious 7.___during which he was taught classical piano by his father. He first found work as a(n) 8.___ journalist, playing and composing music in his free time. During the late sixties, he worked together in 9.__ with a number of other amateur musicians before finally 10.___ professional with the first public performance of his inspirational Tenor Sax Concerto in 1971. From then on, throughout the seventies and eighties, each new work seemed to 11.___ the limits of the orchestral medium and also helped to bring classical music to a wider audience. What many people consider Hoffsberger's defining quality that kept his music fresh and original was that he never lost

5


the human 12.___which gave him the ability to sit down and jam with musicians and artists from all walks of life. B restored

C renovated

D refurbished

2. A form

B kind

C source

D outlet

3. A branching

B budding

C blooming

D bursting

4. A overcame

B overshadowed

C overturned

D overwhelmed

5. A adjoining

B attaching

C co-joining

6. A similarity

B familiarity

C resemblance

7. A family

B background

C childhood

8. A non-contract

B off-the-books

C freelance

9. A collaboration

B combination

C coordination

10. A taking

B getting

C making

11. A overpass

B bypass

C surpass

12. A touch

B feeling

C contact

FF IC IA L

1. A revolutionised

D fusing

D identification D upbringing D odd-job

D contribution D turning

O

D encompass D aspect

Ơ

N

Part 2: Complete the following article by writing the missing words in the spaces. Use only one word for each space. Living in Cities.

U

Y

N

H

Urbanization has been a constant 1…………… in human development for the past ten thousand years. 2………..… there has always been the necessity for a minimum - sometimes a very substantial minimum - of the 3………..…….. to work the land, such surplus individuals as can be sustained have tended to gather in cities. Two thousand years ago, the cities of antiquity formed an administrative 4………………… for the surrounding area, and a refuge from attack. In the Middle Ages, it was the city folk 5…………….…… were the first to break the stultifying grip of feudalism, and it was in the cities of Northern Italy that the phenomenon of the Renaissance was 6…………….

M

Q

Today only a fraction of mankind lives in the country, and the 7……..…………. which does so is decreasing. Mega-cities with populations exceeding ten million are common, and they will become even 8……………….. in the coming decades. While many 9……….…………. of city life are unattractive - pollution, stress, and separation from 10…………….……. to name but a few - more and more of humanity seems to be choosing an urban existence. Adjusting to this fact is going to be one of the major challenges of the coming century. And what of the country? The chances are that its relative decline will be reversed as the information revolution continues, it is possible to enjoy the benefits of social contact without needing to leave one's rural residence.

Is it Art?

D

ẠY

Part 3. Choose the answer (A, B, C or D) which you think fits best according to the text. The people may be chosen more than once. Corinne Art is the result of an artist using her or his skill or creative imagination for a creative purpose, to give pleasure to the viewer through it's aesthetic qualities, or to get a reaction from the audience to a wider more significant issue outside the work of art itself. That work of art might be a painting, a sculpture, an installation of some kind or an example from the performing arts like dance or mime. I think we sometimes get bogged down by the notion of 'skill'. For many in the anti modern art camp, there needs to be evidence of the artist's craft on show before the work is taken seriously and can

6


FF IC IA L

merit the term 'art', be it intricate drawing skills, expert use of form or an artist's eye for colour. I'm not suggesting that an artist need not have these credentials but hand in hand with craft is, as I said earlier, creative imagination, the ability to see the value or beauty of something unremarkable which would often go unnoticed by the untrained eye. Much of modern art I think possesses this second quality which is why I often leave an exhibition of modern art feeling that I've had the chance to reflect on something that I wouldn't normally have given the time of day to. The art has engaged me, has had an impact, made me think about something in a way that I wouldn't have thought about before.

Ć

N

O

Michael I would certainly call myself an art enthusiast and have been for many years and in my opinion the modern art world is full of second-rate junk which most of us, if we were being totally honest, would agree a 4-year-old child could do. The idea that a slept-in bed such as that 'produced' by Tracy Emin or many of the pieces by Damien Hirst and his ilk are works of art is hard to justify as is the huge price tag that accompanies their work. I find it particulalry galling when extremely talented people out there who have spent years honing their skills and learning the craft of drawing or painting are completely ignored. What's more, one of the dangers of this kind of 'art' is that it serves to alienate the mass of the population from the visual arts. The man in the street viewing one of these pieces is left thinking the world of modern art has no value; worse still, that he lacks the intellectual ability to understand the meaning of the piece when in fact there is little to interpret. Thankfully, one or two great artists make it through, but I'm afraid many are lost amongst the deluge of dross the art-world deems 'art'. For me, the first measure of the worth of an artist must be the degree of skill exhibited in the work or at the very least a pedigree of fine art preceeding any more abstract pieces produced by the artist such as was the case with Picasso.

KĂˆ

M

Q

U

Y

N

H

Robert The idea that modern art is some kind of mass deception and that all modern artists are talentless fraudsters just doesn't hold water. And I'm not talking here about the painters who for centuries have make a living out of copying works of art and selling them on as originals. I'm talking about abstract art and the idea that the great art collectors such as the Saatchis or Rockerfellas and the great museums of art around the world, would somehow allow themselves to be duped into paying a fortune for an abstract painting or scupture. Are these artists really tricking these people into paying huge sums of money for something worthless? Of course not. Though some of these works may not appear to the layman as having any artisitc merit, neither did the great impressionsists or the more abstract works of Picasso or Rothko when they were first exhibited. In the same way that great poetry can speak to us in a way that prose never can, abstract art can engage with the audience in more subtle and effective ways than is the case with art of a more realistic nature. So, they may get their fingers burnt now and again but I don't think the Saatchis will be cursing the day they spent huge sums on works of abstract art. Quite the opposite in fact and in the process of making a canny investment they have helped further raise the profile of some of our great modern artists.

D

áş Y

Janet Here we go again: the media are once more up in arms about the latest 'is it art' shock-horror editorials following the latest Turner Prize shortlisting. When will they learn? For decades art in many forms has moved away from realism and towards abstraction. Ever since the invention and popularisation of photography, art has had to reinvent itself. Patrons who wanted a perfect representation of themselves no longer needed to turn to the artist. Artists started to struggle with the challenge of catching the essence of the thing depicted rather than simply its external appearance. Abstract artists try to convey a pure idea, not the exact relica of the subject concerned. It's true that some works of art are so obscure that you may need to read up on the theory behind the creation, which is usually helpfully supplied in art galleries. But this isn't always necessary. Take Guernica by Picasso. To get a full understanding of this painting it could be argued the audience needs to appreciate the historical context, the bombing of the Basque city during the Spanish Civil War. It would also probably help to have a good understanding of the techniques of abstraction that Picasso

7


had used to create the effect. However, I think most people viewing this masterpiece would be struck by the horror it depicts even without this background knowledge. And I would argue it is the effect of this abstraction that adds to the impact on us compared to a realistic portrayal of such a scene. Which person gives each of these opinions about modern art? 1 Some practices have been going on for hundreds of years. A) Corinne B) Michael C) Robert D) Janet

FF IC IA L

2 Some people may not have the knowledge to understand a work of art fully. A) Corinne B) Michael C) Robert D) Janet

3 Certain aesthetic qualities can be invisible until brought to our attention by the artist. A) Corinne B) Michael C) Robert D) Janet 4 Picasso is an example of an artist who proved his craftsmanship. A) Corinne B) Michael C) Robert D) Janet 5 Appreciation of the work itself is not always the artist's aim. A) Corinne B) Michael C) Robert D) Janet

O

6 The purpose of Art has undergone change. A) Corinne B) Michael C) Robert D) Janet

N

7 Peopledon't always appreciate the works of great artists initially. A) Corinne B) Michael C) Robert D) Janet

Ơ

8 Abstract at is generally overpriced. A) Corinne B) Michael C) Robert

D) Janet

N

H

9 Abstract art isn't always a good investment. A) Corinne B) Michael C) Robert D) Janet

Y

10 We can be touched by a work of art without knowing the context. A) Corinne B) Michael C) Robert D) Janet

U

Part 4.

Q

Read about energy tracking software and answer the questions. Energy monitoring software

M

A Life is improving for managers at the 2,700 stores of Sainsbury, one of the world's largest supermarket groups. A program from PA, a big software company, will make a boring job much simpler: collecting data about each shop's energy consumption, whether from refrigeration, lights or air conditioning. The automated data collection is part of Sainsbury's plan to reduce by 50% emissions of greenhouse gases from existing shops by 2019.

D

ẠY

B Sainsbury and PA may well be pioneers, but they are not alone. While governments discuss levels of carbon emissions, many companies have already started to make reductions, or are at least preparing to – leading to more and more software firms offering products to help. If predictions are correct the market for carbon-management software could soon become as large as those for other important business applications such as enterprise application software (EAS) programs, a $7 billion market last year. C Many companies have measured energy consumption for some time in an attempt to reduce running costs. Other firms have tracked emissions of different types in order to comply with pollution regulations. In recent times, public pressure has led to more companies publishing emissions data in their annual reports or to organizations like the Carbon Monitoring Project.

8


However, most firms will need to upgrade from the basic tools, such as spreadsheets, they they have been using.

FF IC IA L

D Things are changing, in spite of the recession, says Jim Scarfe, CEO of CarbonReduct, a consultancy. Increased energy costs and new regulations are all pushing companies to monitor their emissions and do so with appropriate software, he states. In the USA, for example, the Carbon Reduction Plan will come into force next year. Among other things, it requires firms that use more than 8,000 megawatt-hours of electricity per annum to evaluate and report the energy they consume. E Expecting an increase in demand, many software-publishers have moved into the market, mostly with internet-based services. In a recent survey SRP Research, another consultancy, listed no fewer than 183 suppliers. Some emphasise reporting, others compliance and still others improving business processes. There are well-established companies, such as EnergySoft and LMG. Many startups, such as CarbonModel and GreenData, have appeared. Even Large software firms like Oracle and IBM have also moved into the market.

O

F For the time being, the needs of most firms are simple: making sure that energy data is collected and can be audited. But in the years ahead, this will change, predicts Susanna Sierra of SRP. Companies will need software that collects energy data automatically, while helping them to find the best ways to reduce emissions and allowing them to manage other resources, such as water.

H

Ơ

N

G Scarfe and Sierra both expect that Oracle and SAP, which already dominate most types of business software, will control the market in this area, too, because it is a good match for their other products. These giants also have the resources to buy the best technology. In June SAP purchased Green Standards, a start-up. Oracle is thought to be planning a similar purchase soon. But they have other rivals. LMG has been buying companies selling environmental software. Some expect great things from X8, a start-up founded by Jana Novic, who pioneered EAS software.

N

H All this interest gives an idea of how important the business of monitoring environmental performance is likely to become. Scarfe recently suggested that in time it could even be as big a market as financial accounting.

Y

List of headings

U

i. Huge industrial growth predicted

Q

ii. A rosy lookout for carbon management systems iii. Higher demands from software to come

M

iv. The main offenders

v. Three reasons why firms must monitor carbon output

vi. Basic software will not be enough vii. A fight for a slice of the market viii. New software for a boring job

ẠY

ix. The major players

D

Match each heading to the most suitable paragraph. The headings may be used more than once. 1. Paragraph A 2. Paragraph B 3. Paragraph C 4. Paragraph D 5. Paragraph E 6. Paragraph F

9


7. Paragraph G 8. Paragraph H Write True, False, or Not given 9. Most companies now report their carbon emissions in their annual statements. _________

FF IC IA L

10. The Carbon Reduction Plan is currently working to reduce carbon emissions. _________ IV. WRITING Part 1.

Complete the second sentence so that it has a similar meaning to the first one using between 3 and 6 words including the word given.

O

1. Would you give us your answer as soon as possible. CONVENIENCE Please respond ......................................... 2. It's one thing to think there's a demand for your product and another to actually make a sale. WORLD

N

There's .......................................................... between thinking there's a demand for your product and actually making a sale. TRACK

Ơ

3. I'm so sorry, I didn't realise it was so late. Sorry, I ............................................................................ time.

4. The anti-smoking advertisement does not stop people from smoking

H

INEFFECTIVE

N

The anti-smoking advertisement is………………………………………..…. people from smoking. 5. There is a possibility that you may be able to get a grant

ELIGIBLE

Y

We are pleased…………………………………………………………. a grant

U

Finish each of the following sentences in such a way that it means exactly the same as the sentences printed before it.

Q

1. I took no notice of the man standing at the gate. I_______________________________________________ the man standing at the gate.

M

2. This hotel is inaccessible in winter.

It ________________________________________________this hotel in winter. 3. The only reason the party was a success was that a famous film star attended.

ẠY

Had it not ___________________________________________ a famous film star, the party would not have been a success. 4. Freak weather conditions resulted in the hurricane which devastated the area.

D

The hurricane which devastated the area _______________________________________ freak weather conditions. 5. He discontinues his studies because he is poor. His________________________________________________ his studies. Part 2. The table below shows the worldwide market share of the mobile phone market for manufacturers in the years 2005 and 2006.

10


Summarize the information by selecting and reporting the main features and make comparison where relevant. Write at least 150 words Worldwide Mobile Phone Sales in 2005 & 2006 (% share of market) 2006 % market share

Nokia

32.5

35

Motorola

17.7

Samsung

12.7

Sony Ericsson

6.3

L.G

6.7

BenQ Mobile

4.9

Others

19.2

Total

100.0

21.1

11.8 7.4 6.3 2.4

16.2

100.0

N

Part 3.

FF IC IA L

2005 % market share

O

Company

H

Ơ

It has been said, "Not all learning takes place in the classroom." Compare and contrast knowledge gained from personal experience with knowledge gained from classroom instruction. In your opinion, which source is more important? Why?

D

ẠY

M

Q

U

Y

N

THE END

11


HƯỚNG DẪN CHẤM ĐỀ THI ĐỀ XUẤT

SỞ GIÁO DỤC VÀ ĐÀO TẠO HÀ NỘI TRƯỜNG THPT CHU VĂN AN

Môn: Tiếng Anh – Lớp 11

----------------------------

ANSWER KEY

FF IC IA L

I. LISTENING Part 1. 1. central 2. 600 3. 2 year(s) 4. garage 5. garden

O

6. study 7. noisy

N

8. 595

Ơ

9. concert 10. swimming

H

Part 2.

N

1. classical music/ (classical/ music) concerts 2. bookshop/ bookstore

Y

3. planned

U

4. 1983/ the 1980s

1. C 2. C

3. A

M

Part 3.

Q

5. City Council

4. B 5. C

ẠY

II. VOCABULARY AND GRAMMAR Part 1. Choose the best answer: 2. B stale

3. D worn

4. B mouldy

5. A faded

7 D chipped

8 B dented

9 D stained

10 C stuck

11 dog-eared 12 B thinning

13 D exposure

14 B mailshots

15. C junk

16C place

18. A billboards

19. D product

20. B into

D

1C blunt 6 C flat

17 B strap

Part 2. Mistake correction

12


Mistake

Correction

1

on

to/for

3. 8.

away have

ago had

15. 16.

kitchenwear was

kitchenware is

Part 3. Fill in each blank with a correct preposition. 1. UP

2. AFTER

3. OVER

4. UP

FF IC IA L

Lines

5. UP

Part 4. Use the word given in capitals at the end of some of the lines to form a word that fits in the gap in the same line. 1 applications

2 worldwide/ world-wide

4 beginnings

3 infancy

5 increasingly 8 archaeologists/ archeologists

9 expansion

10

O

7 consequently accuracy

6 unimaginable

III. READING

N

Part 1: Chose the words that best complete the sentences in the text

Ơ

A Great Composer

1. A revolutionised

H

2. C source

N

3. B budding 4. B overshadowed

Y

5. D fusing

8. C freelance

Q

7. D upbringing

U

6. C resemblance

M

9. A collaboration

10. D turning

11. C surpass 12. A touch

ẠY

Part 2: Complete the following article by writing the missing words in the spaces. Use only one word for each space. Living in Cities. 2. Whilst/Although

3. population

6. born

7.proportion

8.more

D

1. factor/theme

4. center

5. who

9.aspects

10.nature

Part 3. Choose the answer (A, B, C or D) which you think fits best according to the text. 1C

2D

3A

4B

5A

6D

7C

8B

9C

10D

Part 4.

13


1. Paragraph A viii 2. Paragraph B ii 3. Paragraph C vi 4. Paragraph D iii 5. Paragraph E vii

FF IC IA L

6. Paragraph F iii 7. Paragraph G ix 8. Paragraph H i Write True, False, or Not given

9. Most companies now report their carbon emissions in their annual statements. F 10. The Carbon Reduction Plan is currently working to reduce carbon emissions. F

O

IV. WRITING Part 1.

2. a world of difference

H

5. you may be eligible for

4. ineffective at stopping

Ơ

3. lost track of the/ completely lost track of the

N

1. at your earliest convenience

N

Finish each of the following sentences in such a way that it means exactly the same as the sentences printed before it. 1. didn’t pay much attention to

2. isn’t possible to reach/ get to

Y

3. been for the attendance/ presence/ appearance of

U

4. was the result of

Q

Part 2.

5. poverty prevents him from continuing

1. Task achievement

M

2. Coherence and cohesion 3. Lexical resource

4. Grammatical range and accuracy

D

ẠY

Part 3.

1. Task response -

Compare and contrast knowledge gained from personal experience with knowledge gained from classroom instruction

2. Coherence and cohesion -

paragraph format (topic sentence, supporting ideas, examples and conclusion)

-

reasonably similar space given to each advantage / disadvantage

3. Lexical resource -

language of personal opinion (agreement / disagreement)

14


-

vocabulary to do with aspects of gaining knowledge and experience

4. Grammatical range and accuracy -

correct use of grammar and spelling

FF IC IA L

Người ra đề

D

ẠY

M

Q

U

Y

N

H

Ơ

N

O

Phương Nhân (0936868540)

15


KÌ THI HỌC SINH GIỎI NĂM HỌC 2015- 2016

VÙNG DUYÊN HẢI & ĐỒNG BẰNG

MÔN THI: TIẾNG ANH LỚP 11

BẮC BỘ

Ngày thi: 23 tháng 4 năm 2016

TRƯỜNG THPT CHUYÊN

Thời gian làm bài: 180 phút

LÊ QUÝ ĐÔN ĐÀ NẴNG

(không kể thời gian giao đề)

ĐỀ THI ĐỀ XUẤT

Đề thi gồm 17 trang

FF IC IA L

HỘI CÁC TRƯỜNG THPT CHUYÊN

A. LISTENING (50 pts) Part 1 (10 pts)

You will hear part of an interview with Harold Mackenzie, who has written a book

O

about early adolescence.

For questions 1-5, choose the correct answer A, B, C or D.

N

1. According to Harold, what is the main reason pre-teens are receiving more publicity?

Ơ

A. Psychologists now understand the importance of the pre-teen years.

H

B. A great deal of research is being done into the way children develop.

N

C. Pre-teens are now demanding more attention from the media.

Y

D. People now realize pre-teens have economic power.

U

2. Harold suggests that pre-teens

A. cannot keep up with their peers.

Q

B. start to choose their own clothes.

M

C. develop unusual tastes.

D. become more aware of their image. 3. Harold claims friendships are important to pre-teens because A. these relationships help them establish their identities.

ẠY

B. the children are beginning to rebel against their families.

D

C. friends are starting to replace family members. D. the children are now capable of reacting to other people.

4. He suggests that an alternative method of academic evaluation would A. enable parents to be more supportive. B. be more effective than examinations. C. mean less stress for pre-teens. 1


D. delay the onset of tension in adolescence. 5. According to Harold, what is the greatest challenge facing parents of pre-teens? A. Deciding what kinds of toys to buy for their children. B. Developing the correct approach to material possessions.

FF IC IA L

C. Establishing a way of communicating effectively with their children. D. Discovering what kind of help their children really need. Part 2 (10 pts)

Listen and decide whether the following sentences are true (T) or false (F).

1. In a previous lecture, Dr North talked about the humankind’s relationship with sailing.

…………… ……………

3. During the last century, the world population has grown very fast.

……………

4. As well as over-fishing, the fact that oceans are more polluted is a

……………

Ơ

N

O

2. The focus of today’s talk will be on European fishing problems.

H

reason for fishing stock being on the point of collapse in the Pacific. 5. In the UK, fish used to be seen as a luxury.

N

Part 3 (10 pts)

……………

Y

You will hear part of a tutorial between two students and their tutor. The students

U

are doing a research project to do with computer use.

Q

Listen and decide whether the following sentences are true (T) or false (F).

1. Sami and Irene decided to do a survey about access to computer

M

facilities because no one has investigated it before.

……………

2. Sami and Irene had problems with the reading for their project because not much had been written about the topic.

……………

3. Sami and Irene get the main data in their survey from observation of

ẠY

students.

……………

D

4. The tutor suggests that one problem with the survey was limitation in the number of students involved.

……………

5. 77% of students surveyed thought that a booking system would be the best solution.

……………

2


Part 4 (20 pts) You will hear a radio report about interactive science and technology centres in Britain. Complete the sentences, using NO MORE THAN TWO WORDS. - The area on which the National Stone Centre stands has been used for a long time for

FF IC IA L

the mining of (1)………………… and (2)………………… .

- Visitors to the centre are surprised to discover how much stone people (3)…………… . - Examples of the use of stone in construction shown are (4)……………… and (5)……………… .

- The headmaster describes the centre as an excellent (6)…………………….. . - At Techniquest, there are structures which (7)…………………….. .

O

- At Techniquest, a special (8)…………………….. is used for teaching people about

N

centrifugal force.

Ơ

- People can learn about the effect that (9)…………………….. can have on each other at

H

Techniquest.

N

- A dragon is used for teaching people about (10)…………………….. at Techniquest.

Y

B. VOCABULARY AND GRAMMAR (30 pts)

Q

pts)

U

Part 1. Choose the word/ phrase that best completes each of the following sentences. (10

A. surge

M

1. Advances in special effects have brought a new …………… of life to the film industry. B. lease

C. time

D. flash

2. The horror film scared the small child out of her …………… . A. skin

B. nerves

C. brain

D. wits

ẠY

3. The word “friendship” can be …………… applied to a wide variety of relationships. A. lightly

B. slightly

C. loosely

D. sparingly

D

4. In the height of the season, tourists …………… all over the ruins taking photographs. A. swarm

B. flock

C. throng

D. mass

5. I know it’s difficult but you’ll just have to …………… and bear it. A. laugh

B. smile

C. grin

D. chuckle

3


6. Some people regard a sentence of Community Service as a very …………… option. A. light

B. soft

C. simple

D. comfortable

7. The computer has changed the world …………… . A. irreplaceably

B. irrevocably

C. irredeemably

D. irreparably

A. avail

B. result

C. benefit

FF IC IA L

8. He tried to persuade his wife to go to the Antarctic for their holiday but to no ……… . D. good

9. The controversy has been …………… by the arrival of foreign businesses which are seen as a threat to the local economy. A. spurred

B. charged

C. inspired

D. fuelled

never get anywhere. B. hole

C. groove

N

A. trough

O

10. If you don’t make a move soon, you’ll find yourself stuck in a …………… and you’ll D. rut

Ơ

11. She awoke with an …………… sense of foreboding which she could find no reason A. intelligible

B. intensive

H

for.

C. indelible

D. intangible

N

12. His behaviour was always beyond …………… . B. recrimination

Y

A. criticism

C. fault

D. reproach

U

13. When you win the race, you can …………… in the glory of your achievement. B. bask

Q

A. stand

C. bathe

D. lie

A. slide

M

14. Don’t …………… over the main points so quickly - they’re important. B. slip

C. skate

D. ski

15. The two boys really …………… it off from the moment they met. A. hit

B. struck

C. made

D. put

ẠY

16. I for one am not sorry. He should have …………… than to lend them money. A. thought more

B. been better

C. know more

D. known better

D

17. “Last I heard, she was working in South Africa.” “……………?” A. Why did she

B. How come

C. How did she

D. Why come

18. No, thank you. I don’t really like wine, or any kind of alcohol …………… . A. it matters

B. in this matter

C. for the matter

D. for that matter 4


19. I heard he passed his exams, but …………… that I know little else. A. other than

B. more of

C. instead of

D. in addition

C. needing

D. need be

20. “I’ll never finish this by five.” A. is needed

B. need to be

FF IC IA L

“If ……………, I’ll work overtime.”

Part 2. Read the text and find 10 mistakes and correct them. You should indicate in which line the mistake is. (5 pts)

For the past eight years, many of the world’s leading classical musicians have gathered together in Switzerland’s most glitzy ski resort to play, to teach and socialise. If this were

O

all, it would be the ultimate classical music insiders’ club. But the attraction of Verbier,

N

their charm and relevance, is that it is also home for three weeks to more than 100 young

Ơ

musicians from 31 countries, starried-eye about meeting the masters and getting a crashing

H

course at the highest possible level. Conductors of the world’s top orchestras are off hand to get the young musicians into shape, coaxing fine performances of so daunting

N

challenges as Mahler’s Third Symphony and Brahms’ First Symphony.

Y

Verbier is the creation of the Swede, Martin Engstroem, who for many years was a leading

U

agent. He wanted to run his own festival and, having some of the best contacts of the

Q

business, it was not hard to find a Swiss ski report to look for a summer boost, rich villa owners keen to open their houses to musical celebrities and stars used to being indulgent.

M

Engstroem is the most relaxed and charming of men, but in his way he is a dictator. The

music heard at Verbier tends to be of his classical taste with barely a note of the

ẠY

contemporary.

Part 3. Complete each of the following sentences with a suitable preposition or particle.

D

(5 pts)

1. Ali was …………… an inch of reaching his goal when Ben stepped into the picture and

messed up his plans. 2. We haven’t met …………… a month of Sundays and I can hardly remember what he looks like. 5


3. Tim distrusts his new friends and prefers to keep them …………… arm’s length. 4. As the day wore ……………, I began to feel more and more uncomfortable in their company.

FF IC IA L

5. That car must have set Joe …………… quite a bit; it’s top of the range. Part 4. Complete the text by writing the correct form of the word in capitals. The first has been done as an example. (10 pts)

In 1997 I went back to Beijing for the first time since the (0) disastrous (DISASTER)

events of 1989. The Chinese (1)…………………… (AUTHORISE) had been reluctant to re-admit foreign journalists who had witnessed the Tiananmen Square student protests.

O

Even eight years later, it was still (2)…………………… (PROBLEM) to get into the

N

Square with a television camera, but we managed it. I looked for the bullet holes on the

Ơ

steps of the central monument, but they had all been expertly filled in; a faint discoloration

H

perhaps, but almost (3)…………………… (PERCEIVE). The most critical moment in Chinese history after Mao Zedong’s death seemed to have been entirely forgotten.

N

My time in China had given me an (4)…………………… (ENDUREANCE) interest in

Y

Chinese art, so I decided to go to Liu Liu Chang, where for centuries there has been an

U

antiquities market. Unfortunately, many things for sale there nowadays are modern

Q

(5)…………………… (IMITATE). Empty-handed and somewhat (6)…………………… (ILLUSION), I went into a tea house and sat through the usual ceremony, but there were

M

(7)………………… (IDENTIFY) differences here too: it seemed quicker and the tea

lacked that extraordinary lingering scent. Thoroughly (8)…………………… (HEART), I returned to my hotel: one of the enormous, (9)…………………… (FACE) places which

ẠY

have sprung up everywhere. Yet here, in a dark shop tucked away off the lobby, my melancholy mood disappeared, for I met a (10)…………………… (SURVIVE) from

D

1989, who remembered me instantly. Not everything had been entirely forgotten.

6


C. READING COMPREHENSION (60 pts) Part 1. For each gap, choose the correct answer A, B, C or D which best fits the context. (15 pts) The thought of taking a year out from studying is (1)………… . Fun, freedom, adventure,

FF IC IA L

perhaps even money seem to be on offer for a year. So why not go (2)………… it? A year

out can provide all of these things. But before you make a (3)………… decision, take some time to think about whether a year out is really what you want. If it is, then

(4)………… make sure you know what you want to do with it. A year with nothing planned can be depressing and a (5)………… waste of time.

You need to take a positive attitude if you (6)………… to benefit. You need to start

O

preparing for it well before the time comes to take the (7)………… . Most universities and

N

employers look favourably on such (8)………… if they have shown good reasons for

Ơ

taking a year out and have not just (9)………… into it. They (10)………… the maturity,

H

confidence, social and other skills a gap year can bring. Colleges find that gap year students (11)………… to be more mature and motivated. They mix better with their

N

fellow students and work harder because they have a better idea of where they want to go.

Y

They also (12)………… more to college life in general, because they are used to being a

U

part of a team, either at work or study.

Q

Employers also look kindly on (13)………… gap year students. They want employees who are experienced and mature and can deal better with the unexpected. If their year out

M

has also equipped them with skills which other applicants may not have, such as a better

(14)………… of a foreign language, then that too is a plus. Your family may well have (15)………… feelings. On the one hand, they will be proud of

ẠY

your independence; on the other hand, there will be worries about your safety. A. disparaging

B. intimidating

C. alarming

D. enticing

2.

A. at

B. for

C. in

D. off

3.

A. sharp

B. firm

C. deep

D. solid

4.

A. must

B. should

C. can

D. do

5.

A. deep

B. large

C. sheer

D. full

6.

A. can

B. were

C. should

D. are

D

1.

7


A. plunge

B. dive

C. jump

D. spin

8.

A. employees

B. applicants

C. internees

D. aspirants

9.

A. drifted

B. floated

C. flopped

D. flown

10.

A. honor

B. calculate

C. value

D. estimate

11.

A. happen

B. look

C. tend

D. end

12.

A. present

B. participate

C. contribute

D. involve

13.

A. old

B. former

C. later

D. present

14.

A. grasp

B. grab

C. grapple

D. hold

15.

A. different

B. crossed

C. muddled

D. mixed

Part 2. Fill ONE suitable word in each blank. (15 pts)

FF IC IA L

7.

O

Does the thought of making a presentation to a group of peers bring you (1)………………

N

in a cold sweat? If so, you’re not alone. The mere idea of having to “stand and deliver” in

Ơ

front of others is enough to strike dread into the (2)……………… of even the most

H

experienced business person, let alone students. Yet effective spoken communication is an essential (3)……………… for career success in today’s business and academic

N

environments.

Y

So what can people do to add sparkle to their speaking skills and (4)……………… this

U

understandable but unfounded fear of speaking in public? The bad news is that

Q

presentation nerves are quite normal and you will probably always suffer from them. The good news is that interesting speakers are (5)……………… and not born. You can learn

M

the techniques that will (6)……………… you into a calm and convincing speaker.

The first step is to persuade yourself you can do it. Just like an actor waiting in the wings, or an athlete (7)……………… up for the big race, you need to get yourself on a

ẠY

confidence high. Try focusing your thoughts on moments of particular success during your life to (8)……………… . Remember that the physical symptoms of nerves are most

D

obvious to you. The audience won’t see your knees knocking or your hands trembling so don’t worry about it. Some of the worst presentations are those where the speaker clearly hasn’t (9)……………… enough time to it beforehand. Let’s face it; a presentation that’s slung together half an hour before it’s going to be (10)……………… isn’t going to impress anyone. 8


Part 3. Read the passage and choose the best answer to each of the questions. (15 pts) HISTORY OF THE CHICKENPOX VACCINE Chickenpox is a highly contagious infectious disease caused by the Varicella zoster virus; sufferers develop a fleeting itchy rash that can spread throughout the body. The disease

FF IC IA L

can last for up to 14 days and can occur in both children and adults, though the young are particularly vulnerable. Individuals infected with chickenpox can expect to experience a

high but tolerable level of discomfort and a fever as the disease works its way through the system. The ailment was once considered to be a “rite of passage” by parents in the

U.S. and thought to provide children with greater and improved immunity to other forms of sickness later in life. This view, however, was altered after additional research

O

by scientists demonstrated unexpected dangers associated with the virus. Over time, the

N

fruits of this research have transformed attitudes toward the disease and the utility of

Ơ

seeking preemptive measures against it.

H

A vaccine against chickenpox was originally invented by Michiaki Takahashi, a Japanese doctor and research scientist, in the mid-1960s. Dr. Takahashi began his work to isolate

N

and grow the virus in 1965 and in 1972 began clinical trials with a live but weakened form

Y

of the virus that caused the human body to create antibodies. Japan and several other

U

countries began widespread chickenpox vaccination programs in 1974. However, it took

Q

over 20 years for the chickenpox vaccine to be approved by the U.S. Food & Drug Administration (FDA), finally earning the U.S. government’s seal of approval for

M

widespread use in 1995. Yet even though the chickenpox vaccine was available and

recommended by the FDA, parents did not immediately choose to vaccinate their children against this disease. Mothers and fathers typically cited the notion that chickenpox did not

ẠY

constitute a serious enough disease against which a person needed to be vaccinated. Strong belief in that view eroded when scientists discovered the link between Varicella

D

zoster, the virus that causes chickenpox, and shingles, a far more serious, harmful, and longer-lasting disease in older adults that impacts the nervous system. They reached the conclusion that Varicella zoster remains dormant inside the body, making it significantly more likely for someone to develop shingles. As a result, the medical community in the U.S. encouraged the development, adoption, and use of a vaccine against chickenpox to 9


the public. Although the appearance of chickenpox and shingles within one person can be many years apart - generally many decades - the increased risk in developing shingles as a younger adult (30-40 years old rather than 60-70 years old) proved to be enough to convince the medical community that immunization should be preferred to the traditional

FF IC IA L

alternative.

Another reason that the chickenpox vaccine was not immediately accepted and used by parents in the U.S. centered on observations made by scientists that the vaccine simply did not last long enough and did not confer a lifetime of immunity. In other words, scientists

considered the benefits of the vaccine to be temporary when given to young children. They also feared that it increased the odds that a person could become infected with chickenpox

O

later as a young adult, when the rash is more painful and prevalent and can last up to three

N

or four weeks. Hence, allowing young children to develop chickenpox rather than take a

Ơ

vaccine against it was believed to be the “lesser of two evils.” This idea changed over time

H

as booster shots of the vaccine elongated immunity and countered the perceived limits on the strength of the vaccine itself.

N

Today, use of the chickenpox vaccine is common throughout the world. Pediatricians

Y

suggest an initial vaccination shot after a child turns one year old, with booster shots

U

recommended after the child turns eight. The vaccine is estimated to be up to 90%

Q

effective and has reduced worldwide cases of chickenpox infection to 400,000 cases per year from over 4,000,000 cases before vaccination became widespread. ■ (A) In light of

M

such statistics, most doctors insist that the potential risks of developing shingles outweigh

the benefits of avoiding rare complications associated with inoculations. ■ (B) Of course, many parents continue to think of the disease as an innocuous ailment, refusing to take

ẠY

preemptive steps against it. ■ (C) As increasing numbers of students are vaccinated and the virus becomes increasingly rarer, however, even this trend among parents has failed to

D

halt the decline of chickenpox among the most vulnerable populations. ■ (D) 1. The word “tolerable” in the 1st passage is closest in meaning to (A) sudden.

(B) bearable.

(C) infrequent.

(D) unexpected.

2. According to paragraph 1, which of the following is true of the chickenpox virus? (A) It leads to a potentially deadly disease in adults. 10


(B) It is associated with a possibly permanent rash. (C) It is easily transmittable by an infected individual. (D) It has been virtually eradicated in the modern world. 3. Which of the following best expresses the essential information in the highlighted

FF IC IA L

sentence?

(A) U.S. parents believed that having chickenpox benefited their children.

(B) U.S. parents believed that chickenpox led to immunity against most sickness. (C) U.S. parents wanted to make sure that their children developed chickenpox.

(D) U.S. parents did not think that other vaccinations were needed after chickenpox. 4. Which of the following can be inferred from paragraph 2 about the clinical trials for the (A) They took longer than expected.

H

(C) They took a long time to finish.

Ơ

(B) They cost a lot of money to complete.

N

O

chickenpox vaccine?

(D) They were ultimately successful.

N

5. The word “notion” in the 2nd passage is closest in meaning to (A) history.

Y

(B) findings.

(C) fact.

(D) belief.

U

6. According to paragraph 3, which of the following is true of Varicella Zoster?

Q

(A) It typically attacks adults who are over 60 years old. (B) It is linked to a serious disease that occurs more commonly in adults.

M

(C) It likely is not a serious enough threat to human health to require a vaccine.

(D) It is completely eradicated from the body after chickenpox occurs.

7. According to paragraph 3, all of the following is true about the chickenpox virus

ẠY

EXCEPT:

(A) It causes two distinct yet related ailments.

D

(B) People did not view it as a serious public health threat. (C) It tended to quickly become dormant and remain inoperative over time. (D) Vaccination against it would help prevent the onset of shingles.

8. The author uses “booster shots” as an example of (A) a way to increase the effectiveness of the chickenpox vaccine. 11


(B) a preferred method of chickenpox rash and fever treatment. (C) a scientifically approved medicine to eliminate chickenpox. (D) a strategy for parents to avoid vaccinating their child altogether. 9. According to paragraph 4, many parents did not choose the chickenpox vaccine because

FF IC IA L

(A) they believed that the virus was weak and not especially harmful.

(B) they thought that scientists did not have enough data to reach a conclusion.

(C) they were unsure about the utility of the vaccine given its expected duration. (D) they were convinced it was potentially very toxic, particularly for older children.

10. Look at the four squares [■] that indicate where the following sentence could be added

O

to the passage.

N

Meanwhile, some continue to remain unconvinced, citing a supposed potential of the

Ơ

vaccine to do harm. (B) B

(C) C

(D) D

N

(A) A

H

Where would the sentence fit best?

Y

Part 4. Read the following passage and do the tasks that follow. (15 pts)

U

THE ALEXANDER TECHNIQUE AND DISABILITY

Q

A

The Alexander Technique is a method of psychophysical re-education developed by F.

M

Matthias Alexander more than a century ago, initially as a result of trying to solve a vocal

problem. It is a technique for the elimination of ingrained habits of 'misuse' that interfere with the healthy and harmonious functioning of ourselves as a whole, often the underlying

ẠY

cause of many conditions, such as back pain, neck and shoulder tension, fatigue, breathing disorders and other stress-related illnesses.

D

B

Our natural reflex mechanisms for balance and posture are largely dependent on the coordination of the head, neck and back. The Technique addresses the causes of 'misuse' and lack of poise that may be interfering with this relationship. When these mechanisms are allowed to work in harmony, 'good use' spontaneously returns, resulting in easier 12


breathing, freer, lighter movement and a greater ability to control our reactions and our movements. In other words, the Technique enables us to 'use' ourselves better, and, in that sense, is concerned with helping anybody - the so called 'able bodied' as well as disabled people to overcome their disabilities. Hence, the Alexander teacher's approach when

FF IC IA L

working with the disabled is, in essence, the same as with any pupil of the Technique. C

For example, if we take a violinist with a 'misuse' problem of the upper limbs causing technical limitations to his or her playing, the Alexander teacher will work on improving the pupil's overall 'use' by encouraging the inhibition of the habitual muscular tension

pattern that interferes with the co-ordination of the head/neck/back relationship in order to

O

enable him/her to play with more ease. Similarly, when working with a pupil who has lost

N

mobility in the left arm from a stroke, the teacher will first of all address the

Ć

head/neck/back relationship, and the inhibition of extraneous tension that prevents

H

maximum use of the affected limb. In this way, it is possible to enable the stroke patient to retrain mobility of the paralysed part of the body.

N

D

Y

The approach and what results can be expected vary greatly depending on the disability.

U

For the stroke patient, especially if lessons are commenced early after the stroke, the

Q

Alexander Technique can play an important role in rehabilitation and mobility retraining. With a blind person, the work is likely to focus instead more directly on eliminating

M

tension habits that have developed to compensate for the loss of sight, e.g. insecurity E

KĂˆ

leading to stiff and overcautious walking, balancing difficulties and poor head poise.

áş Y

Working with the disabled pupil, the Alexander teacher can offer help with everyday activities, things that the average person takes for granted, such as the ability to brush

D

one's teeth, shave, tie one's shoelaces or cut a slice of bread. By looking at compensatory tension patterns, the teacher can, in many instances, help the disabled person find a new means whereby they can perform these everyday tasks.

13


F In this respect, the lessons may extend to include the disabled person's carer, for example the person who regularly has to help someone in and out of a wheelchair. Using the Alexander Technique, the carer learns not only to lift and give support in the most efficient

FF IC IA L

way to avoid damaging his/her own back, but, as the two learn together, they also become

better skilled at working out strategies enabling the disabled person to become more independent. G

There are, of course, several factors which have to be taken into consideration when

working with disabled pupils. They may suffer intense pain and discomfort, loss of

O

Kinaesthetic awareness (sometimes with total loss of sensitivity in parts of the body),

N

severe lack of co-ordination, loss of mobility, memory loss, blindness, deafness, and

Ơ

speech impairment. The effect this has on the person's emotional and psychological state

H

also has to be taken into account. Some disabled pupils may need longer lessons, because of the time required to move them from the wheelchair, take off casts, slings and other

N

movement aids, etc. Others may only be able to concentrate for short periods of time and,

Y

therefore, require shorter lessons more frequently. It often requires a certain amount of

U

inventiveness on the part of the Alexander teacher, both as far as practical arrangements

Q

and the approach to teaching are concerned, a challenge that, in most cases, is greatly rewarded by the positive results.

M

Questions 1-4

The Reading has seven paragraphs, A-G. Choose the most suitable heading for each paragraph from the List of Headings below. Paragraphs C, E and G have been done for

D

ẠY

you.

LIST OF HEADINGS i

Co-ordination - important for all

ii

Tension and daily routine

iii

Brushing one’s teeth and slicing bread

iv

Fitting the technique to the disability

v

Challenges for the Alexander teacher 14


Musical solutions

vii

Potential drawbacks

viii

Helping the disabled through their helpers Pain problems

x

Better body ‘use’ for all

xi

Retraining limbs

xii

Breaking bad habits

2. Paragraph B:

……………

Paragraph C:

…… xi ……

3. Paragraph D:

……………

Paragraph E:

…… ii ……

4. Paragraph F:

……………

Paragraph G:

…… v ……

H N

Questions 5-10

N

……………

Ơ

1. Paragraph A:

O

ix

FF IC IA L

vi

Y

Complete the summary below using information from the passage. You may use NO

U

MORE THAN TWO WORDS from the passage for each blank. hundred

Q

Alexander Matthias developed the technique named after him more than a years

ago.

With

the

Alexander

Technique,

people

are

M

5. ……………… in a psychophysical way. The Technique works on the body’s

6. ……………… so that they all operate 7. ……………… . As a result, bad

habits are 8. ……………… and the individual is able to live a healthy life.

ẠY

Alexander’s technique can help any of us to use ourselves better. As regards the disabled person, the expected results and exact method used vary, according to

D

the 9. ……………… of the individual, e.g. shorter and more regular sessions in the case of clients who find it difficult to concentrate. With disabled clients, in fact, a number of factors have to be considered, and for the teacher, who often needs to be very inventive, this is 10. ……………… .

15


D. WRITING (60 pts) Part 1. Rewrite the following sentences beginning as shown or using the word given. Do not change the meaning of the original sentences. (5 pts) 1. The two bicycles are virtually the same. CHOOSE

FF IC IA L

There is …………………………………….. the two bicycles.

2. We suspected the weather would get cold so we took warm clothes. ANTICIPATION We took warm clothes .………………………………………………………… cold. 3. He failed the exam because he hadn’t revised or understood the questions. MUCH

He failed the exam as ………………………………………………… misunderstanding the questions.

O

4. My sister makes much of eating healthy food. ATTACHES

N

……………………………………………………………………………………………

Ơ

5. Why on earth did you do something as dangerous as that? POSSESSED

N

Part 2. Graph description (20 pts)

H

……………………………………………………………………………………………

Y

The graph below shows the quantities of goods transported in the UK between 1974

U

and 2002 by four different modes of transport.

Q

Summarize the information by selecting and reporting the main features and make

D

ẠY

M

comparisons where relevant. You should write at least 150 words.

16


Part 3. Essay writing (35 pts) Some people say that many people now have degrees, which makes a degree less important. To what extent do you agree with this idea?

FF IC IA L

Your essay should be about 250 words. The end

ĐT: 0982759772

D

ẠY

M

Q

U

Y

N

H

Ơ

N

O

GV ra đề: Nguyễn Hà Đoan Phương

17


TRƯỜNG THPT CHUYÊN

KÌ THI HỌC SINH GIỎI NĂM HỌC 2015 - 2016

LÊ QUÝ ĐÔN ĐÀ NẴNG

MÔN THI: TIẾNG ANH LỚP 11

ĐỀ THI ĐỀ XUẤT

ĐÁP ÁN

FF IC IA L

A. LISTENING (50 pts) Part 1 (10 pts) 5 câu mỗi câu 2,0 điểm. Tổng cọng 10 điểm. 1. D

2. D

3. A

4. C

5. C

Part 2 (10 pts) 5 câu mỗi câu 2,0 điểm. Tổng cọng 10 điểm. 1. F

2. T

3. T

4. F

5. F

1. F

2. T

3. F

4. F

5. T

O

Part 3 (10 pts) 5 câu mỗi câu 2,0 điểm. Tổng cọng 10 điểm.

Part 4 (20 pts) 10 câu mỗi câu 2,0 điểm. Tổng cọng 20 điểm. 6. teaching resource

3. consume/ eat 7. roll uphill

5. tennis courts

8. dentist’s chair

9. sound waves

H

10. control systems

4. tunnels

N

2. limestone

Ơ

1. lead

N

B. VOCABULARY AND GRAMMAR (30 pts)

Y

Part 1. Choose the word/ phrase that best completes each of the following sentences. (10

U

pts)

20 câu mỗi câu 0,5 điểm. Tổng cọng 10 điểm.

Q

1. B 2 D 3. C 4. A. 5. C 6. B 7. B 8. A 9. D 10. D

M

11. D 12. D 13. B 14. C 15. A 16. D 17. B 18. D 19. A 20. D

Part 2. Read the text and find 10 mistakes and correct them. You should indicate in which line the mistake is. (5 pts) Phát hiện một lỗi 0,25 điểm sửa lỗi đó 0,25 điểm. Tổng cọng 10 lỗi 5 điểm. Mistake

Correction

Line

Mistake

Correction

2

glitzy

glitziest

7

so

such

4

their

its

10

of

in

5

starried-eye

starry-eyed

11

to look

looking

5

crashing

crash

12

indulgent

indulged

6

off

on

14

of

to

D

ẠY

Line

1


Part 3. Complete each of the following sentences with a suitable preposition or particle. (5 pts) 5 câu mỗi câu 1,0 điểm. Tổng cọng 5 điểm. 1. within

2. in

3. at

4. on

5. back

FF IC IA L

Part 4. Complete the text by writing the correct form of the word in capitals. The first has been done as an example. (10 pts) 10 câu mỗi câu 1,0 điểm. Tổng cọng 10 điểm. 1. authorities

2. problematic

3. imperceptible

4. enduring

5. imitators

6. disillusioned

7. identifiable

8. disheartened

9. faceless

10. survivor

O

C. READING COMPREHENSION (60 pts)

N

Part 1. For each gap, choose the correct answer A, B, C or D which best fits the context.

Ơ

(15 pts)

H

15 câu mỗi câu 1,0 điểm. Tổng cọng 15 điểm.

1. D 2. B 3. B 4. D 5. C 6. D 7. A 8. B 9. A 10. C

N

11. C 12. C 13. B 14. A 15. D

Y

Part 2. Fill ONE suitable word in each blank. (15 pts)

U

10 câu mỗi câu 1,5 điểm. Tổng cọng 15 điểm. 2. heart

3. skill

6. turn

7. warming 8. date

Q

1. out

4. overcome

5. made

9. devoted/ dedicated

10. delivered

M

Part 3. Read the passage and choose the best answer to each of the questions. (15 pts)

10 câu mỗi câu 1,5 điểm. Tổng cọng 15 điểm. 1. B 2. C 3. A 4. D 5. D 6. B 7. C 8. A 9. C 10. B

ẠY

Part 4. Read the following passage and do the tasks that follow. (15 pts) 1. xii 2. x

3. iv 4. viii

6. reflex mechanisms/ reflexes

8. eliminated

9. requirements/ disability

D

7. in harmony/ harmoniously

5. re-educated

10. a challenge/ challenging

2


D. WRITING (60 pts) Part 1. Rewrite the following sentences beginning as shown or using the word given. Do not change the meaning of the original sentences. (5 pts) 1. There is very little to choose between the two bicycles.

FF IC IA L

5 câu mỗi câu 1,0 điểm. Tổng cọng 5 điểm. 2. We took warm clothes in anticipation of the weather getting cold.

3. He failed the exam as much out of lack of revision as misunderstanding the questions. 4. My sister attaches much importance to eating healthy food.

5. What on earth possessed you to do something as dangerous as that?

O

Part 2. Graph description (20 pts)

N

1. Content: (6 pts)

Ơ

All the relevant information has been included.

H

2. Organisation: (6 pts)

The answer flows well and is clear and easy to read, that is, the essay is easily

Y

3. Use of language: (8 pts)

N

understood and well-organised.

U

Appropriate linking words and phrases as well as a good level of grammar have been

Q

used. Also, the vocabulary is accurately used.

M

Part 3. Essay writing (35 pts)

1. Content: (12 pts) The question has been fully answered by dealing with all parts of it.

ẠY

2. Organisation: (9 pts) The answer flows well and there is a clear progression of opinions and ideas.

D

3. Use of language: (14 pts) Ideas and opinions are linked by appropriate words and phrases. The use of grammar and vocabulary is accurate and not too simple.

3


HỘI CÁC TRƯỜNG CHUYÊN VÙNG DUYÊN HẢI VÀ ĐỒNG BẰNG BẮC BỘ TRƯỜNG THPT CHUYÊN - ĐHSPHN

ĐỀ THI MÔN: TIẾNG ANH KHỐI 11 - NĂM 2016

Thời gian làm bài: 180’ I. LISTENING Part 1: You will hear a conversation between two senior students who have to organize a competition for the university’s Open Day. First you have some time to look at

FF IC IA L

questions 1 – 5 and choose the answer that best fits according to what you hear. 1. Clare and Grant must arrange a competition which will especially interest A.

school students.

B.

Open Day committee members.

C.

Open Day visitors.

B.

an iPad

C.

an iPhone

N

an iPod

Ơ

A.

O

2. What will the prize be?

a university department

B.

Clare and Grant

C.

Rick Smith

Y

N

A.

H

3. Who will be responsible for buying the prize?

fun

B.

guesswork

C.

ability

M

A.

Q

U

4. What is the most important aspect for entrants in the competition?

5. In the science fiction series on television, what is on the other side of the gateway? a portal

B.

a new world of education

ẠY

A. C.

a different time period

D

Part 2. You will hear part of an interview with Stan Levin, a dance critic, about a modern ballet production involving animals. For questions 6 – 10, decide whether the following statements are true (T) or false (F). You now have some time to look at part 2. 6. The ballet being discussed here is believed to have caused considerable controversy. 7. It appears that the function of the dogs in the ballet is to act as a contrast to the human characters. 1


8. Stan believes that the increasing use of technology in dance is creating a new art form. 9. The aspect of the ballet which is of greatest interest to audiences is the way the dogs copy the actions of one character. 10. The audience’s unwillingness to accept the dogs caused the lapse in mood during the

FF IC IA L

performance Stan saw. Part 3: You will hear an interview with a Marketing Director. First you have some time to look at questions 11 – 20 then fill in the blanks with missing information. Write NO MORE THAN TWO WORDS and/or A NUMBER for each answer. Lecture on (11.) ________________ Example: tourism and (12.) ________________

O

Common misconception is that marketing points to (13.) ________________ in what is being provided.

N

Marketing is actually essential in maintaining (14.) ________________

H

such different (16.) ________________.

Ơ

Selling a product is easier because it is (15.) ________________ and customers don’t have

N

Aim: offer service beyond hopes of (17.) ________________

Y

Important to

U

a. keep informed b. (18.) ________________

Q

One way to achieve this: (19.) ________________

M

(20.) ________________ must always be available for any queries or problems.

II. LEXICO-GRAMMAR Part 1. Choose the word or phrases that best completes each sentence. 1. It won’t ________ matter if you arrive a few minutes late.

ẠY

A. greatly

B. largely

C. grandly

D. considerably

2. All three TV channels provide extensive ________ of sporting events.

D

A. vision

B. coverage

C. broadcast

D. network

3. Time was running out, so the committee had to make a ________ decision. A. brief

B. snap

C. sharp

D. curt

4. Because of an unfortunate ________ your order was not despatched by the date requested. A. hindrance

B. oversight

C. negligence

D. transgression

5. Tempers began to ________ as the lorries forced their way through the picket lines. 2


A. break

B. fray

C. grate

D. fire

6. The government has made no ________ in the flight against inflation; indeed, the situation has worsened recently. A. headway

B. effect

C. avail

D. triumph

7. The completion of the new Town Hall has been ________ owing to a strike. B. held down

C. held up

D. held on

FF IC IA L

A. held off

8. People under 21 years old are not ________ to join this club. A. desirable

B. eligible

C. advisable

D. admissible

9. News of the attempted coup began to ________ through to the outside world. A. pour

B. filter

C. broadcast

D. drip

10. My secretary was ________ to have typed those letters already. B. supposed

C. requested

D. ordered

O

A. asked a computer.

B. graded

C. monitored

Ơ

A. managed

N

11. To prevent flooding in winter the water flowing from the dam is constantly ________ by D. conducted

B. exasperated

C. distracted

D. aroused

N

A. disturbed

H

12. The children’s bad behavior in class ________ their teacher beyond endurance.

Y

13. He bought that house, ________ that he would inherit money under his uncle’s will. C. estimating

U

A. considering B. assuming

D. accounting

14. The postal services were ________ for several weeks by the strike. B. perturbed

Q

A. disrupted

C. disarrayed

D. deranged

B. as long as

A. even if

M

15. You may borrow my bicycle ________ you are careful with it. C. as much as

D. expecting

16. How many people do you think his car would ________? A. occupy

B. hold

C. fit

D. load

ẠY

17. Financial worries gradually ________ his health and he was obliged to retire early. A. undermined B. disabled

C. exhausted

D. invalidated

D

18. If you weren’t satisfied, you ________ have complained to the manager. A. can

B. could

C. need

D. will

19. I don’t feel like buying a ________ in a poke; we’d better check the content. A. pig

B. cattle

C. buffalo

D. ox

20. If you want to have a good flat in London, you have to pay through the ______ for it. A. mouth

B. eye

C. nose

D. head 3


Part 2: There is a mistake in the four underlined parts of each sentence. Find the mistakes (A, B, C or D) and correct them. 1. Hyde Park, the family estate of Franklin D. Roosevelt, sets on top of a bluff A

B

overlooking the Hudson River. D

FF IC IA L

C

2. If it receives enough rain at the proper time, hay will grow quickly as grass. A

B

C

D

3. It is essential that the temperature is not elevated to a point where the substance A

B

formed may become unstable and decompose into its constituent elements. D

O

C

4. Although no country has exactly the same folk music like that of any other, it is B C

N

A

Ơ

significant that similar songs exist among widely separated people.

H

D

Y

N

5. Whitman wrote Leaves of Grass as a tribute to the Civil War soldiers who

U

had laid on the battlefields and whom he had seen while serving as an army nurse. A

B

C

D

Q

Part 3. Fill in each gap with one preposition or particle.

M

Steve had been working for his company for five years when he put in for a promotion and

he was delighted when he was offered a managerial post. But not long after he had taken up the job, the company changed (1)_____ to a new method of accounting. Steve began suffering from two classic symptoms of stress – headaches and insomnia. “I’d thought I

ẠY

would enjoy the increased responsibility, but I felt I was completely (2) snowed ________” he says. “I was toiling (3) __________ until eight o’clock every night just to keep on top of

D

the paperwork.” Steve’s experience is far from unique. Many situations at work can cause stress, ranging from taking on new duties or responsibility to poor working conditions, lack of training, fear of being laid (4) __________ and even harassment and bullying. In the current climate of rapid change, many organizations seek flexibility and so expect workers to be able to turn their hands to a variety of jobs and this too can be very stressful for employees. So what can 4


you do about it if you think you’re suffering from workplace stress? The good news is that employers are legally obliged to ensure employees’ health, safety and welfare at work and this includes bringing (5) __________ measures to reduce stress. Part 4. Read the text below. Use the word given in CAPITALS to form a word that fits in the space. There is an example at the beginning (0). Write your answers on the

FF IC IA L

separate answer sheet. It is (0) …commonly…( COMMON) believed that a break from everyday routine can only do you good. Every summer, you can spot prospective (1)________ (HOLIDAY) at airports

and waiting for car ferries. They are (2) ________(MISTAKE) - you can tell them a mile away by their sun hats and hopeful expressions.

For all their optimism, what often actually happens can be a rude (3) ________(WAKE)

O

from the blissful holiday dreams of the rest of the year. Sunburn, mosquitoes and (4) ________( FORESEE) expenses can make you think twice about

how

(5)

N

________(BENEFIT) getting away from it all really is.

Ơ

The fact is, the (6) ________(LIKELY) of something going wrong is maximised when

H

you are abroad and, (7) ________(FORTUNE), your ability to deal with crisis and

N

catastrophe is often minimised. This could be because of language problems, (8) ________(

U

seem different and unreal.

Y

FAMILIAR) with the culture, or simply a different climate, all of which make everything So, what is the answer? (9) ________(DOUBT), an annual escape from

Q

normal working life is a very positive thing. However, the (10) ________(WISE) of seeking

M

an exotic location is questionable when you think of al the things that can go wrong.

III. READING

Part 1. Read the following passages and decide which answer best fits each gap.

ẠY

History Set in Stone

Many species of animals and plants have disappeared from the earth. They have died out, or

D

become (0) _______. But sometimes animals or plant (1) _______ can be found buried in rocks. These are called fossils. Imprints in rocks (2) (_______ prints, for example) are also called fossils. Not every creature (3) _______ as a fossil. Many simply rot away completely and leave no (4) _______ of their existence. Because many creatures and plants have disappeared without leaving any fossils, we will never know anything about them. 5


The study of fossils, or palaeontology, to give it its scientific (5) _______,

became

established at the beginning of the nineteenth century. Before this research began, people did not believe that fossils had once been (6) _______. Large fossil teeth were seen as evidence of a race of giants in the past, while ammonites, a very (7) _______ type of fossil which you might easily find yourself on a beach or among rocks, were called snakestones because of miracle.

FF IC IA L

their snake-like (8) _______. People believed that snakes had been (9) _______ to stone by a The most famous fossils of all are the dinosaurs. There are, of course, no dinosaurs on (10)

_______ in zoos. They were not (11) _______to extinction by humans as some animals have been, but became extinct millions of years before our own species developed. The reason

why the dinosaurs became extinct is still a mystery. Many theories have (12) _______ the

O

disappearance of dinosaurs with major (13) _______ change.

One possibility is that a gigantic meteorite crashed into the earth (14) _______so much dust

N

into the atmosphere that the (15) _______ of sunlight was reduced. The temperature would

0

A extinct

B. extinguished

D. obsolete

1

A. bones

B. evidence

C. parts

D. remains

2

A. toe

B. paw

C. palm

D. hand

3

A. lasts

B. survives

C. continues

D. develops

4

A. marks

B. proof

C. remnants

D. trace

5

A. name

B. term

C. description

D. status

6

A. alive

B. physical

C. living

D. flesh

A. common

B. usual

C. normal

D. frequent

A. type

B. manner

C. shape

D. figure

9

A. petrified

B. made

C. ossified

D. turned

10

A. exhibition

B. appearance

C. sight

D. display

11

A. hunted

B. brought

C. chased

D. driven

12

A. connected

B. joined

C. explained

D. initiated

13

A. climatic

B. temporal

C. weather

D. seasonal

14

A. disturbing

B. displacing

C. putting

D. pushing

15

A. heat

B. amount

C. degree

D. period

D

Y

Q

M

áş Y

8

KĂˆ

7

N

C. remote

U

extinct.

H

Ć

have fallen and, as a consequence, many types of plants and animals would have become

6


Part 2. Read the text below and think of the word which best fits each gap. Use only one word in each gap. CHEATING AT COMPUTER GAMES It's something that gamer will tell you. Computer games shouldn't be so hard that they drive you mad, but nor should they be so easy that they (1) _________ to offer enough of a challenge. Inevitably,

FF IC IA L

however, you get stuck sometimes. What do you do then? Ask the internet, of course. Many other

gamers have figured (2) _________ what to do and posted the solution online. The answer is just a (3) _________ clicks away.

Purists say this is cheating. They argue that solving a puzzle yourself, (4) _________ gamers had to do in the old days, might have (5) _________ longer, but it was more satisfying. (6) _________ you

O

know that detailed 'walkthroughs’ are available online,

free (7) _________ charge, for almost any game, the temptation is to ask for virtual help (8)

N

_________ the first sign of trouble, (9) _________ robs players of a true sense of achievement.

Ơ

I say this is rubbish. (10) _________ a search and downloading a solution has many merits.

H

It stops me throwing my controller at the screen, and makes me more likely to finish games rather

N

than giving up when they start to get tricky. So all in all, I get better value for money. The search is also a reminder that I'm a member of a broader community, many of whom have been this way

U

Y

before.

Q

Part 3. Read the following passage and choose the answer that best fits The Change in Art after World War II

M

In the 1930s, before the onset of war, rationing, and army drafts, art reflected the

somewhat serene lives of the people. Mundane scenes such as factory workers or office settings were routinely painted to depict the era. They were reminiscent of the people living a

ẠY

routine life in middle-class, ordinary settings. Yet in 1939, fighting spread throughout the world. War and the subsequent struggles for power, existence, and peace brought great

D

unrest for countries around the world following World War II. With the changes wrought by war, many countries felt the need to convey a new, postwar image. It was from this need that abstract expressionism evolved as a modem and recognized art form. Abstract artist Jackson Pollock gave a clear picture of the emergence of abstract art when he said, “The modern painter cannot express this age-the airplane, the atom bomb, the radio—in the old forms of the Renaissance or of any other past culture. Each finds its own 7


technique." It was the art of this revolutionary painter that helped define the abstract movement. Postwar artists like Pollock developed free-form aesthetics by abandoning conventions of past styles while maintaining focused, self-reflexive qualities and the feelings of each individual artist. The method for creating abstract art involved paintings free of religious, political, and popular subjects. The paintings were instead comprised of bright

FF IC IA L

colors and shapes, characterized by personal expression rather than the development of a predictable art style. Much personal empowerment grew out of this profound freedom of expression.

After World War II and during the uncertainty of the Cold War, the world tottered

back and forth between stability and instability. People felt great anxiety amidst their growing prosperity. They viewed the modem art of the time as bold, triumphant, and self-

O

assured. Although the work seemed to exude postwar confidence, artists portrayed profound

unease and viewed their work much differently. Their images were the expression of

N

desperation in the midst of a tough reality inspired by unrest and contrasted with material

Ơ

growth. The psychology of the abstract art form emerged from this altered mindset that was

H

at once strong and vulnerable, confident and subdued. Consequently, artists at the time had

N

the need to feel their experiences in ways that were intense, immediate, direct, subtle, unified, and vivid. "Painting is a state of being … painting is self-discovery. Every good

Y

artist paints what he is," stated Pollock. Abstract expressionism, as the new art style

U

became known, was a way to embody the artist's yearning for stability in an unstable

Q

world as well as a way to emphasize his own personal ideas and use those as expression.

M

Pollock's chief ambition in his art was to incorporate opposition. He did this by pairing

order with chaos, reason with passion, and modernism with primitivism. Similar to other abstract artists, he preferred to portray notions of the subconscious, giving free reign to forgotten personal memories and psychic impulses. George Tooker, another artist of the

ẠY

time, painted The Subway, which illustrated postwar expectations of individuality and

conformity. The affluence of the nation's newfound economic success combined with

D

anxiety over political instability to form a dual consciousness that is said to haunt America's identity still. Each had a set of signature styles that expressed personal and societal isolation of the artist in abstract ways. During this time, modem art became identified widely as "American" art, having its

focal point primarily on the nation. The Museum of Modem Art in New York began to ship abstract expressionistic works to be displayed in places like Milan, Madrid, Berlin, 8


Amsterdam, Paris, and London. ■ A) Some critics overseas were dismayed, stating that this type of abstract art was not new. ■ B) As this art was practiced elsewhere, they continued by saying it was not good quality painting and was not purely American. ■ C) One writer hailed typical American abstract art as “heir of the pioneer and immigrant." ■ D) Another saw the artists as heroic rebels, comparing them to movie stars of the same caliber as James Dean

FF IC IA L

and Marlon Brando or teen idols such as Elvis Presley. As the US was celebrating a highly contradictory mix of freedom and individuality,

abstract expressionism became a political pawn of sorts. The art reflected the ambiguity of

the world at the time as war-ravaged countries worked to recover their economy and people

worked to achieve a normal state of life. The artists of abstract expressionism effectively updated image. A. demonstrate the drastic change in art

Ơ

B. explain the change in America's culture

N

1. The author discusses art from the 1930s in order to

O

captured the emotion of the nation as it emerged from a time of stress and tried to form an

N

D. list the events that transpired

H

C. describe the hardships of the people

C. situations

M

D. traditions

Q

B. agreements

U

A. perceptions

Y

2. The word “conventions” in the passage is closest in meaning to

3. Which of the following can be inferred from paragraph 2 about the change in art? A. Artists sought ways to distinguish their art from previous artists. B. The painters used traditional design elements in whole new ways.

ẠY

C. Consistency in art overpowered the need for originality. D. Artwork reflected the personal empowerment of the artist.

D

4. The word “exude” in the passage is closest in meaning to A. discourage B. portray C. replace D. instruct 9


5. Which of the sentences below best expresses the essential information in the highlighted sentence in the passage? Incorrect choices change the meaning in important ways or leave out essential information. A. Abstract expressionism represented both the artist’s desires for certainty and the artist’s own personal expressions.

FF IC IA L

B. This new art form revealed the artist’s personal style and confidence of expression. C. The Abstract expressionist artist used bold strategies in his art to reveal inner feelings and personal expressions.

D. This new style showed the artist’s determination in life along with his desire for personal freedom.

6. According to paragraph 3, although the work of abstract artists appeared confident, it was

O

in fact B. representative of the country’s wealth

Ơ

C. reflective of the anxiety of the era

N

A. identical to the nation’s certainty

H

D. expressive of the artists’ low self-esteem

N

7. All of the following are mentioned paragraph 4 as Pollock’s techniques painting EXCEPT:

Y

A. Stark displays of contrasts and opposition

U

B. Feelings and impulses from within the mind C. Emotion mixed with ordinary scenes

Q

D. Intense emotions from personal experiences

M

8. The word “each” in the passage refers to

A. artist

B. dual consciousness C. physic impulse

ẠY

D. notion of the unconscious

D

9. According to paragraph 5, abstract expressionism was critiqued for being A. widely followed and admired B. labeled "American" C. exhibited worldwide D. claiming to be modern 10. Look at the four squares [■] that indicate where the following sentence could be added to the passage. 10


Nevertheless, the art persisted, demonstrating the authority of postwar American modernism. Where would the sentence best fit? A. First square B. Second square

FF IC IA L

C. Third square D. Fourth square Part 3: The following passage has six sections, A-F.

Choose the correct headings for sections A-F from the list of headings below. Write the List of Headings No give-aways for addictive products

ii.

Sales of razor blades increase astronomically

iii.

Monopoly of consumables is vital for success

iv.

Video gaming a risky business

v.

A novel method of dual marketing ruled out

vi.

Freebie marketing restricted to legal goods

vii.

Buyer ingenuity may lead to bankruptcy

U

Y

N

H

Ơ

N

i.

O

correct number, i-x, in boxes 1-6 on your answer sheet.

Q

viii. A marketing innovation A product innovation

x.

More money to be made from high quality products

M

ix.

4. Section D______

2. Section B ______

5. Section E ______

3. Section C ______

6. Section F______

ẠY

1. Section A ______

In the late 1890s, while travelling as an itinerant salesperson for the Crown, Cork and

D

A

“FREEBIE" MARKETING

Seal Company, King C. Gillette observed how his corked bottle caps were discarded immediately after opening. Nevertheless, his company turned a healthy profit and there was immense business value, Gillette soon came to realise. In a product that was used only a few times, Gillette had his own personal breakthrough while struggling with a straight-bladed razor—a slow, fiddly and potentially dangerous instrument that required sharpening on a 11


regular basis. A simple, disposable blade that could be thrown away when it dulled would meet a real need and generate strong profits, he correctly reasoned. After founding the American Safety Razor Company in 1901, his sales leapt from 168 blades in 1903 to 123,648 blades only a year later. B

What King C. Gillette pioneered is far more than a convenient and affordable way for

FF IC IA L

men to shave, however, it is the business practice now known as “freebie marketing” that has inspired many more companies over the years. Gillette's approach was contrary to the

received wisdom of his era, which held that a single, durable, high-quality and relatively expensive consumer item with a high profit margin was the best foundation for a business.

Freebie marketing involves two sets of items: a master product that is purchased once, and a consumable product that is frequently disposed of and repurchased on an ongoing basis. In

O

this instance, the master product is often sold with little to no profit margin and is sometimes

even dispensed at a loss. As the consumables are purchased over months and years, however, Freebie marketing only works if the producer of the master item is also able to

Ơ

C

N

this can yield a much greater overall profit.

H

maintain control over the creation and distribution of the consumables. If this does not

N

happen, then cheaper versions of the consumable items may be produced, leaving the

Y

original company without a source of profit. The video game company Atari, for example,

U

initially sold its Atari 2600 consoles at cost price while relying on game sales for profit. Several programmers left Atari, however, and began a new company called Activision which

Q

produced cheaper games of a similar quality. Suddenly, Atari was left with no way to make

M

money. Lawsuits to block Activision failed, and Atari survived only by adding licensing D

measures to its subsequent 5200 and 7800 consoles. In other instances, consumers sometimes find that uses for a master product

circumvent the need to purchase consumables. This phenomenon is well known to have

ẠY

afflicted the producers of CueCat barcode readers. These were given away free through Wired magazine with the intention that they would be used by customers to scan barcodes

D

next to advertisements in the publication and thus generate new revenue flows. Users discovered, however, that the machines could be easily modified and used for other purposes, such as building a personal database of book and CD collections. As no licensing agreement was ever reached between Wired and its magazine subscribers, CueCat were powerless to intervene, and after company liquidation the barcode readers soon became available in quantities over 500,000 for as little as US$0.30 each. 12


E

Not all forms of freebie marketing are legal. One notable example of this is the use of

freebie marketing to “push” habit-forming goods in areas where there is otherwise no market. For illegal substances this is already restricted on the basis of the products illegality, but the use of freebie marketing to promote legal goods such as tobacco, alcohol and pharmaceuticals is also outlawed because the short-term gain to a small number of F

FF IC IA L

commercial outlets is not deemed worth the social cost of widespread substance abuse. Another practice that is prohibited under antitrust laws is a form of freebie marketing

known as “tying”. This is when a seller makes the sale of one good conditional on the acquisition of a second good. In these instances the first good is typically important and highly desirable, while the second is inferior and undesirable. A music distributor who has

the rights to an album that is in high demand, for example, might only allow stores to

O

purchase copies of this album if they also buy unpopular stock that does not sell very easily.

Because this typically relies on the manipulation of a natural monopoly on the part of the

N

distributor, such practices are widely understood to constitute anti-competitive behaviour.

Ơ

Complete the summary below.

H

Choose NO MORE THAN TWO WORDS from the text for each answer.

N

Freebie marketing is not permitted by law for either illegal or legal 7. ________ products.

Y

This type of promotion of goods such as tobacco and alcohol is not considered worth the

U

8.________ and has consequently been outlawed. “Tying" is also prohibited. This is when the sale of an attractive product is 9. ________ on

Q

the purchase of another. It tends to occur when the seller takes advantage of a natural

M

monopoly and is generally considered to be 10. ________

IV. WRITING:

Part 1 : Complete the second sentence so that it has a similar meaning to the first sentence, using the word given. Do not change the word given. You must use between

ẠY

three and eight words, including the word given. Write only the missing words on the separate answer sheet.

D

1. Her lack of qualifications didn’t deter her from applying for the job. (PUT) She _______________________ by her lack of qualifications. 2. He doesn’t sleep very heavily and therefore wakes up rather easily. (LIGHT) Being a ________________________ rather easily. 3. Katy hasn’t been feeling well lately. (RUN) Katy has ________________________ lately. 13


Rewrite the sentences with the beginning in such a way that their meanings remain unchanged. 4. Under no circumstances should you phone the police. The last _______________________________________ 5. You have to be very patient to work as a primary school teacher these days.

FF IC IA L

Working as a primary school teacher calls __________________________ Part 2:

The graph below compare figures for the production and consumption of energy in the US from 1950 to 2000. It also predicts figures for 2025.

Summarise the information by selecting and reporting the main features, and make comparisons where relevant.

áş Y

KĂˆ

M

Q

U

Y

N

H

Ć

N

O

Write at least 150 words.

D

Part 3:

Some people think that a sense of competition in children should be encouraged. Others believe that children who are taught to co-operate rather than compete become more useful adults. Discuss both these views and give your own opinion. ----THE END---14


KEY – DUYÊN HẢI 2015-2016 – GRADE 11 I. LISTENING Part 1: 1. C

2. B

3.A

4.A

5.C

7. F

8.T

9. T

10. F

6. T

FF IC IA L

Part 2: Part 3: 12. legal advice

13. weakness

14. customer numbers

15. tangible

16. expectations

17. clients

18. constantly improve

19. conduct surveys

20. a representative

N

II. LEXICO-GRAMMAR 3.B

6.A

7.C

8.B

11.C

12.B

13.B

16.B

17.A

18.B

M

Part 3 Part 4

15.B

19.A

20.C

3.A not be

4. B as

5.A had lain

2.under

3.away

4.off

5.in

1.over

14.A

N

U

2.D like

5.B 10.B

9.C

Q

1.B sits

4.B

H

2.B

Y

1.A

Ơ

Part 1

Part 2

O

11. services marketing

6. Likelihood

2. Unmistakable/ unmistakeable

7. Unfortunately

3. Awakening

8. Unfamiliarity

4. Unforseen/ unforseeable

9. Undoubtedly/ doubtless

5. Beneficial

10. wisdom

D

ẠY

1. Holidaymakers

III. READING Part 1 1.D

2.B

3.B

4.D

5.A


6.A

7.A

8.C

9.D

10.D

11.A

12.A

13.A

14.B

15.B

Part 2 2. out

3. few

4. as

5. taken

6.when(ever)/ once

7. of

8. at

9. which

Part 3 1.A

2.D

3.A

4.B

6.C

7.C

8.A

9.B

1.ix

2.viii

3.iii

4.vii

6.v

7.habit-

8.social cost

9.conditional

5.A

10.C

N Ơ

10.anticompetitive behaviour

Part 1:

Y

1. was not put off applying for the job

N

H

IV. WRITING:

5.i

O

Part 4

forming

10. doing

FF IC IA L

1. fail/cease

Q

3. been feeling run down

U

2. light sleeper means he wakes up/ light sleeper, he wakes up. 4. thing you should do is (to) phone the police

M

5. for a great/ good deal of patience these days Transcripts Part 1

Hi, Grant. What sort of competition do you think we should organise?

GRANT:

Well Clare, the Open Day Committee was pretty clear on that — it must be

D

ẠY

CLARE:

something with youth appeal.

CLARE:

That makes sense—after all, most of the visitors will have just left high school.

GRANT:

Yeah, so I was thinking—technology.

CLARE:

Do you mean something which uses the latest technology—like an iPod?

GRANT:

Something like that but a bit more expensive maybe.


What about the latest iPhone? I’m saving up for one right now.

GRANT:

Let's make it an iPad—I wish I’d had a tablet computer when I started university.

CLARE:

Yeah...that’s a great idea. That should get a lot of our younger visitors interested.

GRANT:

Right...let's go with that then.

CLARE:

Fine. We could go into town now and buy it I saw great deals advertised at the

Rick

Smith store.

GRANT:

Oh, I don’t think we’ll have to worry about that A university purchase order will

FF IC IA L

CLARE:

probably be arranged through the Resources and Supplies Section. CLARE:

Well, that’s settled then. What about the competition—is it going to be a game of skill or a guessing game or something else? What do you think would work best?

CLARE:

Good question. I don’t think it should be anything too hard or anything that will

O

GRANT:

True. So...something that anyone can do?—Nothing competitive—no skill or

Ơ

GRANT:

N

make the visitors look silly—some of them have such fragile egos. intelligence involved?

That’s right. But the main thing is that the contestants have a lot of fun.

GRANT:

How do we do that?

CLARE:

Well 1 was thinking of a popular TV series: science fiction or science fantasy—I

Y

N

H

CLARE:

U

don’t actually know the difference. Go on.

CLARE:

IPs a series where in every episode the main characters step through a portal into

Q

GRANT:

M

another world or another era. What’s a portal?

CLARE:

It’s like a gateway or entrance to something...

GRANT:

Okay, I get it—they’ll be stepping into the new world of tertiary learning—so

GRANT:

ẠY

somehow we encourage people to step through this ‘portal’—then what? They get their photo taken.

GRANT:

Is that all?

CLARE:

Not exactly—let me think.

GRANT:

I can’t see how that’s a competition...unless we pick the best photograph...but

there's

not much excitement or involvement in that for the participants!

D

CLARE:


CLARE:

Mmm. Wait. We don’t decide on the winner...I mean no one person does...we get

them, the public, to do it. GRANT:

How?

CLARE:

Put all the photos on Facebook—and the one with the most votes wins.

GRANT:

I agree...good idea. But there’s Just one more thing I’m not clear about—how do

CLARE:

FF IC IA L

we get hold of a portal?

I was thinking graduates of the Engineering Department could construct it as part

of their contribution to Open Day. Part 2

You will hear part of an interview with Stan Levin, a dance critic, about a modern ballet

production involving animals. Choose the answer, A, B, C or D which fits best according to

O

what you hear.

N

Interviewer: Stan, you are known as being something of a conservative as far as dance is

Ơ

concerned, so I was intrigued when you told me you wanted to discuss Alain Platel's ballet Wolf on tonight's programme. Wolf generated a furore in certain circles when it was first performed,

H

didn't it?

N

Stan: Yes, it's attracted its fair share of criticism, but it's also been welcomed as one of the most

Y

fascinating modern dance productions in recent years.

U

Interviewer: Some of our viewers may not have seen the ballet and they may be wondering

Q

why all the fuss, so could I ask you to describe briefly what Wolf is about. Stan: Well, basically, it's about homeless people living in a disused shopping mall and

M

returning to some sort of precivilised life. And it features some startling innovations, including

the use of dogs as characters. Interviewer: How do the dogs come into it? Stan: Well, as I understand it, the pack of dogs represents this return to a primitive state. At

ẠY

least, that's the idea Platel is trying to convey. Interviewer: What do you think of the idea of using animals on stage in this way? Can it be

D

justified?

Stan: Well, more and more choreographers these days are moving beyond the traditional limits of dance, and I don't disapprove of this in principle. Many are turning to technology, for instance, using computers to plan the actual choreography. Interviewer: Sometimes even using projections of dancers alongside the real ones…


Stan: Exactly. I find all this very interesting - take the work of Annette Sanderson in New York, for instance - but I think it's now going beyond the genre of dance and turning into something else. Whereas I think Platel is coming from the other direction, if you like, working more with improvisation and basic ensemble techniques. Interviewer: How do audiences respond to Wolf?

FF IC IA L

Stan: By and large, quite enthusiastically. I think some people are surprised at how well it all

works. The dogs generally keep very close to one of the characters. Apparently, the dancer

works intensively with them during rehearsal, and the dogs have earned to imitate his movements. That fascinates audiences. Of course, sometimes the dogs distract attention from an important piece of dancing, but I don't feel this is a real problem.

Interviewer: Do the dogs do anything special during the performance?

O

Stan: No. Their main function is to add atmosphere. It’s not like a circus, with the dogs

N

performing tricks! At the same time, you realise they have been trained and are, in a sense,

Ơ

putting on a show simply by remaining on stage with the human performers. During the performance I saw, a member of the audience in the front row tried to call the dogs over to him,

N

though of course, this wasn't the dogs' fault.

H

which made them look away from the dancers toward the audience. It spoiled the mood -

Y

Interviewer: So the dogs fulfil a kind of symbolic function in the story?

U

Stan: Yes and no. They are attached to one of the characters, a tramp, and we are meant to

Q

understand that they have become a pack. I must say this works rather well: you really do get the impression that the dogs and the tramp have bonded to form a sort of community. But for

M

me, the most striking aspect of the production was the lurking possibility of aggression, largely

Part 3

as a consequence of the presence of the animals.

ẠY

Interviewer: Continuing our theme of business marketing, I have with me today Mr Brian Kinsella, who is here to talk about the differences between marketing a product and marketing a

D

service.

Brian Kinsella: Good morning. Now I understand that many of you here today are interested in a career in services marketing. Well I have been the marketing director for Oceania Travel for nearly 11 years, so I feel that I can present what I consider to be the most important aspects of marketing a service. However, before I begin, I want to clarify what I mean by services


marketing. This not only means aspects like holiday destinations but also professional services such as legal advice. In short, anyone that sells a service. Actually, a lot of the traditional services such as lawyers, accountants, etc. have not felt too comfortable marketing their services. It's almost perceived in industries such as these that the need to market indicates a weakness in the services provided. However, more and more such

FF IC IA L

industries are realising the importance of marketing to sustain their customer numbers, especially when their competitors are marketing themselves.

Now, the main difference between marketing a product and a service is that the customers

cannot understand exactly what the service will be. They can see a product and can comprehend exactly what that product will do for them. A service is more intangible - by that I mean

whatever each customer gains from the service is often very personal. For example, with a

O

travel agency, clients choose to travel abroad for a multitude of motives. Some people travel

N

overseas for the experience and really want to get to know the culture of the local people.

Ć

Others wish to escape from reality, totally relax in sophisticated comfort and be waited on hand and foot.

H

Obviously, our clients will not be judging what we offer by the same standards, and travel

N

agents, like other such service industries, have an extremely difficult job in satisfying a range of

Y

customers from diverse backgrounds with different expectations.

U

Our company has overcome this dilemma in a number of ways. First of all, our travel

Q

consultants are given extensive training in customer service and buyer behaviour. Our aim is not just to be a profit-making organisation, but also to meet and exceed the expectations or

M

dreams of our clients. Our mission statement, in fact, is primarily to offer a service which is

KĂˆ

above and beyond the hopes of our clients. In addition, we regularly visit the tourist destinations we promote and inform all of our staff about any changes in specific areas. Not only is it important to be fully informed about every possible aspect of the service you are

áş Y

marketing, it is also essential to constantly improve the service offered, At Oceania Travel, we regularly conduct surveys with all of the people that visit our resorts of choice. Any negative

D

feedback we try to remedy at once. Our clients are met by a company representative during their stay, and we have a set procedure for dealing with any complaints. Our clients are not expected to have to approach the hotel reception, as we have a 24-hour contact service direct to our representatives, and this representative should always welcome any customer problems or questions. In the event of a complaint, the representative will then try to remedy the complaint


with the hotel. If the problem cannot be rectified by the hotel manager, our representative is authorised to remedy the situation him or herself- For situations beyond the representative's authority, our complaints department is contacted. The complaints department guarantees a solution within the day. If the customer is still not satisfied, they are welcome to approach our head office on their return.

FF IC IA L

So you see that marketing a service is catering more for the clients' Dictations than anything

else and it is that which makes services marketing a very intricate business. Now that's the end of my presentation, b t if there is anything you want to ask, then please feel free to do so.

D

áş Y

KĂˆ

M

Q

U

Y

N

H

Ć

N

O

Thank you.


HỘI CÁC TRƯỜNG CHUYÊN

ĐỀ THI CHỌN HỌC SINH GIỎI LẦN

VÙNG DUYÊN HẢI VÀ ĐỒNG BẰNG BẮC

THỨ IX

BỘ

MÔN: TIẾNG ANH - KHỐI 11 Thời gian: 180 phút

FF IC IA L

ĐỀ ĐỀ XUẤT ĐIỆN BIÊN

A. LISTENING (50 points)

Part 1: You will hear part of a radio programe in which two people, Sally White and Martin Jones, are discussing the popularity of audio books. For questions 1- 5, choose the answer A, B, C or D which fits best according to what you hear. (10 pts)

O

1. Sally feels that the main advantage of audio books is that they

Ơ

B. make more books accessible to children

N

A. encourage children to read more.

D. are read by experienced actors.

H

C. save parents from having to read to children.

N

2. What does Martin say about the woman who came into his shop?

Y

A. She no longer worries about long journeys.

U

B. Her children used to argue about what to listen to

Q

C. she no longer takes her children to France. D. Her children don’t like staying in hotels.

M

3. Martin says that in the USA there is a demand for audio books because people there

A. were the first to obtain audio books B. feel that they do not have time to read books.

ẠY

C. are used to listening to the spoken words on the radio D. have to drive long distances

D

4. Sally says that authors may record their own books on tape if A. their book has just been published. B. they want it read a certain way. C. they have already read extracts from it aloud. D. there are no suitable actors available. Page 1 of 21


5. According to Sally, successful abridgements depend on A. their closeness to the original B. the length of the original C. the style of the author D. the type of story 1.

2.

3.

FF IC IA L

Your answers: 4.

5.

Part 2. For questions 1-5, you will hear a talk about water shortage in deserts. Decide whether the following statements are true (T) or false (F). (10 pts)

O

1. ______ Scientists have already found one answer to the problem of water shortages.

N

2. ______ They got their idea from a small beetle that is an expert at surviving in hot and dry

Ơ

conditions in Afiran desert.

3. ______ Scientist designed the shape and material like those of the beetle’s bumps to collect

H

water from the air.

N

4. ______ The scientists believe this new technology could be helpful in many dry places.

Y

5. ______ One of the scientists, Philseok Kim, said their design could help speed up the

2.

3.

4.

5.

M

1.

Q

Your answers:

U

process of quickly converting steam to liquid water in thermal power plants.

Part 3. You hear a talk about short-sightedness. Listen to the talk and complete the notes below. Write NO MORE THAN THREE WORDS and/or A NUMBER. (20 pts)

ẠY

Researchers predicted in a (1) ___________________ in the journal Opthalmology that by 2050, half of the world's population will be short-sighted. (2) ___________________ for this

D

condition is myopia. It is also known as near-sightedness. It is when we have difficulty (3) ___________________ that are far away from us. The researchers say that around 4.8 billion people will suffer from myopia myopia. This is a (4) ___________________ in cases from 2000 to 2050. Sixty years ago, around 15 per cent of the Chinese population was shortsighted. Now, almost 90 per cent of Chinese teenagers (5) ___________________ have Page 2 of 21


trouble seeing faraway objects. A recent survey found that up to 95 per cent of teenagers in South Korea are near-sighted. The researchers said they could not pinpoint (6) ___________________ this phenomenon and say it could be a combination of factors. They did say that looking at computer screens and mobile phones could be (7) ___________________. Researchers said (8)

FF IC IA L

___________________in myopia may be linked to "lifestyle changes resulting from a combination of decreased time outdoors and increased (9) ___________________". Science journalist Sarah Zhang said, "spending time outdoors, especially in early childhood, reduces the onset of myopia". Professor Kovin Naidoo, (10) ___________________the report, offered some advice. He said: "You could spend a long time reading computers and screens,

O

but also spend hours outdoors." 6.

Ơ

1.

N

Your answers: 2.

7. 8.

H

3.

N

4.

10.

U

Y

5.

9.

Q

B. LEXICO-GRAMMAR (30 points)

M

Part 1: Choose the word or phrase (A, B, C or D) which best completes each sentence. (10 pts)

1. Journalists were ______ around the hotel, waiting to interview the star. A. sauntering

B. milling

C. trudging

D. staggering

ẠY

2. Tim’s extremely punctual. He turns up every day at nine o’clock on the ______ A. dot

B. spur

C. spot

D. day

D

3. That human rights are ______ is unacceptable in a civilized society. A. abrogated

B. impeached

C. infringed

D. quashed

4. A: You should neverhave agreed to help mend her car! B: “__________________” A. Famous last words

B. Well, you live and learn Page 3 of 21


C. It’s a small word

D. You can’t win them all

5. The rents in this area are ______ the highest in the city. A. far from away

B. away by far

C. far and away

D. far or away

A. on behalf of

B. with obedience to

C. in compliance with

D. with reference to

7. We’ll have to take what he says on ______. A. trust

B. faith

C. belief

FF IC IA L

6. All buliding work must be carried out ______ safety regulations.

D. confidence

8. I don’t like that movie because the storyline seemed ______ . B. analogue

C. poised

D. contrived

O

A. compulsory

N

9. I can’t tell you much about the subject, I’m afraid. I only have a very ______ knowledge of A. fundamental

Ơ

it myself. B. redimentary

C. elemental

D. primary

B. resourse

C. exert

N

A. resort

H

10. No matter how angry he was, he would never ______ to violence. D. resolve

B. disconcerting

U

A. disembodied

Y

11. He looks very aggressive and threatening, and so his soft, gentle voice is rather ______ C. dismissive

D. discordant

Q

12. We were ______ by the officers' decision to divert the whole traffic from the main route. A. rambled

B. shuffled

C. stumbled

D. baffled

M

13. I usually buy my clothes ______. It’s cheaper than going to the dressmaker. B. off the peg

A. on the house

C. in public

D. on the shelf

14. My father _______ when he found out that I had damaged his car. B. saw pink elephants

C. made my blood boil

D. hit the roof

ẠY

A. brought the house down

D

15. She was kept awake for most of the night by the ______ of a mosquito in her car. A. groan

B. moan

C. whine

D. screech

16. Their eventual choice of the house was _____ by the time Peter would take to get to the office. A. related

B. consequent

C. determined Page 4 of 21

D. dependent


17. When the funds finally ______ , they had to abandon the scheme. A. faded away

B. clamped down C. petered out

D. fobbed off

18. I was in a quandary ______ what to say. A. as to

B. for

C. owing to

D. of

19. The argiricultural project suffered a major ______ when winter arrived three weeks ealier A. distortion

B. downfall

FF IC IA L

than expected. C. contraction

D. setback

20. The police arrested the wrong man manily because they ______ the names they had been given by the witness. B. merged

C. confused

D. puzzled

1.

2.

3.

4.

5.

6.

7.

11.

12.

13.

14.

15.

16.

N

Your answers

O

A. bewildered

9.

10.

18.

19.

20.

H

Ơ

17.

8.

N

Part 2: Identify 5 errors in the following passage and correct them, (0) has been done as an example. (5 pts)

I can not stress too much the important on watching your opponent, of knowing exactly

2

where he is on the tennis court and what he is doing. It is usually possible to work out

3

the pattern of his game very early in a match. Test at the front of the court. Try hitting

4

one or two balls up high to see how his shots are like. The more quickly you discover his

5

weaknesses, the easier the match should become.

6

Now and again it may be a good idea to give your opponent an opportunity of a mistake.

7

When, early in the match, it seems that he os very accurate player, but not a forceful one,

8

then you should tempt him to play a winning shot. Give him the opening, for there are

9

some players who simply cannot hit winners. They will try to play an attacking game but

ẠY

M

Q

U

Y

1

D

10 they can't quite finish it off. The way to break down their steady game may be putting 11 them into the front of the court. 12 It is obviously wiser to try to be at the beginning of the match whether your opponent is 13 weaker on his left-hand or on his right-hand side, and then play a little more than fifty 14 percent of your shots down that side. Play a normal attacking game, or the game you Page 5 of 21


15 think you will win, but concentrate on the weaker side. A number of players experience 16 more trouble than others in the back corners of the court - always be ready to recognize 17 this weakness. Perhaps an opponent has a favorite backhand shot, but lacks certainty 18 with his forehand shot. Tempt him to play the forehand shot. Line

0.

Mistake

1

on

Correction of

FF IC IA L

Number 1. 2. 3. 4.

N

O

5.

Ơ

Part 3. Complete each of the following sentences with a suitable preposition or particle. (10

H

pts)

1. She says she won’t help us, but we’ll soon get __________ her.

N

2. The man was forced to give himself __________ to the police.

Y

3. What were you driving __________ when you said you might not see Mark for sometime?

U

4. He came __________ several thousand pounds when his grandfather died.

M

Your answers:

Q

5. I wish you wouldn’t fly __________ me like that every time I made a mistake. 2.

3.

4.

5.

1.

Part 4. Complete the following sentences with the words given in the brackets. You have to

ẠY

change the form of the word. (10 pts) King of the Watchmakers

D

For a period of its history, the city of Coventry had a considerable reputation as the main center of clock and watch-making in Britain, and Coventry timepieces made then were (1) (SYNONYM)

__________________

with

both

quality

and

(2)

(RELY)

_________________. Few people in the city today will have heard of Samuel Watson, but he almost (3) (HAND) __________________ paved the way for Coventry’s involvement in the Page 6 of 21


clock and watch business. He was at the (4) (FRONT) __________________ of the watchmaking revolution in the 1680s, and although it is not known how Watson became involved in the trade, he was a trailblazer for others. Watson made his name in 1682 when he sold a clock to King Charles II and was invited to be the King’s (5) (MATHEMATICS) __________________. The following he began work on an astronomical clock for the King,

FF IC IA L

complete with planets and signs of the zodiac, which took seven years to build. It not only told the time of day but also the (6) (POSITION) _________________ changes of the planets. Queen Mary acquired it in 1691 and it is still in the (7) (OWN) __________________ of the Royal Family. He built several other clocks, and by 1690 the clamour for Watson’s clocks was such that he left Conventry and took up (8) (RESIDE)

O

__________________ in London. He became Master of the London Clockmakers’ Company

N

in 1692, which is testament to his (9) (STAND) _________________ in the growing

Ơ

industry. In 1712, Samuel Watson’s name disappears from the records of the London Clockmakers’ Company, and the (10) (LIKELY) ________________ is that he died in that

H

year.

4.

5.

7.

9.

6.

8.

10.

Y

2.

U

3.

Q

1.

N

Your answer

M

C. READING (60 points)

Part 1: For questions 1–15, read the following passage and decide which answer (A, B, C, or D) best fits each gap. Write your answers in corresponding numbered boxes. (15 pts) SECRETARIES

ẠY

What’s in a name? In the case of the secretary, it can be something rather surprising. The dictionary calls a secretary “anyone who handles correspondence keeps records and does

D

clerical work for others”. But while this particular job (1) _____ looks a bit (2) _____, the word’s original meaning is a hundred times more exotic and perhaps more (3) _____. The word itself has been with us since the 14th century and comes from the medieval Latin word secretaries meaning “something hidden”. Secretaries started out as those members of staff

Page 7 of 21


with knowledge hidden from others, the silent ones mysteriously (4) _____

the

secret

machinery of organizations. out of sight, tucked away

A few years ago “something hidden” probably meant (5)

with all the other secretaries and typists. A good secretary was an unremarkable one, efficiently (6)_______ orders, and then returning mouse-like to his or her station behind the typewriter, but, with the (7) _____ of new technology, the job (8)

itself

FF IC IA L

upgraded

and the role has changed to one closer to the original meaning. The skills required are more (9) _____ and more technical. Companies are (10)

that secretarial staff should already

be (11) ______ trained in, or at least familiar with, a (12)

of

word

processing

packages. In addition to this, they need the management skills to take on some administration,

O

some personnel work and some research. The professionals in the (13) _____ business see all

N

these developments as (14) _____ the jobs which secretaries are being asked to do. in office practice. In the past it was usual to

Ơ

It may also encourage a dramatic (15) B. detail

D. characteristic

2

A. elderly

B. unfashionable

C. outdated

D. aged

3

A. characteristic

B. related

C. likely

D. appropriate

4

A. operating

B. pushing

C. vibrating

D. effecting

5

A. kept

B. covered

C. packed

D. held

6

A. satisfying

B. obeying

C. completing

D. minding

7

A. advent

B. approach

C. entrance

D. opening

8

A. truly

B. validly

C. correctly

D. effectively

9

A. thorough

B. demanding

C. severe

D. critical

10

A. insisting

B. ordering

C. claiming

D. pressing

11

A. considerably

B. highly

C. vastly

D. supremely

12

A. group

B. collection

C. cluster

D. range

13

A. appointment

B. hiring

C. recruitment

D. engagement

14

A. improving

B. intensifying

C. advancing

D. heightening

15

A. turn

B. change

C. switch

D. swing

Q

M

ẠY D

N

A. explanation

Y

1

H

C. definition

U

regard the secretary as almost dehumanized, to be seen and not heard.

Your answers Page 8 of 21


1.

2.

3.

4.

5.

6.

7.

8.

9.

10.

11.

12.

13.

14.

15.

Part 2: For questions 1-10, read the text below and think of the word which best fits each

FF IC IA L

space. Use only one word in each space. (15pts)

ACCIDENTAL INVENTORS

A number of products (1) _______ we commonly use today were developed quite by accident. Two of many possible examples of this concept (2) ________ the leotard and the Popsicle, each of which came (3) ________ when an insightful person recognized a

O

potential benefit in a negative situation.

N

The first of these accidental inventions is the leotard, a close-fitting, one- piece garment

Ơ

worn today by dancers, gymnasts, and acrobats, (4) _______ others. In 1828, a circus (5) __________ named Nelson Hower was faced with the prospect of missing his performance

H

because his costume was (6) ________ the cleaners. In stead of canceling his part of the

N

show, he decided to perform in his long underwear. Soon, other circus performers began

Y

performing the (7) _______way. When popular acrobat Jules Leotard adopted the style, it

U

became (8) ________ as the Leotard.

Q

Another product (9) ______ by chance was the Popsicle. In 1905, eleven – year old Frank

M

Epperson stirred up (10) ________ drink of fruit- flavored powder and soda water and then mistakenly left the drink, (11) ________ the spoon in it, out on the back porch overnight.

As the temperature (12) _______ that night, the soda water froze around the spoon, creating a tasty treat. Years (13)_________ remembering how enjoyable the treat had been.

ẠY

Epperson went (14) _____ business (15) ________ Popsicles.

Your answer 2.

3.

4.

5.

6.

7.

8.

9.

10.

11.

12.

13.

14.

15.

D

1.

Part 3: Read the following passage and answer the questions that follow. (15 pts) Page 9 of 21


The Amazonian wilderness harbors the greatest number of species on this planet and is an irreplaceable resource for present and future generations. Amazonia is crucial for maintaining global climate and genetic resources, and its forest and rivers provide vital sources of food, building materials, pharmaceuticals, and water needed by wildlife and humanity. The Los Amigos watershed in the state of Madre de Dios, southeastern Peru, is representative

FF IC IA L

of the pristine lowland moist forest once found throughout most of upper Amazonian South America. Threats to tropical forests occur in the form of fishing, hunting, gold mining, timber extraction, impending road construction, and slash and burn agriculture. The Los Amigos watershed, consisting of 1.6 million hectares (3.95 million acres), still offers the increasingly scarce opportunity to study rain forest as it was before the disruptive encroachment of

O

modern human civilization. Because of its relatively pristine condition and the immediate

N

need to justify it as a conservation zone and as a corridor between Manu National Park and

Ć

the Tambopata-Candamo Reserved Zone, this area deserves intensive, long-term projects aimed at botanical training, ecotourism, biological inventory, and information synthesis.

H

On July 24, 2001, the government of Peru and the Amazon Conservation Association,

N

represented by Enrique Ort?z, signed a contractual agreement creating the first long-term

Y

permanently renewable conservation concession. To our knowledge this is the first such

U

agreement to be implemented in the world. The conservation concession protects 340,000

Q

acres of old growth Amazonian forest in the Los Amigos watershed which is located in southeastern Peru. This watershed protects the eastern flank of Manu National Park and is

M

part of the lowland forest corridor that links it to Bahuaja-Sonene National Park. The Los

KĂˆ

Amigos conservation concession will serve as a mechanism for the development of a regional center of excellence in natural forest management and biodiversity science.

áş Y

Several major projects are being implemented at the Los Amigos Conservation Area. Louise Emmons is initiating studies of mammal diversity and ecology in the Los Amigos area. Other

D

projects involve studies of the diversity of arthropods, amphibians, reptiles, and birds. Robin Foster has conducted botanical studies at Los Amigos, resulting in the labeling of hundreds of plant species along two kilometers of trail in upland and lowland forest. Los Amigos has also been a major field site for Robin's rapid identification laminated photographic field guides to tropical plants. Michael Goulding is leading a fisheries and aquatic ecology program, which Page 10 of 21


aims to document the diversity of fish, their ecologies, and their habitats in the Los Amigos area and the Madre de Dios watershed in general. With support from the Amazon Conservation Association, and in collaboration with US and Peruvian colleagues, the Botany of the Los Amigos project has been initiated. At Los Amigos, we are attempting to develop a system of preservation, sustainability, and scientific

FF IC IA L

research; a marriage between various disciplines, from human ecology to economic botany, product marketing to forest management. The complexity of the ecosystem will best be understood through a multidisciplinary approach, and improved understanding of the complexity will lead to better management. In essence, we must be informed to make wise management decisions about Amazonian forests. These forests hold the greatest number of

O

species on our planet and are an irreplaceable resource for present and future generations. The

N

future of these forests will depend on sustainable management and development of alternative

Ć

practices and products that do not require irreversible destruction. The botanical project will provide a foundation of information that is essential to other

H

programs at Los Amigos. By combining botanical studies with fisheries and mammology, we

N

will better understand plant/animal interactions. By providing names, the botanical program

Y

will facilitate accurate communication about plants and the animals that use them. Included in

U

this scenario are humans, as we will dedicate time to people-plant interactions in order to

Q

learn what plants are used by people in the Los Amigos area, and what plants could potentially be used by people.

M

To be informed, we must develop knowledge. To develop knowledge, we must collect,

KĂˆ

organize, and disseminate information. In this sense, botanical information has conservation value. Before we can use plant-based products from the forest, we must know what species

áş Y

are useful. We must know what their names are in order to be able to communicate accurately about them. We must be able to identify them, to know where they occur in the forest, how

D

many of them exist, how they are pollinated and when they produce fruit (or other useful products). Aside from understanding the species as they occur locally at Los Amigos, we must have information about their overall distribution in tropical America in order to better understand and manage the distribution, variation, and viability of their genetic diversity and

Page 11 of 21


germplasm. This involves a more complete understanding of the species through studies in the field and herbarium. 1. The phrase “genetic resources” refers to A. plant seeds B. different races of people

FF IC IA L

C. diverse species of plants and animals C. cells that can be used in genetic cures for diseases

2. In paragraph 2, the author emphasizes that the current environmental condition of Amazonian South America is A. mostly unscathed

O

B. restorable through his project

Ơ

D. varying from destroyed to virtually pristine

N

C. irredeemable everywhere but in the Los Amigos watershed

3. The word “concession” could be replaced, without changing the meaning, with B. acknowledgement

C. apology

H

A. grant

D. compromise

N

4. The author implies in paragraph three that the agreement between Peru and the Amazone

Y

Conservation Association is history primarily because it

U

A. was the first long-term agreement regarding land in the Amazone Rainforest.

Q

B. represented the first time a South American government had agreed to renew a conservation agreement.

M

C. is essentially a permanent conservation agreement.

D. represents the first time such an agreement has been in the form of a renewable contract. 5. The author’s main purpose in the passage is to

ẠY

A. demonstrate that conservation efforts have been historically successful and so should be be continued.

D

B. garner support for opposition to destructive activities in the Los Amigos watershed. C. position the Los Amigos watershed agreement as a success towards the achievement of the vital goal of conservation the Amazonian rainforests. D. argue that the study pristine rainforests is essential for documenting and studying the myriad new species that the forests contain. Page 12 of 21


6. The author’s tone in the passage can be best described as A. advocacy for his project over the other competing projects B. general praise for conservation projects in Amazonian South America. C. passionate support for his and related projects. D. zealous advocacy for his point of view.

FF IC IA L

7. The work of Louise Emmons, Robin Foster, and Michael Goulding (in the fourth paragraph) are employed in the passage as A. colleagues of the author’s in his botanical project.

B. examples of the kinds of activities the author and his colleagues are trying to halt. C. scientists who are represent new trends of study in Amazonian botany.

O

D. scientist involved in projects related and amenable to the author’s.

N

8. The author’s botanical project involved all of the following EXCEPT

Ơ

A. studying plants in laboratory.

B. studying how plants are used by humans and animals.

H

C. facilitating pharmaceutical use of plants.

N

D. labeling plants in the Los Amigos area.

Y

9. When the author says that the botanical project will “provide names,” he means that the

U

project will

Q

A. help recognize new species.

B. aid in the standardization of names for new species.

M

C. participate in naming the region’s different zones.

D. clarify the conclusion surrounding the names of different organizations working in Amazonia.

ẠY

10. When the author says that “botanical information has conservation value,” he means that A. a robust understanding of conservationism is aided by botanical information.

D

B. conservationists should strive to preserve botanical information. C. specification is important for conservation. D. political discussions about conservation should use botanical nomenslature. Your answer 1.

2.

3.

4. Page 13 of 21

5.


6.

7.

8.

9.

10.

Part 4. Read the following passage and do the tasks that follow. (15 pts) ADAM’S WINE A Water is the giver and, at the same time, the taker of life. It covers most of the surface of

FF IC IA L

the planet we live on and features large in the development of the human race. On present prediction, it is an element that is set to assume even greater significance. B

Throughout history, water has had a huge impact on our lives. Humankind has always

had a rather ambiguous relationship with water, on the one hand receiving enormous benefit from it, not just as a drinking source, but as a provider of food and a means whereby to travel

O

and to trade. But forced to live close to water in order to survive and to develop, the

N

relationship has not always been peaceful or beneficial. In fact, it has been quite the contrary. very destructive and life-threatening side.

Through the ages, great floods alternated with long periods of drought have assaulted

H

C

Ơ

What has essentially been a necessity for survival has turned out in many instances to have a

N

people and their environment, hampering their fragile fight for survival. The dramatic

Y

changes to the environment that are now a feature of our daily news are not exactly new:

U

fields that were once lush and fertile are now barren; lakes and rivers that were once teeming

Q

with life are now long gone; savannah has been turned to desert. What perhaps is new is our D

M

native wonder when faced with the forces of nature Today, we are more aware of climatic changes around the world. Floods in far flung

places are instant news for the whole world. Perhaps these events make us feel better as we face the destruction of our own property by floods and other natural disasters. In 2002, many parts of Europe suffered severe flood damage running into billions of

ẠY

E

euros. Properties across the continent collapsed into the sea as waves pounded the coastline

D

wreaking havoc with sea defenses. But it was not just the seas. Rivers swollen by heavy rains and by the effects of deforestation carried large volumes of water that wrecked many communities. F Building stronger and more sophisticated river defenses against flooding is the expensive short-term answer. There are simpler ways. Planting trees in highland areas, not just in Page 14 of 21


Europe but in places like Himalayas, to protect people living in low-lying regions like the Ganges Delta, is a cheaper and more attractive solution. Progress is already being made in convincing countries that the emission of carbon dioxide and other greenhouse gases is causing considerable damage to the environment. But more effort is needed in this directing G And the future? If we are to believe the forecasts, it is predicted that two-thirds of the

FF IC IA L

world population will be without fresh water by 2050. But for a growing number of regions of the world the future is already with us. While some areas are devastated by flooding, scarcity of water in many other places is causing conflict. The state of Texas in the United States of America is suffering a shortage of water with the Rio Grande failing to reach the Gulf of Mexico for the first time in 50 years in the spring of 2002, pitting region against

O

region as they view for water sources. With many parts of the globe running dry through

N

drought and increased water consumption, there is now talk of water being the new oil. H

Ơ

Other doom-laden estimates suggest that, while tropical areas will become drier and

uninhabitable, coastal regions and some low-lying islands will in all probability be

H

submerged by the sea as the polar ice caps melt. Popular exotic destinations now visited by

N

countless tourists will become no-go areas. Today’s holiday hotspots of southern Europe and

Y

elsewhere will literally become hotspots- too hot to live in or visit. With the current antic Some might say that this despondency is ill-founded but we have had ample proof that

Q

I

U

behaviour of the weather, it is difficult not to subscribe to such despair. there is something not quite right with the climate. Many parts of the world have experienced

M

devastating flooding. As the seasons revolve, the focus of the destruction moves from one

continent to another. The impact on the environment is alarming and the cost to life depressing. It is a picture to which we will need to become accustomed.

ẠY

Questions 1-8 Choose the most suitable heading for paragraphs A-I from the list of heading below. One

D

of the headings has been done for you as an example. There are more headings than paragraphs, so you will not use all of them. List of headings a. Environmental change has always been with us b. The scarcity of water Page 15 of 21


c. Rivers and seas cause damage d. Should we be despondent? Or realistic? e. Disasters caused by the climate make us feel better f. Water, the provider of food g. What is water?

FF IC IA L

h. How to solve flooding i. Far-flung flooding j. Human’s relationship with water k. The destructive force of water in former times l. Flooding in the future

O

m. A pessimistic view of the future

N

E.g: Paragraph A: ..g…

5. Paragraph F: ………..

Ơ

1. Paragraph B: ………. 2. Paragraph C: ……….

H

3. Paragraph D: ……….

Your answers: 2.

5.

6.

7. Paragraph H: ………. 8. Paragraph I: ………..

3.

4.

7.

8.

M

Q

U

1.

Y

N

4. Paragraph E: ……….

6. Paragraph G: ……….

Question 9-15: Choose the correct option for each of the following.

9. The writer believes that water ____________ . A. is gradually becoming of greater importance.

ẠY

B. will have little impact on our lives in future. C. is something we will need more than anything else.

D

D. will have even greater importance in our lives in the future.

10. Humankind’s relationship with water has been ___________ . A. two-sided.

B. one-sided.

C. purely one of greater benefits.

D. fairly frightening

11. The writer suggests that ___________ . Page 16 of 21


A. we are in awe of the news we read and see on TV every day B. change to the environment leaves us speechless C. we should not be in awe of the news we read and see on TV every day D. our surprise at the environment change brought about by nature is something new A. has to be coordinated internationally B. is more expensive than building sea and river defenses

FF IC IA L

12. According to the text, planting trees ______________ .

C. is a less expensive answer to flooding than building river defenses D. is not an answer to the problem of flooding in all regions 13. By 2050, it is projected that _____________ .

O

A. at least haft the world population will have fresh water

N

B. the majority of the world population will have fresh water

Ơ

C. one-third of the world population will have fresh water

D. fresh water will only be available to haft of the world population

H

14. According to the text, in the future low-lying islands ___________ . B. will not be under water

Y

C. are likely to be under water

N

A. will still be habitable

D. will probably not be under water

U

15. According to the writer _____________ .

Q

A. people do not need to get used to environment damage B. people will need to get used to climate changes that cause environmental damage

M

C. people are now more used to environmental damage than they have been in the past

D. the general despondency about environmental changes is ill-founded Your answer

ẠY

9.

11.

14.

15.

12.

D

13.

10.

D.WRITING (60 pts) Part 1: (5 pts) A/ Finish each of the following sentences in such a way that it is as similar as possible in meaning to the sentence printed before it. (2 pts.) Page 17 of 21


1. Her primus stove ceased to function on the last day. (GASP) Her primus stove ………………………………………. on the last day. 2. At this time of year, the area is often affected by violent storms. -> At this time of year, the area often ………………………………… violent storms B/ For each of the sentence below, write a new sentence as similar as possible in meaning

way. (3 pts.) 7. Trade has deteriorated and staff are being laid off. (BAD)

FF IC IA L

to the original sentence but using the word given. This word must not be altered in any

-> ……………………………………………………………………………………………… 8. Don’t tell the boss about our scheme. (BREATHE)

O

-> ………………………………………………………………………………………………

N

9. Since the advertisement, we have had more applications than we can deal with.

Ơ

(SWAMPED)

-> ………………………………………………………………………………………………

H

Part 2: (20 pts)

N

The graphs below give information about computer ownership as a percentage of the

Y

population between 2002 and 2010, and by level of education for the years 2002 and 2010.

U

Summarise the information by selecting and reporting the main features, and make

D

ẠY

M

Q

comparisons where relevant. Write at least 150 words.

Page 18 of 21


FF IC IA L O

N

…………………………………………………………………………………………………

Ơ

…………………………………………………………………………………………………

H

…………………………………………………………………………………………………

N

………………………………………………………………………………………………… …………………………………………………………………………………………………

Y

…………………………………………………………………………………………………

U

…………………………………………………………………………………………………

Q

…………………………………………………………………………………………………

M

…………………………………………………………………………………………………

………………………………………………………………………………………………… ………………………………………………………………………………………………… …………………………………………………………………………………………………

ẠY

………………………………………………………………………………………………… …………………………………………………………………………………………………

D

………………………………………………………………………………………………… ………………………………………………………………………………………………… ………………………………………………………………………………………………… ………………………………………………………………………………………………… ………………………………………………………………………………………………… Page 19 of 21


………………………………………………………………………………………………… ………………………………………………………………………………………………… ………………………………………………………………………………………………… Part 3. Essay writing (35 pts) In recent years, the family structure and the role of its members are gradually changin. Write an essay (about 250words) to express your opinion.

FF IC IA L

What kinds of changes can ocurr? Do you think these changes are possitive or negative?

………………………………………………………………………………………………… ………………………………………………………………………………………………… …………………………………………………………………………………………………

O

…………………………………………………………………………………………………

N

…………………………………………………………………………………………………

Ơ

………………………………………………………………………………………………… …………………………………………………………………………………………………

H

…………………………………………………………………………………………………

N

…………………………………………………………………………………………………

Y

…………………………………………………………………………………………………

U

…………………………………………………………………………………………………

Q

………………………………………………………………………………………………… …………………………………………………………………………………………………

M

…………………………………………………………………………………………………

………………………………………………………………………………………………… …………………………………………………………………………………………………

ẠY

………………………………………………………………………………………………… …………………………………………………………………………………………………

D

………………………………………………………………………………………………… ………………………………………………………………………………………………… ………………………………………………………………………………………………… ………………………………………………………………………………………………… ………………………………………………………………………………………………… Page 20 of 21


………………………………………………………………………………………………… ………………………………………………………………………………………………… ………………………………………………………………………………………………… ………………………………………………………………………………………………… …………………………………………………………………………………………………

FF IC IA L

………………………………………………………………………………………………… ………………………………………………………………………………………………… ………………………………………………………………………………………………… ………………………………………………………………………………………………… …………………………………………………………………………………………………

O

…………………………………………………………………………………………………

N

…………………………………………………………………………………………………

Ơ

………………………………………………………………………………………………… …………………………………………………………………………………………………

H

…………………………………………………………………………………………………

N

…………………………………………………………………………………………………

Y

…………………………………………………………………………………………………

Q

U

…………………………………………………………………………………………………

M

......................THE END...................

D

ẠY

Người ra đề: Nguyễn Hạnh Tuyết (mobile: 0974866717)

Page 21 of 21


HỘI CÁC TRƯỜNG CHUYÊN VÙNG DUYÊN HẢI VÀ ĐỒNG BẰNG BẮC BỘ ĐỀ ĐỀ NGHỊ ĐIỆN BIÊN

ĐỀ THI CHỌN HỌC SINH GIỎI LẦN THỨ IX MÔN: TIẾNG ANH - KHỐI 11 Thời gian: 180 phút

KEY

FF IC IA L

A. LISTENING (50 points) Part 1: You will hear part of a radio programe in which two people, Sally White and Martin Jones, are discussing the popularity of audio books. For questions 1- 5, choose the answer A, B, C or D which fits best according to what you hear. (10 pts) Your answers: 2. A

3. D

4. B

5. C

O

1. B

N

H

Ơ

N

Part 2. For questions 1-5, you will hear a talk about water shortage in deserts. Decide whether the following statements are true (T) or false (F). (10 pts) Your answers: 1. F 2. T 3. F 4. F 5. T

M

Q

U

Y

Part 3. You hear a talk about short-sightedness. Listen to the talk and complete the notes below. Write NO MORE THAN THREE WORDS and/or A NUMBER. (20 pts) 1. report published 6. definite reasons 2. The medical term 7. a leading cause 3. focusing on objects 8. the sudden rise 4. seven-fold increase 9. near-work activities 5. and young adults 10. a co-author of B. LEXICO-GRAMMAR (30 points)

ẠY

Part 1: Choose the word or phrase (A, B, C or D) which best completes each sentence. (10 pts) 2. A

3. C

4. B

5. C

6. C

7. A

8. D

9. B

10. A

11. B

12. D

13. B

14. D

15. C

16. C

17. B

18. C

19. D

20. C

D

1. B

Part 2: Identify 5 errors in the following passage and correct them, (0) has been done as an example. (5 pts) Number

Line

Mistake Page 1 of 7

Correction


1

on

of

1.

2

on

out

2.

4

how

what

3.

7

when

if

4.

10

down

up

5.

12

be

decide

FF IC IA L

0.

O

Part 3. Complete each of the following sentences with a suitable preposition or particle. (10 pts) 1. get round = persuade 2. give oneself up = surrender 3. drive at = imply, suggest 4. come into = inherit 5. fly at = attack (with blows or words)

Ơ

N

Part 4. Complete the following sentences with the words given in the brackets. You have to change the form of the word. (10 pts) 2. reliability

3. single- 4. forefront handedly

5. mathematician

6. positional

7. ownership

8. residence

10. likelihood

H

1. synonymous

Y

N

9. standing

1. C

2. C

3. D

4. A

5. A

6. B

M

Q

U

C. READING (60 points) Part 1: For questions 1–15, read the following passage and decide which answer (A, B, C, or D) best fits each gap. Write your answers in corresponding numbered boxes. (15 pts) 7. A

8. D

9. B

10. A

12. D

13. C

14. A

15. B

11. B

ẠY

Part 2: For questions 1-10, read the text below and think of the word which best fits each space. Use only one word in each space. (15pts)

D

1. that / which 2. are

3. about

4. among

5. performer

6. at

7. same

8. known

9. invented

10. a

11. with

12. dropped

13. later

14. into

15. producing

Part 3: Read the following passage and answer the questions that follow. (15 pts) 1. C

2. D

3. A

4. C Page 2 of 7

5. C


6. C

7. D

8. D

9. B

10. A

Part 4. Read the following passage and do the tasks that follow. (15 pts) 2. a

3. e

4. c

5. h

6. b

7. m

8. d

9. D

10. A

11. D

12. C

13. C

14. C

15. B

FF IC IA L

1. j

D.WRITING (60 pts)

O

Part 1: (5 pts) A/ Finish each of the following sentences in such a way that it is as similar as possible in meaning to the sentence printed before it. (2 pts.) 1. Her primus stove GAVE ITS LAST GASP on the last day. 2. At this time of year, the area often FEEL THE EFFECT OF violent storms.

Y

N

H

Ơ

N

B/ For each of the sentence below, write a new sentence as similar as possible in meaning to the original sentence but using the word given. This word must not be altered in any way. (3 pts.) 3. Trade has gone from bad to worse and staff are being laid off. 4. Don’t breathe a word about your scheme to the boss. 5. Since the advertisement, we have been swamped with applications.

M

Q

U

Part 2: (20 pts) The graphs below give information about computer ownership as a percentage of the population between 2002 and 2010, and by level of education for the years 2002 and 2010. Summarise the information by selecting and reporting the main features, and make comparisons where relevant. Write at least 150 words.

D

ẠY

Part 3. Essay writing (35 pts) In recent years, the family structure and the role of its members are gradually changin. What kinds of changes can ocurr? Do you think these changes are possitive or negative? Write an essay (about 250words) to express your opinion.

Page 3 of 7


LISTENING TAPESCRIPT Part 1: You will hear part of a radio programe in which two people, Sally White and Martin Jones, are discussing the popularity of audio books. For questions 1- 5, choose the

D

ẠY

M

Q

U

Y

N

H

Ơ

N

O

FF IC IA L

answer A, B, C or D which fits best according to what you hear. (10 pts)

Page 4 of 7


Page 5 of 7

ẠY

D KÈ M Y

U

Q N

Ơ

H

N

FF IC IA L

O


Y

N

H

Ơ

N

O

FF IC IA L

Part 2. For questions 1-5, you will hear the historian, George Davies, talking about society and the theatre in England in the time of William Shakespeare. Decide whether the following statements are true (T) or false (F). (10 pts) Scientists may have found one answer to the problem of water shortages. It is a new material that collects water from the air. They got their idea from a small beetle that lives in the desert in Africa. The deserts of the Skeleton Coast in Namibia are very dry, but the Namib desert beetle is an expert at surviving in hot and dry conditions. Its shell is covered in small bumps that collect tiny water droplets from the air. The water is enough to keep the beetle alive. Scientists studied the shape and material of the beetle's bumps. They want to use what they found out about the beetle to make their own materials that can collect water from the air. This will help people who live in very dry areas. The scientists believe this new technology could help in many areas of our life. They said it could be very useful for power plants and for the heating and air conditioning in airplanes, cars and trains. One of the scientists, Philseok Kim, said: "Thermal power plants, for example, rely on condensers to quickly convert steam to liquid water. [Our] design could help speed up that process and even allow for operation at a higher temperature, significantly improving the overall energy efficiency." Another scientist, Joanna Aizenberg, said she was looking forward to the future of getting ideas for new technologies from nature. She said: "Everybody is excited about bio-inspired materials research."

D

ẠY

M

Q

U

Part 3. You hear a radio interview in which the presenter, Terry Davis, is talking to Dr Elizabeth Jones, an expert on climate. Listen to the interview and complete the notes below. Write NO MORE THAN THREE WORDS and/or A NUMBER. (20 pts) Researchers predicted in a new report published in the journal Opthalmology that by 2050, half of the world's population will be short-sighted. The medical term for this condition is myopia. It is also known as near-sightedness. It is when we have difficulty focusing on objects that are far away from us. The researchers say that around 4.8 billion people will suffer from myopia. This is a seven-fold increase in cases from 2000 to 2050. Sixty years ago, around 15 per cent of the Chinese population was short-sighted. Now, almost 90 per cent of Chinese teenagers and young adults have trouble seeing faraway objects. A recent survey found that up 95 per cent of teenagers in South Korea are near-sighted. The researchers said they could not pinpoint definite reasons for this phenomenon and say it could be a combination of factors. They did say that looking at computer screens and mobile phones could be a leading cause. Researchers said the sudden rise in myopia may be linked to "lifestyle changes resulting from a combination of decreased time outdoors and increased Page 6 of 7


D

ẠY

M

Q

U

Y

N

H

Ơ

N

O

FF IC IA L

near-work activities". Science journalist Sarah Zhang said, "spending time outdoors, especially in early childhood, reduces the onset of myopia". Professor Kovin Naidoo, a coauthor of the report, offered some advice. He said: "You could spend a long time reading computers and screens, but also spend two hours outdoors."

Page 7 of 7


Bằng số

FF IC IA L

ĐỀ THI CHỌN HỌC SINH GIỎI LẦN THỨ IX MÔN: TIẾNG ANH - KHỐI 11 Thời gian: 180 phút Đề thi gồm: 14 trang (Thí sinh viết câu trả lời vào bảng cho sẵn trong đề)

SỞ GIÁO DỤC VÀ ĐÀO TẠO TỈNH QUẢNG NINH TRƯỜNG THPT CHUYÊN HẠ LONG

Giám khảo 1

Điểm Bằng chữ

Giám khảo 2

Số phách

N

H

Ơ

N

O

A. LISTENING (40 points) HƯỚNG DẪN PHẦN THI NGHE HIỂU • Bài nghe gồm 3 phần, mỗi phần được nghe 2 lần, mỗi lần cách nhau 15 giây, mở đầu và kết thúc mỗi phần nghe có tín hiệu. • Mở đầu và kết thúc bài nghe có tín hiệu nhạc. Thí sinh có 3 phút để hoàn chỉnh bài trước tín hiệu nhạc kết thúc bài nghe. • Mọi hướng dẫn cho thí sinh (bằng tiếng Anh) đã có trong bài nghe.

D

ẠY

M

Q

U

Y

Part 1: You will hear part of an interview with a woman who describes her experience during an earthquake. For questions 1-5, choose the answer A, B, C or D which fits best according to what you hear. (10 pts) 1. When Julie's car went out of control? A. She knew straight away it was an earthquake. B. She suspected a flat tyre. C. They had to get out for their own safety. D. They thought some work needed doing to it. 2. What frightened her most about the incident? A. a possible fire. B. the up-and-down movements. C. the loss of balance. D. a distant sound. 3. The actual earthquake lasted for _____. A. several seconds B. several minutes C. a couple of hours D. several hours 4. Aftershocks mean that _____. A. there is no time to be frightened. B. a bigger earthquake is coming. Page 1 of 14


FF IC IA L

C. you begin to feel very frightened. D. all you want to do is run away. 5. Before she left Japan, Julie _____. A. wanted to get back to the hotel. B. experienced a totally different earthquake. C. refused to be left on her own. D. was constantly disturbed by aftershocks. Your answers: 1. 2. 3.

4.

5.

N

H

Ơ

N

O

Part2. For questions 1-5, you will hear part of a radio discussion about iris recognition system. Decide whether following statements are True (T) or False (F). (10 pts) 1. ______ Jim says that the idea behind iris recognition systems (IRS) was thought of many years ago. 2. ______ Jim believes that iris recognition machines will be adopted on a large scale chiefly because they can be depended on. 3. ______ Iris recognition machines were used at a school to improve efficiency at a school canteen. 4. ______ Jim feels that people who object to iris recognition machines regard them as a threat to personal freedom. 5. _____ According to Jim, reduced expenses will convince government to adopt iris recognition system. Your answers: 1. 2. 3. 4. 5.

U

Y

Part 3. You will hear a radio programme about Bio-mimicry, the science of copying nature in order to create new technologies. Listen and complete the notes below. Write NO MORE THAN THREE WORDS and/or A NUMBER. (20 pts)

D

ẠY

M

Q

1. Biomimicry imitates nature’s structures, processes and ....................................................... to create new ideas. 2. Velcro was developed after its inventor observed the sticking qualities of one particular plant’s ....................................................... 3. Wood has the beneficial feature of being able to ....................................................... 4. The synthetic ‘wood’ created for the post office will not ....................................................... according to surrounding environmental conditions. 5. The Namibian Fog Basking beetle uses its ability to ....................................................... in order to live in harsh conditions. 6. A ....................................................... on the beetle’s shell repels water and aids the formation of large droplets. 7. The Sahara Forest project utilized this concept in the design of a ....................................................... 8. Systems are ....................................................... in the natural world, in which waste from one organism goes on to become nutrients for another. 9. The ‘Able Project’ uses ....................................................... from the composting process as an input to the fish farm.

Page 2 of 14


10. The extinction of one species of ....................................................... finding a cure for excess stomach acid.

has prevented scientists from

Your answers: 2.

3.

4.

5.

6.

7.

8.

9.

10.

FF IC IA L

1.

D

ẠY

M

Q

U

Y

N

H

Ơ

N

O

B. LEXICO-GRAMMAR (30 points) Part 1: Choose the word or phrase (A, B, C or D) which best completes each sentence. (20 pts) 1. At the start of the course, everyone is assigned a(n) ____ of studies. A. advisor B. consultant C. counselor D. guide 2. You are bound to find information on the stock market crash of 1987 in the newspaper ______. A. files B. archives C. records D. collections 3. After the concert, everyone had to ______ home through the thick snow. A. trudge B. tread C. trace D. trickle 4. She had made a firm decision and wasn’t ______by anything I said against it. A. detracted B. prevailed C. induced D. swayed 5. This disagreement is likely to ______relations between the two countries. A. disaffect B. alienate C. sour D. estrange 6. As the evening______ on, I became more and more bored. A. passed B. drew C. stuck D. wore 7. He was ______ of the press because they had written some terrible things about him in the past. A. alert B. wary C. guarded D. tentative 8. If you want to be a rock star, talent helps, but what it really______ down to is luck. A. boils B. revolves C. centres D. refines 9. Although the patient received intensive treatment, there was no______ improvement in her condition. A. decipherable B. legible C. discernible D. intelligible 10. Some novels can be killed______ dead by bad reviews when they are first published. A. axe B. hammer C. rock D. stone 11. You can exercise your ____________ to cancel the contract immediately, but you wouldn't receive any money at that point. A. duty B. obligation C. right D. possibility 12. We were all in ____________ of the fact that the new manager was our old friend Duncan. A. surprise B. shock C. awe D. amazement 13. My uncle Warren is a thoroughly despicable character. His one ____________ feature is that he loves children. A. saving B. recovering C. improved D. redeeming 14. I don't think anyone understood what I was saying at the meeting, did they? I totally failed to get my point ____________. A. around B. about C. across D. along 15. There was a veritable ____________ of angry phone calls from members of the public complaining about the new controversial series on TV. A. gale B. flood C. storm D. earthquake Page 3 of 14


N

O

FF IC IA L

16. The sales practices of this company will have to be totally ____________ if we are to save it. A. overthrown B. overhauled C. overrun D. overwhelmed 17. For lunch, I always have something quick and easy: a sandwich, a salad, toast and the ____________. A. same B. similar C. like D. rest 18. The government announced today that they intend to ____________ the rise of crime in the inner cities by increasing police budgets in the areas most affected. A. beat B. win C. retaliate D. counter 19. I went to see the boss about a pay rise and he brushed me ____________ with a weak excuse about a business dinner and left me standing there! A. up B. away C. around D. off 20. The new teacher was taken advantage of by the students and often had to ____________ her authority. A. assert B. confirm C. inflict D. strike Your answers 1. 2. 3. 4. 5. 6. 7. 8. 9. 10. 11. 12. 13. 14. 15. 16. 17. 18. 19. 20.

KĂˆ

M

Q

U

Y

N

DISAPPEARING WORLD The destruction of the rainforests is a pressed problem of our times but not one that is regarded equally seriously by everyone. The more affluent nations regard the issue as one of preservation; deforestation must stop. When it comes to the poorer countries, the issue is not so cut and dried. For these people, the rainforests represent a source of economic prosperity, a point that obviously takes precedence on ecological concerns. A solution must be found before the damage caused by the deforestation that is destroying the rainforests becomes irrevocable. Deforestation is carried out by those involving in the timber industry and also by migrant farmers. The latter occupy an area of land, strip it, farm it until its natural mineral supply is used up and then move on. The land is left useless and exposed and a process of erosion comes into effect, washing soil into rivers thereby killing fish and blocking the water's natural course. The land is not the only victim. Rainforests are a rich populated habitat. In the rainforests of Madagascar there are at most 150,000 individual species of plants and animals which are found anywhere else in the world and more are being discovered all the time. Furthermore, approximately 50% of all endangered animal species live in the world's rainforests. The destruction of the forests effectively represents a complete removal of all these plants and animals. Deprived of their natural environments, they will disappear altogether. Again, this process is reversible. Man, no matter how powerful he considers himself, does not have the power to establish the species he is so willfully destroying.

D

áş Y

Line 0 1 2 3 4 5 6 7 8 9 10 11 12 13 14 15 16 17

H

Ć

Part 2: Identify 5 errors in the following passage and correct them, (0) has been done as an example. (5 pts)

Your answer: Line Mistake

Correction Page 4 of 14


0

pressed

FF IC IA L

pressing

H

Ơ

N

O

Part 3. Complete each of the following sentences with a suitable preposition or particle. (5 pts) 1. Breaking his leg a second time put Peter’s football career ………………….. jeopardy. 2. Bob, don't be cruel and stop mocking …………………. the way Lucy pronounces French words. 3. Their situation was critical. They had no map, their water supplies were running low and they had no food to subsist …………………. . 4. Please do ................ your safety belt before we begin the journey 5. He shot the bird while it was …………………. the wing. Your answers: 1. 2. 3. 4. 5.

D

ẠY

M

Q

U

Y

N

Part 4. Complete the following sentences with the words given in the brackets. You have to change the form of the word. (10 pts) SKIING HOLIDAYS IN COLORADO To ski or snowboard in Colorado is to experience the pinnacle of winter sports. The state of Colorado is known for its spectacular scenery and (1. BREATH) _____________________ views, which inspire today's travelers as much as they spurred on the (2. SETTLE) _____________________ who first arrived in this part of the US over a century ago. And whether you're seeking the outdoor adventure of a (3. LIFE) _____________________ exciting nightlife or a great family getaway, Colorado has everything you need. November through April, snow conditions are (4. CONSIST) _____________________ and reliable, featuring Colorado's (5. LEGEND) _____________________ “champagne powder” snow. Extensive snow making and grooming operations always keeps trails in top shape. The mountain destinations in the Colorado Rockies can turn your wildest ski dreams into thrilling (6. REAL) ____________________. There, you'll find the best skiing and snowboarding resorts on (7. PICTURE) _____________________ slopes, as well as the finest ski schools in the US. Together, they present an (8. PARALLEL)_____________________ winter paradise. And the best part is that you'll enjoy friendly, (9. CARE) _____________________ service in resorts that are (10. COMMIT) _____________________ to delivering the highest quality amenities. Your answer 1.

3.

5.

7.

9.

2.

4.

6.

8.

10.

Page 5 of 14


ẠY

M

Q

U

Y

N

H

Ơ

N

O

FF IC IA L

C. READING (60 points) Part 1: For questions 1–15, read the following passage and decide which answer (A, B, C, or D) best fits each gap. Write your answers in corresponding numbered boxes. (15 pts) According to research (1)_______ by the US scientists, global warming could (2)_______ the growth of toxic algae and bacteria in the world’s seas and lakes, with a(n) (3)_______ that may be felt in ten years‟ time. Studies have shown that shifts (4)_______ by climate change make ocean and freshwater environments more (5)_______ to toxic algae blooms and allow harmful microbes and bacteria to (6)_______. In one study, scientists modeled future ocean and weather (7)_______ to predict the effects on blooms of Alexandrium catenella, or the toxic „red tide‟, which can (8)_______ in shellfish and cause severe symptoms, (9)_______ paralysis, in humans who eat the contaminated seafood. In another study, scientists found that desert dust, which contains iron, (10)_______ into the ocean from the atmosphere could (11)_______ to increases of harmful bacteria in the seawater. Also, adding desert dust to seawater significantly (12)_______ the growth of Vibrios, a group of ocean bacteria that can cause gastroenteritis and (13)_______ diseases in human beings. Global warming has also been (14)_______ for rising ocean temperatures, and a warming ocean, which we know is happening, increases the (15)_______ of disease that affects both wildlife and humans. 1. A. made B. launched C. conducted D. released 2. A. indulge B. spur C. nurture D. deaden 3. A. impact B. improvement C. target D. affection 4. A. broken down B. come round C. given in D. brought about 5. A. weakened B. susceptible C. influential D. incessant 6. A. reduce B. build C. prosper D. proliferate 7. A. plans B. frames C. patterns D. forecasts 8. A. accumulate B. shatter C. decline D. hold 9. A. consisting B. comprising C. composing D. including 10. A. blown B. deposited C. carried D. fallen 11. A. bring B. cause C. lead D. get 12. A. motivated B. advocated C. stimulated D. aroused 13. A. genetic B. curable C. innate D. infectious 14. A. blamed B. enforced C. mistaken D. faulted 15. A. opportunity B. likelihood C. feasibility D. commonness Your answers 1. 2. 3. 4. 5. 6. 7. 8. 9. 10. 11. 12. 13. 14. 15.

D

Part 2: For questions 1-10, read the text below and think of the word which best fits each space. Use only one word in each space. There is an example at the beginning (0). (15pts) DANGERS OF TECHNOLOGY Much has been heard recently (0) about possible health hazards, including memory loss and brain tumours, from the use of mobile phones. With the possible half a billion mobile phones in (1) __________ throughout the world, in Britain alone, one person in four owns one, (2) ____________ is worrying enough, even if, so far, no concrete evidence has come to (3) _____________. Page 6 of 14


O

FF IC IA L

One study by Dr. Alan Preece and his team at Bristol University has shown, however, in a report in the International Journal of Radiation Biology, that tests on volunteers demonstrated no effect on their short-term memory or attention span. Subjects were exposed to microwave radiation for (4) ______________ to thirty minutes, but the one noticeable effect was positive rather than negative; the subjects reacted more rapidly in one test to a visual choice. One explanation of (5) _____________ is that following the transmissions, a warming of the blood led to increased blood flow. For the experiment, places were chosen where the signal was good and the microwave dose light, and then where the signal was poor and the dose (6) _____________ higher. The subjects were tested for recall and mental alertness (7) ___________ exposure to microwaves characteristic of analogue phones, digital phones or no phones at all, without knowing (8) ___________ they were exposed to. It is, of course, early days (9) ____________ and the sample may not be large (10) ______________ to generalize from. More research needs to be done. Your answer 1. 2. 3. 4. 5. 6. 7. 8. 9. 10.

D

ẠY

M

Q

U

Y

N

H

Ơ

N

Part 3: Read the following passage and answer the questions that follow. (15 pts) COMMUNICATING WITH THE FUTURE In the 1980s the United States Department of Energy was looking for suitable sites to bury radioactive waste material generated by its nuclear energy programs. The government was considering burying the dangerous wastes in deep underground chambers in remote desert areas. The problem, however, was that nuclear waste remains highly radioactive for thousands of years. The commission entrusted with tackling the problem of waste disposal was aware that the dangers posed by radioactive emissions must be communicated to our descendants of at least 10,000 years hence. So the task became one of finding a way to tell future societies about the risk posed by these deadly deposits. Of course, human society in the distant future may be well aware of the hazards of radiation. Technological advances may one day provide the solutions to this dilemma. But the belief in constant technological advancement is based on our perceptions of advances made throughout history and prehistory. We cannot be sure that society won’t have slipped backward into an age of barbarism due to any of several catastrophic events, whether the result of nature such as the onset of a new ice age or perhaps mankind’s failure to solve the scourges of war and pollution. In the event of global catastrophe, it is quite possible that humans of the distant future will be on the far side of a broken link of communication and technological understanding. The problem then becomes how to inform our descendants that they must avoid areas of potential radioactive seepage given that they may not understand any currently existing language and may have no historical or cultural memory. So, any message indicated to future reception and decipherment must be as universally understandable as possible. It was soon realized by the specialists assigned the task of devising the communication system that material in which the message was written might not physically endure the great lengths of time demanded. The second law of thermodynamics shows that all material disintegrates over time. Even computers that might carry the message cannot be expected to endure long enough. Besides, electricity supplies might not be available in 300 generations. Other media storage methods were considered and rejected for similar reasons. The task force under the linguist Thomas Sebeok finally agreed that no foolproof way would be found to send a message across so many generations and have it survive physically and be decipherable Page 7 of 14


D

ẠY

M

Q

U

Y

N

H

Ơ

N

O

FF IC IA L

by a people with few cultural similarities to us. Given this restriction, Sebeok suggested the only possible solution was the formation of a committee of guardians of knowledge. Its task would be to dedicate itself to maintaining and passing the knowledge of the whereabouts and dangers of the nuclear waste deposits. This so-called atomic priesthood would be entrusted with keeping knowledge of this tradition alive through millennia and developing the tradition into a kind of mythical taboo forbidding people to tamper in a way with the nuclear waste sites. Only the initiated atomic priesthood of experts would have the scientific knowledge to fully understand the danger. Those outside the priesthood would be kept away by a combination of rituals and legends designed to warn off intruders. This proposal has been criticized because of the possibility of a break in continuity of the original message. Furthermore, there is no guarantee that any warning or sanction passed on for millennia would be obeyed, nor that it could survive with its original meaning intact. To counterbalance this possibility, Sebeok’s group proposed a “relay system” in which information is passed on over relatively short periods of time, just three generations ahead. The message then to be renewed and redesigned if necessary for the following three generations and so on over the required time span. In this way information could be relayed into the future and avoid the possibility of physical degradation. A second defect is more difficult to dismiss, however. This is the problem of social exclusiveness brought about through possession of vital knowledge. Critics point out that the atomic priesthood could use its secret knowledge to control those who are scientifically ignorant. The establishment of such an association of insiders holding powerful knowledge not available except in mythic form to nonmembers would be a dangerous precedent for future social developments. 1. The word "chambers" in the passage is closest in meaning to A. partitions B. openings C. cavities D. fissures 2. What problem faced the commission assigned to deal with the burial of nuclear waste? A. How to reduce the radioactive life of nuclear waste materials B. How to form a committee that could adequately express various nuclear risks C. How to notify future generations of the risks of nuclear contamination D. How to choose burial sites so as to minimize dangers to people. 3. In paragraph 2, the author explains the possible circumstances of future societies A. to warn about the possible natural catastrophe B. to question the value of advances C. to highlight humankind's inability to resolve problems D. to demonstrate the reason nuclear hazards must be communicated 4. The word "scourges" in the passage is closest in meaning to A. pressures B. afflictions C. worries D. annoyances 5. In paragraph 4, the author mentions the second law of thermodynamics A. to support the view that nuclear waste will disperse with time B. to show that knowledge can be sustained over millennia C. to give the basic scientific reason behind the breakdown of material objects D. to contrast the potential life span of knowledge with that of material objects 6. The word "Its" in the passage refers to A. knowledge B. committee C. solution D. guardians 7. In paragraph 5, why is the proposed committee of guardians referred to as the "atomic priesthood"? A. Because they would be an exclusive group with knowledge about nuclear waste sites. B. Because they would use rituals and legends to maintain their exclusiveness C. Because they would be an exclusive religious order Page 8 of 14


H

Ơ

N

O

FF IC IA L

D. Because they would develop mythical taboos surrounding their traditions 8. According to the author, why did the task force under Sebeok propose a relay system for passing on information? A. To show that Sebeok 's ideas created more problems than they solved B. To support the belief that breaks in communication are inevitable over time C. To contrast Sebeok's ideas with those proposed by his main critics D. To compensate for the fact that meaning will not stable over long periods of time 9. According to paragraph 7, the second defect of the atomic priesthood proposal is that it could lead to A. the nonmembers turning knowledge into dangerous mythical forms B. the possible misuse of exclusive knowledge C. the establishment of a scientifically ignorant society D. the priesthood's criticism of points concerning vital knowledge 10. All of the following are mentioned in the passage as difficulties in devising a communication system with the future EXCEPT A. the failure to maintain communication link B. the loss of knowledge about today's civilization C. the inability of materials to endure over time D. the exclusiveness of priesthood Your answer 1. 2. 3. 4. 5. 6. 7. 8. 9. 10.

D

ẠY

M

Q

U

Y

N

Part 4: Read the following passage and do the tasks that follow. (15 pts) Australia's Convict Colonies A. The 1700s in Britain saw widespread poverty and rising crime, and those convicted of crimes faced harsh penalties, including transportation to one of Britain's overseas colonies. Since 1615, convicts had been transported to Britain's America colonies, both as punishment and a source of labour, but this practice was halted by the Revolutionary War in America (1775 - 1783). The British government decided to establish a new prison colony, and Botany Bay in New South Wales was chosen as the site. (Captain Cook, exploring the southeast coast of Australia in 1770, had name the land New South Wales and claimed it for Britain.) Between 1787 and 1868, almost 160,000 convicts, of whom about 25,000 were women, were sent to Australia to serve sentences ranging from 7 years to life. B. Eleven ships set sail from English in 1787 to take the first group of about 750 British convicts to Australia. The fleet reached Botany Bay in 1788, but nearby Sydney Cove was selected as a more suitable site for the new settlement, which later became the city of Sydney. The first few years were difficult, with severe food shortages; by 1792, however, there were government farms and private gardens. Convicts worked on these farms, or on construction projects such as building roads and bridges. Although the settlement was a prison colony, few convicts served their sentences in jail. They lived in houses they had built themselves, and established families, businesses and farms. A settlement was also established on Norfolk Island, where some convicts were sent for crimes committed after arrival in the colony. Two more settlements were established on Van Diemen's Land (now Tasmania), in 1803 and 1804. C. Convicts not involved in public work were assigned to free settlers, providing labor in exchange for food, clothing and shelter. Some masters treated the convicts cruelly, and the punishment of convicts, particularly in the early days, could be arbitrary and savage. Lachlan Macquire, governor of Page 9 of 14


Q

U

Y

N

H

Ć

N

O

FF IC IA L

New South Wales from 1809 to 1819, adopted a more humane approach. He encouraged convicts to reform by rewarding good behavior, even granting pardons to convicts before their sentence was completed. These emancipists, as they were called, were given land and government assistance to help them start farming. His policies were unpopular both with British authorities and wealthy free settlers, however, and the next governors were under orders to ensure that life for convicts became much stricter and more controlled. There were harsher punishments for second offenders, such as working in the "iron gangs", where men were chained together to carry out exhausting work on the roads, or being sent to penal settlements where punishment was deliberately brutal so that it could act as a deterrent. D. In the early years of settlement, the convicts greatly outnumbered free immigrants and settlers. In 1810, convicts made up almost 60 percent of the population, and over 20,000 new convicts arrived between 1821 and 1830. Even in 1831, convicts still comprised 45 percent of the population, with exconvicts and emancipists making up another 30 percent. 25 percent of the population now consisted of people born in the colonies, and free people outnumbered convicts. E. The first group of free settlers had arrived in Australia in 1793 to seek their fortune in the new land. Their numbers grew, with about 8,000 free settlers arriving in the 1820s to take advantage of free land grants and cheap convict labour. In 1831, the British government offered money to support new settlers, hoping to attract skilled workers and single women as immigrants. Between 1831 and 1840, more than 40,000 immigrants arrived in Australia. F. During the 1820s there as a lengthy campaign to win certain right for emancipists, which was opposed by wealthy free settlers. In the 1830s, free immigrants to New South Wales and Van Diemen's Land, unhappy about living in a prison colony where civil liberties were restricted and convict labour resulted in low wages, increasingly voiced their opposition to transportation. Again, wealthy landowners disagreed, but a growing number of reformers in England were also opposed to convict transportation. In 1838, a committee set up by the British Parliament recommended that the government end transportation to New South Wales and Van Diemen's Land, and abolish assignment. The British duly abolished assignment, and transportation - at least to New South Wales - was halted in 1840. Questions 1 - 5: The reading passage has seven paragraphs A - G. Choose the correct heading for paragraphs B - G from the list of headings below.

D

áş Y

KĂˆ

M

List of Headings i. Free settlers ii. Transportation of convicts iii. The first settlements iv. Convict life v. The colonial population vi. The treatment of convicts vii. Opponents of transportation

Example: Paragraph A 1. Paragraph B 2. Paragraph C 3. Paragraph D 4. Paragraph E 5. Paragraph F

______ii____ __________ __________ __________ __________ __________ Page 10 of 14


Your answers: 1.

2.

3.

4.

5.

N

O

FF IC IA L

Questions 6 -10: Complete the notes below. Write NO MORE THAN THREE WORDS for each answer. Australia's Convicts Colonies Events preceding first settlement - 1615 - convicts first transported to (6) ________________ controlled by Britain - 1770 - Cook claims SE Australian coast for Britain, calling it (7) ________________ - 1775 - 1783 - Revolutionary War in America halts transportation there - 1787 - Botany Bay chosen as site for new (8) ______________________; first convict fleet sets sail - 1788 - fleets reaches Botany Bay but (9) ____________________ chosen instead. - 1838 – a committee established by the British Parliament recommended (10)___________________to end transportation to New Sound Wales and Van Diemen’s Land. Your answer 6. 7. 8. 9. 10.

ẠY

M

Q

U

Y

N

H

Ơ

D. WRITING (60 points) Part 1: Use the word given in brackets and make any necessary additions to write a new sentence in such a way that it is as similar as possible in meaning to the original sentence. Do NOT change the form of the given word. You must use between THREE and EIGHT words, including the word given. There is an example at the beginning (0). (5points) (0) Louise should have been given more time to complete her thesis. INSUFFICIENT Louise was given insufficient time to complete her thesis. 1. We have to think carefully about our other expenses before we decide to buy a new car. TAKEN Our other expenses ____________________________________________before we decide to buy a new car. 2. Yesterday, I seemed to do nothing but answer the phone. SPEND Yesterday, I seemed ______________________________________ the phone. 3. It seems that Amy was offended by what you said. OFFENCE Amy ________________________________________________ what you said. 4. I expect the book to be far better because it had been written by such a good novelist. SHORT The book ______________________________________ it had been written by such a good novelist. 5. If anyone can get lost, he will. DEPENDED He _________________________________________________ get lost.

D

Part 2: The chart below shows the Average Monthly Temperatures for three African cities. Summarize the information by selecting and reporting the main features, and make comparisons where relevant. You should write about 150 words. (20 points)

Page 11 of 14


FF IC IA L

D

ẠY

M

Q

U

Y

N

H

Ơ

N

O

……………………………………………………………………………………………………………… ……………………………………………………………………………….…………………………… ……………………………………………………………………………………………………………… ……………………………………………………………………………………………………….…… …………………………………………………………………………………………………….. ……………………………………………………………………………………………………………… ……………………………………………………………………………………………………………… ……………………………………………………………………………………………………………… ……………………………………………………………………………………………………………… ……………………………………………………………………………………………………………… .…………………………………………………………………………………………………………….. …………………………………………………………………………………………………………….. …………………………………………………………………………………………………………….. …………………………………………………………………………………………………………….. …………………………………………………………………………………………………………….. …………………………………………………………………………………………………………….. …………………………………………………………………………………………………………….. …………………………………………………………………………………………………………….. …………………………………………………………………………………………………………….. ……………………………………………………………………………………………………………… ……………………………………………………………………………………………………………… ……………………………………………………………………………………………………………… ……………………………………………………………………………………………………………… .…………………………………………………………………………………………………………….. …………………………………………………………………………………………………………….. …………………………………………………………………………………………………………….. …………………………………………………………………………………………………………….. …………………………………………………………………………………………………………….. …………………………………………………………………………………………………………….. …………………………………………………………………………………………………………….. …………………………………………………………………………………………………………….. …………………………………………………………………………………………………………….. Page 12 of 14


Part 3: Write an essay of about 250 words on the following topic (35 points) Young people are much more aware of and concerned about issues like the environment, poverty, and animal welfare than previous generations. What is your own opinion?

D

ẠY

M

Q

U

Y

N

H

Ơ

N

O

FF IC IA L

Give reasons for your answer, and include any relevant examples from your knowledge or experience. You may continue your writing on the back page if you need more space .…………………………………………………………………………………………………………….. …………………………………………………………………………………………………………….. …………………………………………………………………………………………………………….. …………………………………………………………………………………………………………….. …………………………………………………………………………………………………………….. …………………………………………………………………………………………………………….. …………………………………………………………………………………………………………….. …………………………………………………………………………………………………………….. …………………………………………………………………………………………………………….. …………………………………………………………………………………………………………….. …………………………………………………………………………………………………………….. …………………………………………………………………………………………………………….. …………………………………………………………………………………………………………….. …………………………………………………………………………………………………………….. …………………………………………………………………………………………………………….. …………………………………………………………………………………………………………….. …………………………………………………………………………………………………………….. …………………………………………………………………………………………………………….. …………………………………………………………………………………………………………….. …………………………………………………………………………………………………………….. …………………………………………………………………………………………………………….. …………………………………………………………………………………………………………….. …………………………………………………………………………………………………………….. …………………………………………………………………………………………………………….. …………………………………………………………………………………………………………….. …………………………………………………………………………………………………………….. …………………………………………………………………………………………………………….. …………………………………………………………………………………………………………….. …………………………………………………………………………………………………………….. …………………………………………………………………………………………………………….. …………………………………………………………………………………………………………….. …………………………………………………………………………………………………………….. …………………………………………………………………………………………………………….. …………………………………………………………………………………………………………….. …………………………………………………………………………………………………………….. …………………………………………………………………………………………………………….. …………………………………………………………………………………………………………….. …………………………………………………………………………………………………………….. Page 13 of 14


D

ẠY

M

Q

U

Y

N

H

Ơ

N

O

FF IC IA L

.…………………………………………………………………………………………………………….. …………………………………………………………………………………………………………….. …………………………………………………………………………………………………………….. …………………………………………………………………………………………………………….. …………………………………………………………………………………………………………….. …………………………………………………………………………………………………………….. …………………………………………………………………………………………………………….. …………………………………………………………………………………………………………….. …………………………………………………………………………………………………………….. …………………………………………………………………………………………………………….. …………………………………………………………………………………………………………….. …………………………………………………………………………………………………………….. …………………………………………………………………………………………………………….. …………………………………………………………………………………………………………….. …………………………………………………………………………………………………………….. …………………………………………………………………………………………………………….. …………………………………………………………………………………………………………….. …………………………………………………………………………………………………………….. …………………………………………………………………………………………………………….. …………………………………………………………………………………………………………….. …………………………………………………………………………………………………………….. …………………………………………………………………………………………………………….. …………………………………………………………………………………………………………….. …………………………………………………………………………………………………………….. …………………………………………………………………………………………………………….. …………………………………………………………………………………………………………….. …………………………………………………………………………………………………………….. …………………………………………………………………………………………………………….. …………………………………………………………………………………………………………….. …………………………………………………………………………………………………………….. …………………………………………………………………………………………………………….. …………………………………………………………………………………………………………….. …………………………………………………………………………………………………………….. …………………………………………………………………………………………………………….. …………………………………………………………………………………………………………….. …………………………………………………………………………………………………………….. …………………………………………………………………………………………………………….. …………………………………………………………………………………………………………….. …………………………………………………………………………………………………………….. …………………………………………………………………………………………………………….. …………………………………………………………………………………………………………….. …………………………………………………………………………………………………………….. …………………………………………………………………………………………………………….. …………………………………………………………………………………………………………….. ......................THE END................... Page 14 of 14


SỞ GIÁO DỤC VÀ ĐÀO TẠO TỈNH QUẢNG NINH TRƯỜNG THPT CHUYÊN HẠ LONG

ĐÁP ÁN ĐỀ THI CHỌN HỌC SINH GIỎI LẦN THỨ IX MÔN: TIẾNG ANH - KHỐI 11 (Đáp án gồm 5 trang)

N

O

FF IC IA L

A. LISTENING: (40 pts) Part 1 Part 1: You will hear part of an interview with a woman who describes her experience during an earthquake. For questions 1-5, choose the answer A, B, C or D which fits best according to what you hear. (10 pts) 2 pts/correct answer. 1. B 2. D 3. A 4. C 5. D Part2. For questions 1-5, you will hear part of a radio discussion about iris recognition system. Decide whether following statements are True (T) or False (F). (10 pts) 2 pts/correct answer. 1. T 2. T 3. T 4. T 5. F Part 3. You will hear a radio programme about Bio-mimicry, the science of copying nature in order to create new technologies. Listen and complete the notes below. Write NO MORE THAN THREE WORDS and/or A NUMBER. (20 pts) 2 pts/correct answer. 1. systems 6. waxy finish 2. seed heads 7. greenhouse 3. hold nails 8. cyclical 4. change shape 9. worms 5. harvest moisture 10. frog

Q

U

Y

N

H

Ơ

B. LEXICO-GRAMMAR (60 points) Part 1: Choose the word or phrase (A, B, C or D) which best completes each sentence. (20 pts) 1pts/correct answer. 1. A 6. D 11. C 16. B 2. B 7. B 12. C 17. C 3. A 8. A 13. D 18. D 4. D 9. C 14. C 19. D 5. C 10. D 15. B 20. A Part 2: Identify 5 errors in the following passage and correct them, (0) has been done as an example. 5 pts) (1pt / each correct answer) one identified error: 0,5pt, one right corrected error: 0,5pt

4

on

Correction over

2

7

involving

involved

3 4

11 12

most anywhere

least nowhere

5

16

reversible

irreversible

M

Number

D

ẠY

1

Line

Mistake

Part 3. Complete each of the following sentences with a suitable preposition or particle. (5 pts) 1 pts/correct answer. 1. in 2. at 3. on 4. up 5. on Part 4. Complete the following sentences with the words given in the brackets. You have to change the form of the word. (10 pts) 1 pts/correct answer 1. breathtaking 2. settlers 3. lifetime 4. consistent 5. legendary

6. reality

7. picturesque

8. unparalleled

C. READING (60 points) Page 1 of 6

9. caring

10. committed


O

FF IC IA L

Part 1: (15 points – 1 point/correct answer) (CAE) 1. C 2. B 3. A 4. D 5. B 6. D 7. C 8. A 9. D 10. B 11. C 12. C 13. D 14. A 15. B Part 2: For questions 1-10, read the text below and think of the word which best fits each space. Use only one word in each space. There is an example at the beginning (0). (15 pts) 1.5 pts/correct answer 1. use 2. which 3. light 4. up 5. this 6. much 7. After/ following 8. what 9. yet 10. enough Part 3: Read the following passage and answer the questions that follow. (15pts) 1.5 pts/correct answer (IELTS) 1. C 2. C 3. D 4. B 5. C 6. B 7. A 8. D 9. B 10. D Part 4. Read the following passage and do the tasks that follow. (15 pts) 1.5 pts/correct answer (IELTS PRACTICE) 1. iii 6. American colonies 2. vi 7. New South Wales 3. v 8. prison colony 4. i 9. Sydney Cove 5. vii 10. the government D. WRITING (60 pts)

D

ẠY

M

Q

U

Y

N

H

Ơ

N

Part 1: 1pts/correct answer Use the word given in brackets and make any necessary additions to write a new sentence in such a way that it is as similar as possible in meaning to the original sentence. Do NOT change the form of the given word. You must use between THREE and EIGHT words, including the word given (5 pts.) 1. Our other expenses must be taken into careful consideration/account before we decide to buy a new car 2. Yesterday, I seemed to spend the whole day / all day answering the phone 3. Amy seems to have taken offence at what you said 4. The book fell short of my expectations even though it had been written by such a good novelist 5. He can be depended on to get lost Part 2: (20 points) 1. Completion: 4 pts 2. Content: 5 pts - Cover the main information in the chart yet not go into too many details. - Make general remarks and effective comparisons. 3. Organisation: 4 pts - The ideas are well organized - The description is sensibly divided into paragraphs 4. Language: 4pts - Use a wide range of vocabulary and structure - Good grammar 5. Punctuation and spelling: 3 pt Part 3: (35 points) 1. Content: (40%) - Providing all main ideas and details as required

2. Language: (40%)

-

Communicating intentions sufficiently and effectively

-

Demonstration of a variety of vocabulary and structures appropriate to the level of English language gifted uppersecondary school students Page 2 of 6


Good use and control of grammatical structures

-

Good punctuation and no spelling mistakes

-

Legible handwriting

-

Ideas are well organized and presented with coherence, cohesion, and clarity

-

The essay is well-structured

LISTENING TRANSCRIPT

FF IC IA L

3. Organization and Presentation: (20%)

-

D

ẠY

M

Q

U

Y

N

H

Ơ

N

O

Part 1: (CAE) Presenter: With me in the studio is Julie Fields. Julie was unlucky enough to be on holiday in Japan when the recent earthquake struck. Julie, I’m glad you’re here to tell the tale. We don’t have too many earthquakes here in Britain, so not many of our listeners have first-hand experience of what it is like to be present at, quite literally, an earth-shaking event such as this. What happened to you? Julie: We were driving – it was just the two of us – when our car started to veer wildly from side to side. For a moment, we thought it was a flat tyre, so we pulled over and got out. Then we realized that the earth was still shaking. It seemed to stop after a few seconds … and remember we still hadn’t worked out what it was at this stage … then it came back, with a vengeance! I think that’s the point when Paul said, “It’s an earthquake!" The ground shook violently from side to side, really jerky movements, as I remember. Then there was some up and down movement. I was holding on to the top of the car to try and keep my balance, and Paul was kind of bent double, unable to straighten up or get a foothold on anything. And we could hear rocks falling somewhere, which was the really scary thing. Anyway, the next thing was these plumes of what looked like smoke, but turned out to be dust, rising into the air. That was weird, and I remember staring at them, trying to work out what it was. After what seemed like hours, the earth stopped moving and the sound of the rocks faded away. It’d only been a few seconds, I suppose. We couldn’t move for a few minutes, we were both shaking and I was crying hysterically. I’d never known anything like it, but I had absolutely no control over my emotions. I was crying so hard that I couldn’t catch my breath. Presenter: A harrowing experience. You say you were crying. Was that out of fear, do you think? Julie: No. I’ve thought about this a lot. I don’t think it could have been fear because, after all, I didn’t start crying till after it had stopped. I don’t know what it was. All I can say is that there is no time to be frightened at first because you don’t recognize it while it is happening. Then, when the aftershocks start … that’s when the fear sets in, because when it starts up again you have a very clear memory of the big one and you expect the aftershock to do the same. There’s also the worry that the next one could be even bigger. I’m told that’s quite rare – that they usually diminish in size – but your brain doesn’t work rationally when you’re gripped by that kind of fear. My thought processes in the first few hours following the earthquake were very muddled. I wanted to run away, to stay still, to be airlifted, to go and dive into the sea somewhere – I didn’t know what I wanted to do. I had a really hard time coming to terms with the fact that the one thing you could take for granted – the earth being beneath your feet – couldn’t be taken for granted after all. And every time there was a little shake – or a big shake – I had a different response. Presenter: How long did you stay there after the earthquake? Julie: We were there for another five days afterwards. Presenter: And what were you like during that time? Did you begin to get over it? Julie: Well that was another strange thing. The more we heard other people’s stories, the jumpier we became. I mean, the aftershocks continued the whole time, so we hardly slept at all. It was one thing to be out in the open when it happened, and in a way, I’m glad we were, but back in the hotel, the sounds and the sensations were very different when there was a tremor. We seemed to spend the Page 3 of 6


whole time rushing out of the building, waiting for a while, then slowly going back, in twos and threes. Until the next one, it was like that day and night. Presenter: Julie, it sounds as though you won’t forget it for a long time, if ever. We’re glad you’re back safe and sound on terra firma. Thanks for coming in today to talk to … (Fade) PART 2 (IELTS) Interviewer: My guest on Technology Matters this week is Jim Davies, a leading expert in the field of IRS or iris recognition systems. Jim, perhaps I could start by asking you to explain exactly what IRS is?

FF IC IA L

Jim: In fact, it’s a simple system in theory, one that was first suggested back in the 1930s. Basically, it’s a way of recognizing a person by analyzing the pattern of his or her iris. This pattern is different for every individual on the planet; even identical twins have different iris patterns. The way it works is, you have a camera linked to computer software that can compare the iris pattern it sees with iris patterns on a database; the computer makes a match and then reports on the identification. Interviewer: It just provides a report?

O

Jim: No. That’s simply the first step. An iris recognition machine can be connected to any number of devices. For instance, at airports it will be possible for barriers to be opened by a machine, and this will, in turn, speed the flow of passengers through checkpoints. I must say, though, I don’t think it’s speed that is the biggest appeal of iris recognition machines – the fact that they are reliable will guarantee their popularity in the future.

N

Interviewer: Are iris recognition machines actually being used at the moment?

N

H

Ơ

Jim: Oh, yes. As a matter of fact, we’ve just completed a pilot scheme in northern England. We installed machines at a school there to identify pupils as they came into the canteen. That way they could be given their correct meals automatically, which meant they didn’t have to wait around to be served. It was a great success! And the kids loved it. I think they regarded it as something out of a science fiction film. Interviewer: What about adults? Do you think they will be as impressed?

M

Q

U

Y

Jim: That’s a good question. I admit there are many people who feel that the use of iris recognition machines is a civil liberties issue and infringes on their privacy, but I think people said the same thing about the use of X-ray machines at airports, and now everyone accepts them. So I’m confident that the vast majority of people will come to see the sense behind using these machines, especially when they realize how efficient they are. When they’re properly set up, they take a mere twelve seconds to scan someone’s iris, and of course the customs people themselves are very attracted to the idea because of the time it saves. Interviewer: So is it simply a matter of time before we find iris recognition machines everywhere?

Jim: Nobody knows as yet just how to widespread they’ll be. A lot depends on how quickly the public comes to accept them, and I think the government wants to monitor public reaction before committing itself to the technology.

ẠY

Interviewer: What about the costs involved?

D

Jim: The computerized cameras themselves don’t cost a great deal, but the really huge cost will be when we have to register the whole population. I can quite understand that the government might be nervous about this sort of expense, although of course you wouldn’t ever have to repeat it on such a huge scale once you’d done it. Interviewer: So you believe the future lies with IRS? Jim: I do, yes, primarily because every government wants to be able to confirm identity at places like airports, and iris recognition machines are simply the most effective way of doing this as yet available to us …

Page 4 of 6


D

ẠY

M

Q

U

Y

N

H

Ơ

N

O

FF IC IA L

PART 3 (CAE) Architecture, chemical processes, material sciences – all these faculties are beginning to be touched by ideas which are inspired by nature. Where in, the past, man’s inventions have been considered allpowerful, engineers are now turning to nature, over the last 3.8 billion years, has developed a wealth of successful innovations. So, over the last decade there has been a new wave of scientific interest in Biomimicry, measures which emulate nature’s genius. And we are not restricted to copying the structures found in nature. We can also imitate its processes and its systems. One of the most famous products inspired by nature is the sticky fastening, Velcro, invented in 1941 by the Swiss engineer George de Mestral, who noticed how the seed heads of burdock got tangled in his clothes and his dog’s coat. Meanwhile, engineers at the Centre of Biomimetics at Reading University, have developed a material based on the properties of wood. One of the reasons why wood is so useful to us is that its ability hold nails when they are driven into it. This property is down to the structure of hollow cells, and fibres which hold the nail in place. The new material uses equivalent fibres made of glass fibres, and resin to hold everything in place. The result is a material with the same geometric arrangement as wood, but which has none of the disadvantages of wood – that it changes shape according to moisture levels and temperature. The substitute wood has been developed for the post office. They wanted a container which could contain threatening packages, directing the blast safely and holding any explosive fragments. But the second realm of Bio-mimetic is imitating not the just physical form of natural substances, but also the processes that take place in nature. A perfect example of this is the Sahara Forest Project, which has utilised a processed used by the Namibian Fog Basking Beetle. This beetle is able to live in very harsh, desert conditions using its ability to harvest moisture from the air. The beetle comes out into the desert only at night and, due to its black shell, is able to maintain a temperature cooler than that of its surroundings. The moist breeze blows in from the sea, and droplets of water condense on the beetles shell. At sunrise, the beetle lifts its shell, drinks and returns to its hole. What is even more interesting is that the beetle has hydrophilic bumps on its shell which attract water, separated by a waxy finish which repels water. Consequently, the water gathers in tight, spherical droplets, so that hardly any water is wasted. The Sahara Forest Project uses a process which is, in effect, identical to that of the beetle. Seawater evaporates from the front wall of the greenhouse to create ideal cool growing conditions, and condenses to form fresh water on the downwind side of the greenhouse. However, just mimicking the structure and processes in nature is not enough, if, in doing so, we create tones of waste. Human designs are linear. We take resources from the earth, use what we need, and the rest goes to waste. But in the natural world, systems are cyclical. Waste from one organism goes on to become nutrients for another. One scheme which has taken advantage of nature’s cyclical design is ‘the Able Project’ based in Wakefield, Yorkshire. Here, everything is recycled and reintegrated into the system. Consequently the project is able to turn cardboard into caviar. The cardboard is used as animal bedding, and is then composted using vermiculture. The worms are then used to feed fish. Excrement from the fish is collected and recycled into the system. The age of Biometry is still in its infancy. The novel and yet proven ideas which are inspired by nature are attractive to industries. But we need to be careful. As we lose more animal habitats to development, extinction rates are increasing. And with this we lose the wisdom and ideas embodied in these species. Already, we’ve seen the loss of the Gastric Brooding Frog, an animal able to turn off its production of stomach acid. Had we been able to study this animal before its demise, we could have found a way to alleviate the problems experienced by many sufferers of excess stomach acid. ...............THE END................

Page 5 of 6


D

ẠY

M

Q

U

Y

N

H

Ơ

N

O

FF IC IA L

Người ra đề: Nguyễn Thị Hòa Số điện thoại: 0972408833

Page 6 of 6


ĐỀ THI MÔN ANH KHỐI 11 NĂM HỌC 2015-2016 Thời gian làm bài 180 phút Đề thi gồm 11 trang

HỘI CÁC TRƯỜNG THPT CHUYÊN VÙNG DUYÊN HẢI VÀ ĐỒNG BẰNG BẮC BỘ TRƯỜNG THPT CHUYÊN BIÊN HÒA TỈNH HÀ NAM ĐỀ THI ĐỀ XUẤT

SECTION 1: LISTENING

FF IC IA L

You will hear the recordings TWICE.

M

Q

U

Y

N

H

Ơ

N

O

Part 1: Listen carefully to the short conversation and then choose the best answer to the question. (10 points) 1. What does the man want to do? A. Find work on campus. B. Work in the employment office. C. Help students find jobs. D. Ask the woman questions. 2. Where does the conversation probably take place? A. In the library. B. In a classroom. C. In a campus office. D. In an apartment. 3. How many hours of work does the man want per week? A. No more than ten. B. At least twenty. C. No more than twenty. D. Up to ten. 4. When can the man work? A. Every morning B. Afternoons and weekends C. When he’s in class. D. Weekdays 5. What does the woman tell the man to do tomorrow? A. Fill out a form. B. Give her some additional information. C. Tell her some news. D. Phone her.

ẠY

1.

Your answers:

2.

3.

4.

5.

D

Part 2: Listen and decide whether the statements are true (T) or false (F). (20 points)

What's in a name?

1. The US starts trends in names that Britain sometimes follows. 2. Names that seem to come from nature could come from surnames. 3. 'Ridge' is a name that is inspired by nature. 4. 'Lyric' is a relatively popular name. 5. All the different months are popular as names. 6. 'Blue' is a popular name for boys. 7. 'Apple' is probably becoming popular because of the company of that name.

1


8. People soon accept names that seem strange at first. 9. The Beckham's daughter was named after a famous writer. 10. 'Rowling' is a popular name because of the Harry Potter writer. Your answers: 1.

2.

3.

4.

5.

6.

7.

8.

9.

10.

FF IC IA L

Part 3: Listen and fill in the missing information. Write ONE WORD ONLY for each answer. (20 points)

Geography

Studying geography helps us to understand: the effects of different processes on the (1) _________ of the Earth the dynamic between (2) _________ and population

N

O

Two main branches of study: physical features human lifestyles and their (3) _________

Ơ

Specific study areas: biophysical, topographic, political, social, economic, historical and (4) _________ geography, and also cartography

H

Key point: geography helps us to understand our surroundings and the associated (5) _________

M

Q

U

Y

N

What do geographers do? find data – e.g. conduct censuses, collect information in the form of (6) _________ using computer and satellite technology analyze data – identify (7) _________ , e.g. cause and effect publish finding in form of: a) maps - easy to carry - can show physical features of large and small areas - BUT a two-dimensional map will always have some (8) _________ b) aerial photos - can show vegetation problems, (9) _________ density, ocean floor etc. c) Landsat pictures sent to receiving stations - used for monitoring (10) _________ conditions etc. Your answers:

D

ẠY

1. 2. 3. 4. 5.

6. 7. 8. 9. 10.

2


Section 2: Lexico

Grammar

D

ẠY

M

Q

U

Y

N

H

Ơ

N

O

FF IC IA L

Part 1. Choose the best answer to complete the following sentences. (10 points) 1. I woke up late for my interview because I ____ about it all night and didn’t get much sleep. A. worried B. have been worrying C. had been worrying D. had worried 2. It might sound strange, but dinner yesterday was the first time I ____ octopus. A. was eating B. have eaten C. had been eating D. had eaten 3. Make sure you ____ up the data on your computer, because you might get a virus. A. back B. copy C. store D. save 4. She ____ a name for herself in politics by running for mayor. A. gave B. made C. did D. wrote 5. I thought his business would fail, but he is really making a go of it. The underlined phrase is closest in meaning to which of the following? A. proving himself B. having success with C. trying to overcome D. standing a chance of 6. The Man-eating Tiger of Borneo, as its name ____, killed and ate a number of people. A. calls B. rings the bell C. suggests D. shows 7. Many local authorities realise there is a need to make ____ for disabled people in their housing programmes. A. assistance B. conditions C. admittance D. provision 8. Our salesmen normally ____ their travel expenses from the company once a month. A. settle B. reimburse C. cover D. claim 9. These suggestions are ____ to be accepted by the majority of members. A. unlikely B. impossible C. undoubtedly D. inconceivable 10. I’m not keen on ____ control of the project to a relative newcomer. A. undertaking B. charging C. entrusting D. allotting 11. ____ September 1st, ticket prices for the museum will increase to $6. A. As for B. As of C. As shown D. As well 12. I’m afraid we haven’t got a spare bed. Can you ______ with a mattress on the floor? A. make do B. make by C. make over D. make up 13. ______ receipt of your instructions, I immediately sent a telex message to Algeria. A. On B. In C. With D. By 14. I’m sure that never happened – it’s just a ______ of your imagination. A. fantasy B. figment C. piece D. picture 15. He was unable to keep up the pace ______ by the first three runners. A. set B. staged C. created D. led 16. At first the children enjoyed the game but quite soon the novelty ______. A. went off B. died out C. died down D. wore off 17. The desk was so ______ with papers that it was hard to find anything. A. burdened B. cluttered C. overrun D. muddled 18. I have several problems at the moment, ______ the least of which is lack of money. A. but B. not C. only D. far 19. He phoned to tell me that he couldn’t come tomorrow because he ______ to the dentist. A. had gone B. was going C. would go D. went 20. There has been a great deal of ______ in the press about the results of the murder trial. A. speculation B. prediction C. contemplatior D. sensation Part 2. Underline and correct the five mistakes in the following text. (5 points) Some people like to keep spiders as pets, particularly tarantulas, which are native in North America and can live for up to twenty-five years. Most people, on the other hand, do not like touching spider, and a significant number of people are afraid of them, mainly because of their poison. Moreover, despite their bad reputation, only thirty of the 37,000 known species of spiders are deathlike to humans. Spiders actually provide benefits to humans, from catching and eating harmful insects such as flies and mosquitoes. Part 3. Fill in each blank with a suitable preposition. (5 points) 1. Johnson was ____ the lead until the final bend, where he fell off his bike. 2. Let’s break ____ from the main crowd and go over there for a while? 3. Let’s kick ____ this session by introducing ourselves, shall we?

3


SECTION 3: READING

FF IC IA L

4. That’s enough TV! It’s time to knuckle ____ and get on with your homework now. 5. Something’s cropped ____, so I’m afraid I won’t be able to make it this afternoon. Part 4. Give the correct form of the word in the brackets. (10 points) 1. I hope I (last) his previous secretary – she only worked here for a week! 2. You have to be prepared for every (event) in this line of work. 3. The building looks a bit (future) from the outside but it’s quite traditional inside. 4. I sat completely (motion) as the spider crawled along my arm. 5. The heavy snow meant that the mountain roads were (pass) for over a week. 6. She was charged with being disorderly and (intoxicant). 7. I was unable to follow the (intricate) of the plot. 8. They exchanged (pleasant) for a few minutes before saying goodbye. 9. There is a decline in the (prevail) of cigarette smoking among young men. 10. They are planning for an advertising campaign to (public) the new film.

O

Part 1. Read the text below and decide which answer (A, B, C or D) best fits each gap. (15 points) We really can tell if we are being watched

M

Q

U

Y

N

H

Ơ

N

Stories about how people somehow know when they are being watched have been going around for years. However, few attempts have been made to investigate the phenomenon scientifically. Now, with the completion of the largest ever study of the so-called staring effect, there is impressive evidence that this is a recognisable and (1) ______ sixth sense. The study (2) ______ hundreds of children. For the experiments, they sat with their eyes (3) ______ so they could not see, and with their backs to other children, who were told to either stare at them or look away. Time and time again the results showed that the children who could not see were able to (4) ______ when they were being stared at. In a (5) ______ of more than 18,000 trials (6) ______ worldwide, the children (7) ______ sensed when they were being watched almost 70% of the time. The experiment was repeated with the (8) ______ precaution of putting the children who were being watched outside the room, (9) ______ from the starers by the windows. This was done just in case there was some (10) ______ going on with the children telling each other whether they were looking or not. This (11) ______ the possibility of sounds being (12) ______ between the children. The results, though less impressive, were more or less the same. Dr Sheldrake, the biologist who designed the study, believes that the results are (13) ______ enough to find out through further experiments (14) ______ how the staring effect might actually (15) ______ . B accepted B contained B wrapped B notice B collection B worked through B exactly B added B parted B lying B omitted B transported B convincing B carefully B be looked at

D

ẠY

1 A genuine 2 A involved 3 A shaded 4 A find 5 A sum 6 A worked over 7 A correctly 8 A attached 9 A separated 10 A pretending 11 A prevented 12 A delivered 13 A satisfying 14 A really 15 A come about

C received C comprised C masked C tell C mass C carried on C thoroughly C connected C split C cheating C evaded C transmitted C concluding C definitely C set out

D sure D enclosed D covered D reveal D total D carried out D perfectly D increased D divided D deceiving D ended D distributed D persuading D precisely D be held up

4


1.

Your answers: 2. 3. 4.

5.

6.

7.

8.

9.

10.

11.

12.

13.

14.

15.

Part 2. Read the text below and think of the word which best fits each gap. Use only one word in each gap. (15 points)

FF IC IA L

DANGERS OF TECHNOLOGY

H

Ć

N

O

Much has been heard recently about possible health hazards, including memory loss and brain tumours, from the use of mobile phones. With the possible half a billion mobile phones in (1) _______ throughout the world, in Britain today, one person in four owns one, (2) _______ is worrying enough, even if, so far, no concrete evidence has come to (3) _______ . One study by Dr. Alan Preece and his team at Bristol University has shown, however, in a report in the International Journal of Radiation Biology, that tests on volunteers demonstrated no effect on their short-term memory or attention span. Subjects (4) _______ exposed to microwave radiation for up to thirty minutes, but the one noticeable effect was positive (5) _______ than negative; the subject reacted more rapidly in one test (6) _______ a visual choice. One explanation of (7) _______ is that following the transmissions, a warming of the blood led to increased blood-flow. For the experiment, places were chosen where the signal was good and the microwave dose light, and then where the signal was poor and the dose much higher. The subjects were tested for recall and mental alertness (8) _______ exposure to microwaves characteristic of analogue phones, digital phones or no phones at all, without knowing (9) _______ they were exposed to. It is, of course, early days yet and the sample may not be large (10) _______ to generalize from. More research needs to be done.

2.

4.

5.

7.

9.

6.

8.

10.

Y

3.

Q

U

1.

N

Your answers:

M

Part 3. Read the following passage and mark the letter A, B, C or D on your answer sheet to indicate the correct answer to each of the questions. (15 points)

D

áş Y

KĂˆ

GENETICS In the 1860s, an Austrian botanist and monk named Gregor Mendel began studying the characteristics of pea plants. Specifically, he was interested in the way in which pea plants passed on their characteristics to their offspring. Mendel chose to work with pea plants because they are not selfpollinating. Unlike some plants, pea plants are distinctly male or female, and require the presence of a pea plant of the opposite sex for pollination. In this way, they are roughly analogous to humans and all other mammals, and it is for this reason that Mendel chose to study them. In his experiments, Mendel selected seven distinct traits in pea plants: such as plants producing round seeds versus those producing wrinkled seeds, or tall plants versus short plants. Mendel then spent years breeding plants with different combinations of traits and observing the results. What he concluded was that each trait is controlled by a gene which is passed down by parents. For example, there is gene for pea plants with round seeds and one for plants with wrinkled seeds. Mendel also concluded that a new pea plant must inherit a full set of genes from each of its parents. In cases, where a plant inherited the gene for round seeds from one parent and the gene for wrinkled seeds from the other, the new plants would have round seeds. This led Mendel to conclude that some genes are dominant and others are recessive. Characteristics which are controlled by recessive genes, like

5


H

Ơ

N

O

FF IC IA L

wrinkle seeds in pea plants, only surface if an organism inherits the recessive gene from both of its parents. Although it was greatly expanded upon in the 20th century, Mendel’s basis theory has stood up to more than one hundred years of scientific scrutiny, and a whole field of scientific study, genetics, has arisen around it. It is now known that Mendel’s genes are actually long strands of a complex. Molecule called DNA. Each gene carries instructions for the production of a certain protein. , and it is these proteins which determine the traits of an organism. We also know that genes are transmitted in structures called chromosomes, long chains of genes. Humans have 46 chromosomes, receiving 23 from their mother and 23 from their father. Actually each set of 23 is basically a complete genetic package, but since some genes are dominant and some are recessive, the redundancy events out. Mendel’s observations led him to a simple and elegant theory heredity, but while the basis of his theory will stand, reality has not proven to be quite as simple as theory. Any living organism has thousands of genes. For example, fruit flies have about 13,000 sets of genes, and humans have somewhere between 20,000 and 30,000 adding to the complexity implied by the sheer numbers of genes is the fact that many traits are polygenic; that is, they are controlled by a combination of tens or even hundreds of genes, rather than by a single gene as Mendel had envisioned. So while his experiments produced black and white results (a pea plant had either round or wrinkled seeds), the interactions of genes in determining traits are often not so straightforward, and there may be hundreds or thousands of possible outcomes. Genetics has had a huge impact on the first years of the 21st century. While earlier scientists were largely limited to investigating the genes of organisms and classifying which genes controlled which traits, recent advances in chemistry and molecular biology have actually allowed scientists to begin to alter those genes. The implications of this development are nearly infinite. While still in its infancy, this new science, called genetic engineering, has allowed scientists to change organisms in fundamental ways. Scientists can now deactivate harmful genes, promote the function of useful genes, or introduce foreign genes into an organism to produce an entirely new trait.

D

ẠY

M

Q

U

Y

N

1. According to paragraph 1, Mendel’s reason for choosing pea plants for his experiments was that A. they were easier to breed than other types of plants B. their method of reproduction was similar to that of mammals C. they passed interesting characteristics to their offspring D. he was interested in studying why some plants are self-pollinating 2. The phrase “the other” in the passage refers to A. trait B. seed C. gene set D. parent 3. According to the information in paragraph 2, what led Mendel to conclude that some genes were recessive? A. In some cases, pea plants completely failed to inherit characteristics from their parents. B. Some of his pea plants produced seeds that were progressively more and more wrinkled. C. Some characteristics only seemed to surface if both parents had that characteristic. D. In some cases, his pea plants did not seem to inherit a full set of genes from each parent. 4. The word “scrutiny” in the passage is closet in meaning to A. investigation B. opposition C. application D. theory 5. All of the following are mentioned in the passage as supplements to Mendel’s original theory EXCEPT A. an explanation of how some genes dominate others B. the chemical description of genes C. the counting of genes and gene grouping in organisms D. the manipulation of genes to produce specific traits 6. According to paragraph 3, what is ultimately responsible for the production of specific traits in an organism? A. The replication of chromosomes B. The production of proteins within the organism C. The use of proteins to create DNA in the organism D. The structural complexity of the DNA molecule 7. According to paragraph 4, what fact complicates Mendel’s theory?

6


O

FF IC IA L

A. The fact that many traits are controlled by several genes B. The fact that the exact numbers of genes for organism are uncertain C. The fact that organisms can have very different numbers of genes D. The fact that Mendel had only thought in black and white terms 8. Based on the information in paragraph 4, what can be inferred about the genetic make up of organisms? A. Humans have the highest number of genes that are polygenic. B. Their traits are actually impossible to predict. C. Only organisms that lack polygenetic traits are properly understood. D. More advanced organisms generally have higher numbers of gene sets. 9. The word “envisioned” in the passage is closest in meaning to A. imagined B. required C. represented D. tested 10. Which of the following is not true? A. According to Mendel, traits were passed down through genes, which could either be recessive or dominant. B. Mendel’s basic theory has proved to be very complicated. C. Humans have 46 long chains of genes. D. Thanks to genetics engineering, scientists can now make fundamental changes to organisms. Your answers: 2.

3.

4.

5.

6.

8.

9.

10.

Ơ

Part 4. Read the passage.

7.

N

1.

U

Y

N

H

Tyes and Greens There are a number of settlements in this part of East Anglia with names containing the word “tye”. The word is Anglo-Saxon in origin, and the Oxford English Dictionary quotes the earliest usage of the term as dating from 832. Essentially a “tye” was a green, or a small area of open common land, usually sited away from the main village or settlement, perhaps at the junction of two or more routes. Local people and passing travellers had the right to pasture their horses, pigs and other farm animals on the tye.

M

Q

In the Pebmarch area there seem to have been five or six of these tyes, all except one, at the margins of the parish. These marginal clearings are all away from the richer farming land close to the river, and, in the case of Cooks Green, Hayles Tye, and Dorking Tye, close to the edge of still existing fragments of ancient woodland. It seems likely then that, here, as elsewhere in East Anglia, medieval freemen were allowed to clear a small part of the forest and create a smallholding. Such unproductive forest land would, in any case, have been unattractive to the wealthy baronial or monastic landowners. Most of the land around Pebmarch village belonged to Earls Colne Priory, a wealthy monastery about 10 kilometres to the south, and it may be that by the 13th and 14th centuries the tyes were maintained by tenant farmers paying rent to the Priory.

D

ẠY

Hayles Tye seems to have got its name from a certain John Hayle who is documented in the 1380s, although there are records pointing to occupation of the site at a much earlier date. The name was still in use in 1500, and crops up again throughout the 16th and 17th centuries, usually in relation to the payment of texas or tithes. At some point during the 18th century the name is changed to File’s Green, though no trace of an owner called File has been found. Also in the 18th century the original dwellings on the site disappeared. Much of this region was economically depressed during this period and the land and its dwellings may simply have been abandoned. Several farms were abandoned in the neighboring village of Alphamstone, and the population dwindled so much that there was no money to support the fabric of the village church, which became very dilapidated. However, another possibility is that the buildings at File’s Green burnt down, fires being not infrequent at this time.

7


FF IC IA L

By 1817 the land was in the ownership of Charles Townsend of Ferriers Farm, and in 1821 he built two brick cottages on the site, each cottages occupied by two families of agricultural laborers. The structure of these cottages was very simple, just a two-storey rectangle divided in the centre by a large common chimney piece. Each dwelling had its own fireplace, but the two families seem to have shared a brick bread-oven which jutted out from the rear of the cottage. The outer wall of the bread-oven is still visible on the remaining cottage. The fireplaces themselves and the chimney structure appear to be older than the 1812 cottages and may have survived from the earlier dwellings. All traces of the common land had long disappeared, and the two cottages stood on a small plot of less than an acre where the laborers would have been able to grow a few vegetables and keep a few chickens or a pig. The bulk of their time was spent working at Ferriers farm.

Both cottages are clearly marked on maps of 1874, but by the end of the century one of them had gone. Again, the last years of the 19th century were a period of agricultural depression, and a number of smaller farms in the area were abandoned. Traces of one, Mosse’s Farm, still partly encircle by a very overgrown moat, may be seen less than a kilometer from File’s Green. It seems likely that, as the need for agricultural labor declined, one of the cottages fell into disuse, decayed and was eventually pulled down. Occasional fragments of rubble and brick still surface in the garden of the remaining cottage.

N

O

In 1933, this cottage was sold to the manager of the newly-opened gravel works to the north-west of Pebmarch village. He converted these dwelling into one. This, then, is the only remaining habitation on the site, and is called File’s Green Cottage.

John Hayle, who is _______(1)_______, apparently gave his name to Hayles Tye.

1500s

the name of Hayles Tye was still in use, ______(2)______ again in the following two centuries in relation to taxes.

18th century

Hayles Tye was renamed _____(3)______; the original dwellings may either have disappeared, or were _____(4)_____ .

1817

the land was ____(5)_____ by Charles Townsend.

1821

Q

U

Y

N

H

1380s

M

Ơ

Complete the text below, which is a summary of paragraph 3-6. Use NO MORE THAN THREE WORDS from the passage to fill each blank space.

Charles Townsend built _____(6)_____ cottages on the site, _____(7)____ inhabited by two families, but by the end of the nineteenth century only one cottage _____(8)______ .

1933

D

ẠY

The cottage, now called File’s Green Cottage, was bought by the local _____(9)_____ manager who converted the cottage into _____(10)_____ .

1. 2. 3. 4. 5.

Your answers: 6. 7. 8. 9. 10.

Section 4: Writing 8


FF IC IA L

Part 1. Transform or rewrite the following sentences. (5 points) 1. At the moment people think the accident is Nick’s fault. → Nick is currently ………………………………………………………………………………… . 2. “I didn’t harm anybody!” cried the accused. → The accused denied ……………………………………………………………………………… . 3. Your scheme is brilliant, but it won’t succeed. (doomed) → Brilliant though …………………………………………………………………………… failure. 4. There was no further delay in awarding the prizes. (ado) → The prizes ………………………………………………………………………………………... . 5. I was in the middle of my speech when Pipa interrupted me. (cut) → Pipa ………………………………………………………………………….. through my speech. Part 2. (20 points)

The pie charts below show average household expenditure in Hong Kong and Britain in the year 2000. Write a report of about 150 words for a university lecturer describing the information below.

Household expenditure in hong Kong in 2000

O

Household expenditure in Britain in 2000

N

Clothing 7% Housing 18%

Ơ

Other goods and services 28%

Housing 32%

N

H

Other goods and services 36%

Clothing 4%

Food 27%

Transport 9%

Q

U

Food 22%

Y

Transport 17%

……………………………………………………………………………………………………………

M

…………………………………………………………………………………………………………… ……………………………………………………………………………………………………………

…………………………………………………………………………………………………………… ……………………………………………………………………………………………………………

ẠY

…………………………………………………………………………………………………………… …………………………………………………………………………………………………………… ……………………………………………………………………………………………………………

D

…………………………………………………………………………………………………………… …………………………………………………………………………………………………………… …………………………………………………………………………………………………………… …………………………………………………………………………………………………………… …………………………………………………………………………………………………………… ……………………………………………………………………………………………………………

9


…………………………………………………………………………………………………………… …………………………………………………………………………………………………………… …………………………………………………………………………………………………………… ……………………………………………………………………………………………………………. ..................................................................................................................................................................... .....................................................................................................................................................................

FF IC IA L

..................................................................................................................................................................... Part 3. (35 points) Write a paragraph of about 250 – 300 words on the following topic:

O

The private motor vehicle has greatly improved individual freedom of movement. Moreoer, the automobile has become a status symbol. Yet the use of private motor vehicles has contributed to some of today’s most serious problems. Do you agree or disagree with these statements? Give reasons for your answer and include any relevant examples. ……………………………………………………………………………………………………………

N

……………………………………………………………………………………………………………

Ơ

…………………………………………………………………………………………………………… ……………………………………………………………………………………………………………

H

……………………………………………………………………………………………………………

N

…………………………………………………………………………………………………………… ……………………………………………………………………………………………………………

Y

……………………………………………………………………………………………………………

U

……………………………………………………………………………………………………………

Q

…………………………………………………………………………………………………………… ……………………………………………………………………………………………………………

M

……………………………………………………………………………………………………………

…………………………………………………………………………………………………………… …………………………………………………………………………………………………………… ……………………………………………………………………………………………………………

ẠY

…………………………………………………………………………………………………………… …………………………………………………………………………………………………………… ……………………………………………………………………………………………………………

D

…………………………………………………………………………………………………………… …………………………………………………………………………………………………………… …………………………………………………………………………………………………………… ……………………………………………………………………………………………………………. ……………………………………………………………………………………………………………. ……………………………………………………………………………………………………………

10


…………………………………………………………………………………………………………… …………………………………………………………………………………………………………… …………………………………………………………………………………………………………… …………………………………………………………………………………………………………… …………………………………………………………………………………………………………… ……………………………………………………………………………………………………………

FF IC IA L

…………………………………………………………………………………………………………… ……………………………………………………………………………………………………………

…………………………………………………………………………………………………………… ……………………………………………………………………………………………………………

D

ẠY

M

Q

U

Y

N

H

Ơ

N

O

…………………………………………………………………………………………………………….

--- The end ---

11


Answer key SECTION 1 – LISTENING Part 1: (2 x 5 = 10 points) 1. A

2. C

3. C

4. B

5. D

1. T

2. T

3. T

4. T

5. F

6. F

7. T

Part 3: (2 x 10 = 20 points) 6. images 7. patterns 8. distortion(s) 9. traffic 10. weather

9. T

10. F

N

Tape scripts

8. T

O

1. surface 2. environment 3. impact(s)/ effect(s) 4. urban 5. problems

FF IC IA L

Part 2: (2 x 10 = 20 points)

Part 1:

M

Q

U

Y

N

H

Ơ

(man) I’m looking for a part-time job on campus. (woman) Then you’ve come to the right place. The campus employment office is here just to help students like you find jobs on campus. (man) I’m glad to hear that because I really need to start earning some money. (woman) Let me ask you some question to help determine what kind of job would be best. First of all, how many hours a week do you want to work? (man) I need to work at least ten hours a week, and I don’t think , I can handle more than twenty hours with all the courses I’m taking. (woman) And when are you free to work? (man) All of my classes are in the morning , so I can work every weekday from noon on. And of course I wouldn’t mind working on the weekends. (woman) I’ll try to match you up with one of our on-campus student jobs. Please fill out this form with some additional information about your skills, and leave the form with me today. Then you can call me back tomorrow, and maybe I’ll have some news for you. (man) Thanks for your help.

Part 2:

D

ẠY

Presenter 1: Today we are going to talk about names, particularly fashions in names, you know, the kind of names famous people use for their children. You’ve been looking into this recently, haven’t you Finn? Presenter 2: I have indeed and it’s a fascinating topic. The US leads here with new names and we in Britain follow sometimes, but we tend to go for the more traditional names. So, the big trend is using nouns as names. Presenter 1: Nouns, what sort of nouns? Presenter 2: Well they can be abstract qualities like Honor or Passion. There’s a long tradition of this kind of name, like Faith or Charity, which used to be common names. A new name is Haven, that’s growing in popularity. And similar names like Shelter, Harbor and Bay also convey feelings of safety and warmth. Presenter 1: Mmm. I suppose Passion is used to mean ‘extreme enthusiasm’ nowadays, and people use the word a lot, so maybe it’s a good choice for a modern name. Haven has a nice, safe feel to it. Presenter 2: OK, then there are names which come from nature or animals, although with some of

12


ẠY

M

Q

U

Y

N

H

Ơ

N

O

FF IC IA L

these it’s hard to know whether they come from nature or a surname – that’s another trend. Here we have Frost, Wolf, Fox, Bear, for boys, of course. And a new name: Ridge. Presenter 1: Ridge, like a mountain ridge? The top of a mountain range? Presenter 2: Yes, weird, huh? It’s seen as a tough, outdoorsy name for a boy. OK, then there are musical names. Harmony and Melody have been around for ages, but Lyric is a new one. Presenter 1: Lyric, wow! Presenter 2: Yes, it came in at number 325 in the US a couple of years ago. That doesn’t sound very popular, but there are so many different names being used at the moment that it means it isn’t so unusual. Other noun categories are months – May, June and April are common, but January is uncommon and November very unusual. And then you have colours. Beyoncé and Jay-Z called their daughter Blue Ivy – a very distinctive name, a colour plus the name of a plant. Blue is very popular for girls right now, and Red or Grey for boys. Presenter 1: I’ve just thought of another category. Food names, like Olive or Clementine. Presenter 2: Yes, that’s another one. Flower names are pretty common, but food names are unusual. Gwyneth Paltrow and Chris Martin named their daughter Apple, of course. Presenter 1: Yes, poor child. Presenter 2: Actually, Apple is becoming more and more popular, although people think that’s because of the technology connection, not the fruit. New names always seem strange at first, but you quickly get used to them, like all the names from places or jobs. Chelsea and Brooklyn seem like normal names now; they were strange when they were first used. Taylor, Mason, Cooper are all first names from jobs. Presenter 1: And they are also surnames, I think that’s how they started. Presenter 2: You’re probably right there. Presenter 1: What about the Beckhams’ daughter, Harper? Presenter 2: She was named after Harper Lee, the American novelist who wrote To Kill a Mockingbird. That’s another trend, naming children after famous writers, musicians – the British band One Direction have had an effect on names – or fictional characters, like Bella or Edward from the Twilight series, or Hermione from Harry Potter. Presenter 1: Sorry, can I just interrupt there. I’ve just has a message passed on by the producer. A listener has just phoned in with a very strange story about a family in Holland with six children. Their names are all anagrams of the letters: A, E, L and X. Presenter 2: Ah yes, I’ve heard about this. This family are famous in the world of bloggers on names. Let me see if I remember the names … Alex and Axel … and Lexa – they’re the easy ones – Xela ('Zela') and Xael ('Zay-el') and the last one is Xeal ('Zeal') – I’m guessing about the pronunciations, by the way. Presenter 1: You mean there are names X-E-L-A and X-A-E-L? Presenter 2: Yes, but they are very unusual. I think the Dutch family are stopping at six children, but there are about eighteen more possible anagrams they could use. Presenter 1: Nooo, you’re kidding! Presenter 2: All seem horrible to me, but all are possible names. This is similar to another trend of giving children names all starting with the same letter, like the Kardashian family, all beginning with K, Kim, Kourtney and so on. Presenter 1: The Kardashians have had enough publicity, let’s not talk about them. What about your name? Finn, that sounds like a good Irish name …

D

Part 3:

13


14

ẠY

D KÈ M Y

U

Q N

Ơ

H

N

FF IC IA L

O


FF IC IA L O Grammar

N

Section 2: Lexico

12A

13A

14B

15A

16D

17B

18B

H

11B

Ơ

Part 1. Choose the best answer to complete the following sentences. (0.5 x 20 = 10 points) 1C 2D 3A 4B 5B 6C 7D 8D 9A 10C 19B

20A

ẠY

M

Q

U

Y

N

Part 2. Underline and correct the five mistakes in the following text. (1.0 x 5 = 5 points) Some people like to keep spiders as pets, particularly tarantulas, which are native in North America 1 and can live for up to twenty-five years. Most people, on the other hand, do not like touching spider, 2 and a significant number of people are afraid of them, mainly because of their poison. Moreover, 3 despite their bad reputation, only thirty of the 37,000 known species of spiders are deathlike to 4 humans. Spiders actually provide benefits to humans, from catching and eating harmful insects such as 5 flies and mosquitoes. 1. in → to 2. spider → spiders 3. Moreover → However 4. deathlike → deadly 5. from → by Part 3. Fill in each blank with a suitable preposition. (1.0 x 5 = 5 points) 1. in 2. away 3. off 4. down 5. up

D

Part 4. Give the correct form of the word in the brackets. (1.0 x 10 = 10 points) 1. outlast 2. eventuality 3. futuristic 4. motionless 5. impassable 6. intoxicated 7. intricacies 8. pleasantries 9. prevalence 10. publicize

15


SECTION 3: READING Part 1: (1 x 15 = 15 points) 1. A

2. A

3. D

4. C

5. D

6. D

7. A

8. B

9. A

10.C 11.A 12.C 13.B 14.D 15.A

Part 2: (1.5 x 10 = 15 points)

3. light

5. rather

7. this

2. which

4. were

6. with

8. after

Part 3: (1.5 x 10 = 15 points) 1. B

2. D

3. C

4. A

5. A

6. B

7. A

10. enough

8. D

9. A

10. B

O

Part 4: (1.5 x 10 = 15 points)

9. what

FF IC IA L

1. use

6. two/ two brick 7. each one/ each/ each cottage

N

1. documented 2. copping up/ and crops up/ and cropped up 3. File’s Green 4. burnt down/ abandoned 5. owned

H

Ơ

8. remained/ survived 9. gravel works 10. one dwelling

N

Section 4: Writing

Q

U

Y

Part 1. Transform or rewrite the following sentences. (1.0 x 5 = 5 points) 1. Nick is currently being blamed for the accident. 2. The accused denied causing / having caused harm to anybody. 3. Brilliant as / though your scheme is, it is doomed to failure. 4. The prizes were awarded without more ado. 5. Pipa cut me short / cut in halfway through my speech.

D

ẠY

M

Part 2 & Part 3. (20 points & 35 points) For Part 2 and Part 3, the student’s task is assessed according to the general criteria of writing skills.

--- The end --16


Trường THPT Chuyên Nguyễn Trãi Tỉnh Hải Dương ĐỀ THI MÔN TIẾNG ANH KHỐI 11 (2016) Part I: Listening I. Questions 1-6 Complete the table using NO MORE THAN THREE WORDS AND/OR A NUMBER for each answer.

Emu Park

Price Example Bunkhouse $ 5.90 / night Cabins at $ 1.00 / night or 1.................................. with air conditioning Weekly cost to share room 4. .................................

Hostel

Facilities

Extras

FF IC IA L

Hostel East Coast Backpackers

5 minutes to beach 2. ................................ Rooms overlooking beach have 5. ............................

3. ................................... package Good for

Ơ

N

O

6. ................................ Questions 7-10 Complete the notes using NO MORE THAN THREE WORDS for each answer. East Coast Backpackers' Hostel address: 7. ......................................................................... Road. Bus will have words 8. ................................................................... written on the front. Computer access costs 9 .......................................................................... . Shop stocks things like 10. ..................................... and .............................................

M

Q

U

Y

N

H

II. You will hear people talking in 5 situations. For questions 1-5, choose the best answer, A, B or C 1. you hear a man talking about a football match. Why was the match cancelled? A. because of the bad weather B. because of football hooligans B. C. because of an accident 2. you hear someone talking on a mobile phone. Who is she talking to? A. a customer B. a colleague C. a relative 3. You overhear two people in a travel agent’s arguing about a trip. What do they disagree about? A. how warm it will be B. how sunny it will be C. how humid it will be 4. You hear a weather forecast on the radio. What will the weather be like tomorrow in the North? A. wet B. windy C. bright 5. Listen to this news report about a flood. What was the probable cause of the flood? A. a river bursting its banks B. a broken pipe C. heavy rainfall

D

ẠY

PART II. GRAMMAR-VOCABULARY I. Choose the best answer. 1. Everyone was having fun, but I was too …………………to join in, even though I wanted to. A. inhibited B. modest C. vain D. superior 2. According to the ................ of the contract, tenants must give six months’ notice if they intend to leave. A. laws B. rules C. terms D. details 3. You are under no obligation to help as assistance is purely …………… A. voluntary B. free C. charitable D. donated 4. I’m afraid you may find the truth somewhat ………………………… A. inedible B. unpalatable C. indigestible D. unmanageable 5. She’s very ………… She can be relied on to do her job properly. A. efficient B. cautious C. serious D. conservative 6. The youth team really ………. themselves in the semi-final. A. surmounted B. exceeded C. beat D. excelled 7. In my opinion, it’s only common …………… to wear a seat belt in a car. A. judgement B. sense C. intelligence D. wit 8. How do you ……………… about the pollution problem in this country?


Ơ

N

O

FF IC IA L

A. feel B. think C. believe D. view 9. I’m ……………. in my present job; I need something more challenging. A. irritated B. annoyed C. crazy D. frustrated 10. I find his behaviour towards me quite ………….. He treats me like an idiot. A. offending B. insulting C. crude D. ill-mannered 11. Several of my friends are …………… reporters. A. newspaper B. newspapers C. newspaper’s D. newspapers’ 12. Why did Mary ask you …………. a bicycle? A. that if you had B. do you have C. that you had D. if you had 13. Our classroom is supplied with ………….. A. an heavy equipment B. a heavy equipment C. heavy equipments D. heavy equipment 14. There are several means of mass communication. The newspaper is one, television is ..............… A. other B. the other C. another D. others 15. Her guest apologized for causing her so much …………… A. problem B. complication C. trouble D. damage 16. This cloth ……… very thin. A. feels B. touches C. holds D. handles 17. Bread and butter ………….. his favorite breakfast. A. is B. are C. make D. making 18. As Roger had resigned, the company decided to take no further ………….against him. A. action B. demand C. activity D. conduct 19. Neither Peter nor his parents …………….. going to spend the summer abroad. A. is B. are C. was D. has been 20. I only paid £3 for this dress. It was a real …………………… A. find B. bargain C. sale D. cheap

H

II. Complete the sentences with the correct form of the words in bold.

D

ẠY

M

Q

U

Y

N

A Mexican cookery course On holiday last year my two travelling companions and I joined a day’s cookery course in a Mexican restaurant. There were eight (1)………, all keen to learn the secrets of the nation’s cuisine. The students ranged from people who already had some (2) ………….…… in the kitchen, to totally (3) ………………..…… people like myself. Our teacher, Liana Cabrera, started with a short talk, the handed out some notes giving (4) ……….. of terms we would be coming across. Soon we were trying out a range of exotic ingredients, with (5) …….. good results. Cabrera started giving cookery lessons five years ago, and has become quite a (6) ……………, with long waiting lists for her courses. And because of her extensive knowledge of almost-forgotten regional dishes she is also a regular (7)……….. to cookery programmes on national television. In the afternoon I joined the salsa-making team, with rather (8) …… results. My colleagues complained that my food was so (9) …….. hot it made their eyes water. Their own efforts turned out (10) ……… better than mine. The communal meal at the end of the day was delicious, and I’d not only learnt something about cooking, but I’d also broadened my understanding of Mexican culture.

PARTICIPATE EXPERT SKILL EXPLAIN SURPRISE CELEBRATE CONTRIBUTE DISASTER PAIN CONSIDER

III. Complete each of the following sentences with a suitable particle. 1. A: “Isn’t your class picnic today?” B: “No, our plans for the picnic fell………………...” 2. Who cared ……………… your cat while you were out of town? 3. The store had to lay …………………. a number of clerks because sales were down. 4. A: “Did Amanda ever complete the project?” B: “She’s almost finished. She just has a few minor problems left to iron …………...”


5. Vaccines have permitted doctors to virtually stamp ……a number of diseases, including smallpox and polio. IV. From four underlined parts, choose the one that needs correction and then correct it.

FF IC IA L

1. Any property that a bankrupt person may still have is usually divided among the various A B C people to whom money are owned. D 2. The body does not stay at the alike temperature from morning till night. A B C D 3. A loudspeaker functions on the same principle that the telephone receiver does, but it is A B C

Q

U

Y

N

H

Ơ

N

O

more larger and louder. D 4. Thunder that is audible from distances as far away as ten miles. A B C D 5. Good dental hygiene and a proper diet are necessary for the maintain of sound teeth. A B C D PART III – READING I. Complete each blank with ONE suitable word. Traffic jams are nothing new In the age before the motor car, what was travelling in London like? Photographs taken 100 years ago showing packed streets indicate that it was much the (1) …….as it now. It has been calculated that, even with new anti-congestion systems in place, commuters who choose the car to get to work travel at an average speed of 17 kph from their homes in the suburbs to offices in the centre. (2)………is virtually the same speed that they (3)……….. have traveled at in a horse and carriage a century ago. As towns and cities grow, (4)……does traffic, whether in the form of the horse and carriage (5)……the modern motor car. It would seem that , wherever (6)…….are people who need to go somewhere, they would rather be carried than walk or pedal. The photographs show that, in terms of congestion and speed, traffic in London hasn’t changed over the past 100 years. London has had traffic jams ever (7)……..it became a huge city. It is only vehicles that have changed. However, apart (8)……..the congestion which affected London long (9)………the car came along, the age of the horse produced relatively (10)…….unpleasantness. This age, for example, saw none of the exhaust fumes which city dwellers have to live with today.

D

ẠY

M

II. Read the following passage and decide which option A, B, C or D best fits each sentence. Work on the human brain has indicated how different parts are centres of activity for different skills, feelings, perceptions and so on. It has also been shown that the left and right halves, or hemispheres, of the brain are (1) ……….... for different functions. While language is processed in the left, or analytical hemisphere, for most people music is processed in the right, or emotional hemisphere. However, professional musicians have the (2) ……... to process music in the left hemisphere more often than those without musical training do. This (3) …... they are having a different experience – which is likely to be the case because they are analysing music rather than just listening to it. (4) …... of music like tone, pitch and melody are all probably processed in different parts of the brain. Some features of musical experience are processed not just in the auditory parts of the brain, but in the visual ones. We don’t yet fully understand the (5) …….. of this. The tempo of music seems to be (6) ………... related to its emotional impact, with fast music often (7) ………... as happier and slower music as sadder. It is the same with the major biological rhythm of the body: our heart (8) ……….. quickens when we’re happy, but slows when we’re sad. Military music may have (9) ………... from attempts to get us ready for (10) ….. by using fast drumming to stimulate our hearts to beat faster. Music is perhaps one of the most complex experiences the brain copes with and it has become an absolutely vital part of our rituals and ceremonies. It has power beyond language to communicate mood and co-ordinate our emotional states. 1. A amenable 2. A tendency

B dependable B inclination

C responsible C possibility

D reliable D intention


3. A proposes 4. A Views 5. A expectations 6. A surely 7. A felt 8. A pulse 9. A evolved 10. A battle

B advances B Aspects B implications B plainly B endured B speed B extended B fight

C introduces C Factors C assumptions C evidently C encountered C pace C advanced C quarrel

D suggests D Pieces D propositions D directly D touched D rate D elevated D struggle

D

ẠY

M

Q

U

Y

N

H

Ơ

N

O

FF IC IA L

III. Read the following passage and choose the best answer to each question. PAINTERS OF TIME ‘The world’s fascination with the mystique of Australian Aboriginal art.’ Emmanuel de Roux A The works of Aboriginal artists are now much in demand throughout the world, and not just in Australia, where they are already fully recognised: the National Museum of Australia, which opened in Canberra in 2001, designated 40% of its exhibition space to works by Aborigines. In Europe their art is being exhibited at a museum in Lyon, France, while the future Quai Branly museum in Paris – which will be devoted to arts and civilisations of Africa, Asia, Oceania and the Americas – plans to commission frescoes by artists from Australia. B Their artistic movement began about 30 years ago, but its roots go back to time immemorial. All the works refer to the founding myth of the Aboriginal culture, ‘the Dreaming’. That internal geography, which is rendered with a brush and colours, is also the expression of the Aborigines’ long quest to regain the land which was stolen from them when Europeans arrived in the nineteenth century. ‘Painting is nothing without history,’ says one such artist, Michael Nelson Tjakamarra. C There are now fewer than 400,000 Aborigines living in Australia. They have been swamped by the country’s 17.5 million immigrants. These original ‘natives’ have been living in Australia for 50,000 years, but they were undoubtedly maltreated by the newcomers. Driven back to the most barren lands or crammed into slums on the outskirts of cities, the Aborigines were subjected to a policy of ‘assimilation’, which involved kidnapping children to make them better ‘integrated’ into European society, and herding the nomadic Aborigines by force into settled communities. D It was in one such community, Papunya, near Alice Springs, in the central desert, that Aboriginal painting first came into its own. In 1971, a white schoolteacher, Geoffrey Bardon, suggested to a group of Aborigines that they should decorate the school walls with ritual motifs, so as to pass on to the younger generation the myths that were starting to fade from their collective memory. He gave them brushes, colours and surfaces to paint on – cardboard and canvases. He was astounded by the result. But their art did not come like a bolt from the blue: for thousands of years Aborigines had been ‘painting’ on the ground using sands of different colours, and on rock faces. They had also been decorating their bodies for ceremonial purposes. So there existed a formal vocabulary. E This had already been noted by Europeans. In the early twentieth century, Aboriginal communities brought together by missionaries in northern Australia had been encouraged to reproduce on tree bark the motifs found on rock faces. Artists turned out a steady stream of works, supported by the churches, which helped to sell them to the public, and between 1950 and 1960 Aboriginal paintings began to reach overseas museums. Painting on bark persisted in the north, whereas the communities in the central desert increasingly used acrylic paint, and elsewhere in Western Australia women explored the possibilities of wax painting and dyeing processes, known as ‘batik’. F What Aborigines depict are always elements of the Dreaming, the collective history that each community is both part of and guardian of. The Dreaming is the story of their origins, of their ‘Great Ancestors’, who passed on their knowledge, their art and their skills (hunting, medicine, painting, music and dance) to man. ‘The Dreaming is not synonymous with the moment when the world was created,’ says Stephane Jacob, one of the organisers of the Lyon exhibition. ‘For Aborigines, that moment has never ceased to exist. It is perpetuated by the cycle of the seasons and the religious ceremonies which the Aborigines organise. Indeed the aim of those ceremonies is also to ensure the permanence of that golden age. The central function of Aboriginal painting, even in its contemporary manifestations, is to guarantee the survival of this world. The Dreaming is both past, present and future.’ G Each work is created individually, with a form peculiar to each artist, but it is created within and on behalf of a community who must approve it. An artist cannot use a ‘dream’ that does not belong to his or her


Ơ

N

O

FF IC IA L

community, since each community is the owner of its dreams, just as it is anchored to a territory marked out by its ancestors, so each painting can be interpreted as a kind of spiritual road map for that community. H ‘By exporting their paintings as though they were surfaces of their territory, by accompanying them to the temples of western art, the Aborigines have redrawn the map of their country, into whose depths they were exiled,’ says Yves Le Fur, of the Quai Branly museum. ‘Masterpieces have been created. Their undeniable power prompts a dialogue that has proved all too rare in the history of contacts between the two cultures’. Question 1-6: The passage has nine paragraphs, A-H. Choose the correct heading for paragraphs A-F from the list of headings, i-viii, below. List of Headings i Amazing results from a project ii New religious ceremonies iii Community art centres iv Early painting techniques and marketing systems v Mythology and history combined vi The increasing acclaim for Aboriginal art vii Belief on continuity viii Oppression of a minority people 1. Paragraph A __________ 2. Paragraph B __________ 3. Paragraph C __________ 4. Paragraph D __________ 5. Paragraph E __________ 6. Paragraph F __________

Q

U

Y

N

H

Question 7-10: Complete the flow chart below. Choose NO MORE THAN THREE WORDS from the passage for each answer. For (7) __________, Aborigines produced ground and rock paintings. Early twentieth century: churches first prompted the use of (8) __________ for paintings. Mid-twentieth century: Aboriginal paintings were seen in (9) __________. Early 1970s: Aborigines painted traditional patterns on (10) __________ in one community.

D

ẠY

M

IV. Read the following passage and choose the best answer to each question. Which picture do you have of the future? Will life in the future be better, worse or the same as now? What do you hope about the future? Futurologists predict that life will probably be very different in 2050 in all fields of activity, from entertainment to technology. First of all, it seems that T.V channels will have disappeared by 2050. Instead, people will choose a program from a “menu “and a computer will send the program directly to the television. Today, we can use the World Wide Web to read newspapers and see pictures on a computer thousands of kilometers away. By 2050 music, films, programs, newspaper and books will come to us by computer. Then “Holographic Feedback T .V” will have arrived: holograms are pictures that have height, width, and depth .Simple holograms exist today and “virtual reality “games are already popular. By 2050, we will be able to see, smell and touch the things that we see on television. In what concerns the environment? Water will have become one of our most serious problems. In many places, agriculture is changing and farmers are growing fruit and vegetables to export. This uses a lot of water. Demand for water will increase ten times between now and 2050 and there could be serious shortages. Some futurologists predict that water could be the cause of war if we don’t act now. In transport, cars will run on new, clean fuels and they will go very fast. Cars will have computers to control the speed of the car and there won’t be any accidents. Today, many cars have computers that tell drivers exactly where they are. By 2050, the computer will control the car and drive it to your destination. On the other hand, space planes will take people halfway around the world in two hours. Nowadays, the


FF IC IA L

United States Shuttle can go in to space and land on Earth again. By 2050, spaces planes will fly all over the world and people will fly from Los Angles to Tokyo in just two hours. In the domain of technology, robots will have replaced people in factories. Many factories already use robots. Big companies prefer robots – they do not ask for pay rises or go on strike, and they work 24 hours a day. By 2050, we will see robots everywhere – in factories, schools, offices, hospitals, shops, and homes. Last but not least, medicine technology will have conquered many diseases. Today, there are electronic devices that connect directly to the brain to help people hear. By 2050, we will be able to help blind and deaf people see again and hear again .Scientists have discovered how to control genes .They have already produced clones of animals. By 2050, scientists will be able to produce clones of people and decide how they look, how they behave and how much intelligence they have.

D

ẠY

M

Q

U

Y

N

H

Ơ

N

O

1. What can be inferred about the life in 2050 according to the passage? A. Life in 2050 will be much better. B. T. V will be an indispensable means of communication and business. C. People will not be threatened by the shortage of water due to the polar melting. D. The deaf will not have to depend any longer on the electronic hearing devices. 2. What can be inferred from the passage about T.V viewers of the year 2050? A. They will show more interest in virtual games. B. They will become couch potatoes. C. They will be able to check the food before ordering with a T V at home. D. They will have more choices of channels to watch and enjoy. 3. We can conclude from the passage that in 2050 ……………. . A. people will be able to travel around the world in two hours. B. less cars will be used to keep the environment green and clean. C. no one will be injured or will die due to accidents. D. spaces planes will take the place of cars as means of transport. 4. The word “This” refers to which of the following? A. changes in agriculture B. growing fruits and vegetables C. one of the most serious problems D. demand for water 5. The word “domain” is closest meaning to ……………. . A. area B. territory C. boundary D. range 6. Why does the author use “prefer robots”? A. to show the importance of robots in production B. to encourage the workers to resign from the work C. to compare the robots with the workers D. to emphasize the scientists’ roles in inventing robots 7. Why do big companies “prefer robots”? A. Robots ask for pay rises B. Robots don’t require salary or stop work C. Robots like to work in big companies D. Companies can’t afford to hire employees 8. It can’t be inferred from the passage EXCEPT ……………… . A. Cloning will offer babies for the one who won’t get married. B. Clones of people may live forever. C. Cloning is one of the examples of scientist’s controlling human genes. D. Clones of people will produced with the help of the electric devices. 9. The passage mainly discusses ……………. A. the effect of the futurologist’s prediction on our lives. B. the effect of the telecommunication and technology on our future life. C. changes in our future life compared with our lives today. D. the importance of cloning in the future. 10. What is the author’s attitude in the passage? A. concerned B. explanatory C. outraged D. emotional PART IV – WRITING: I. Complete the second sentence so that it has the similar meaning to the first one, by using the word given. (Do not change the word given. You must use between THREE or EIGHT words, including the word given) 1. There’s no way that you’re staying out all night with your friends, I’m afraid. question Your staying out all night with your friends …………………., I’m afraid. 2. My business partner and I are in complete agreement. difference There is ………………………………………………….. my business partner and me. 3. In the end they left me to settle up with the restaurant. foot


In the end I ……………………………………………… the restaurant bill. 4. He himself admits to having wasted a lot of money. admission By his ………………………………………………….wasted a lot of money. 5. Getting upset over Michael’s departure is pointless. tears There’s no …………………………………………over Michael’s departure.

FF IC IA L

II. Finish each of the following sentences in such a way that it means exactly the same as the sentence printed before it. 6. Although I tried hard, I couldn't lift the suitcase. Try ....................................................................................................... 7. The fisherman's life was one of great poverty. Throughout........................................................................................... 8. Someone has suggested abolishing income tax. It ........................................................................................................... 9. I do not enjoy cooking for five hungry children. Cooking ................................................................................................ 10. My parents find fault with everything I do. No matter ..............................................................................................

D

ẠY

M

Q

U

Y

N

H

Ơ

N

O

III. The graphs below provide information on global population figures and figures for urban populations in different world regions.Summarize the information by selecting and reporting the main features, and make comparisons where relevant.

IV. While the Ministry of Education and Training places high hopes on the new assessment method using comments instead of marks, primary schools have voiced their opposition. What is your opinion?


ĐÁP ÁN Part I: Listening Exercise 1 1. $ 14 2. ( has) swimming pool 4. $ 30 5. ( own) bathroom 8. Golden Sands 9. $ 4 an/ per hour Exercise 2 1. C 2. B 3. A 4. B 5. B Part II. I. Choose the best answer. 1. A 2. C 3. A 11. A 12. D 13. D

4. B 14. C

5. A 15. C

6. D 16. A

7. B 17. A

8. A 18. A

9. D 19. B

Ơ

N

O

II. Complete the sentences with the correct form of the words in bold. 1. participants 6. celebrity 2. expertise 7. contributor 3. unskilled 8. disastrous 4. explanations 9. painfully 5. surprisingly 10. considerably

FF IC IA L

3. (scuba) diving 6. fishing 7. Shute Harbour 10. soap and toothpaste

2. for

3. off

4. out

5. out

N

1. through

H

III. Complete each of the following sentences with a suitable particle

M

Q

U

Y

IV. From four underlined parts, choose the one that needs correction and then correct it 1. D are owned => is owned 2. C alike => same 3. C is more => is much 4. B that => 0 (deleted) 5. C maintain => maintenance

PART III – READING

4. so

5. or

6. there

9. before

10. little

ẠY

I. Complete each blank with ONE suitable word. 1. same 2. This/ That/ It 3. would/must 7. since

8. from

D

II. Read the following passage and decide which option A, B, C or D best fits each sentence. 1. C 2. A 3. D 4. B 5. B 6. D 7. A 8. D 9. A 10. A III.

1. A – vi 3. C – viii 5. E – iv 7. thousands of years 9. overseas museums

2. B – v 4. D – i 6. F – vii 8. (tree) bark 10. school walls

10. B 20. B


IV. Read the following passage and choose the best answer to each question. 1. D 2. C 3. C 4. B 5. A 6. C 7. B 8. C

9. C

10. A

PART IV – WRITING:

FF IC IA L

I. Complete the second sentence so that it has the similar meaning to the first one, by using the word given. (Do not change the word given. You must use between THREE or EIGHT words, including the word given) 1. is out of the question 2. no difference of opinion between 3. was left to foot 4. own admission, he (has) 5. point (in) shedding tears

D

ẠY

M

Q

U

Y

N

H

Ơ

N

O

II. Finish each of the following sentences in such a way that it means exactly the same as the sentence printed before it. 1. Try as I might, I couldn't lift the suitcase. 2. Throughout his life the fisherman was very poor/suffered from great poverty. 3. It has been suggested that income tax (should be) abolished. / that we/they/the government should abolish income tax. 4. Cooking for five hungry children is no fun/gives me no pleasure. 5. No matter what I do my parents find fault with it/me.


SỞ GIÁO DỤC & ĐÀO TẠO HẢI PHÒNG TRƯỜNG THPT CHUYÊN TRẦN PHÚ

KÌ THI CHỌN HỌC SINH GIỎI KHU VỰC DUYÊN HẢI BẮC BỘ NĂM HỌC 2016- 2017 MÔN THI: Tiếng Anh

ĐỀ ĐỀ NGHỊ

LỚP: 11

Ngày thi: … tháng 4 năm 2016

FF IC IA L

(Thời gian làm bài 180 phút không kể thời gian giao đề) PART ONE. LISTENING (40p.)

Question 1 (10p.) You will hear an interview with Michael Jacobson about bilingual children. Choose the answer A, B, C or D which fits best according to what you hear. The audio will be played twice.

D

ẠY

M

Q

U

Y

N

H

Ơ

N

O

1. The language problem being discussed is that of A. English-speaking children in a non-English-speaking environment. B. non-English-speaking children in an English-speaking environment. C. English-speaking children in an English-speaking environment. D. non-English-speaking children in a non-English-speaking environment. 2. When the families arrive in France, A. the children already speak French. B. the children are pressured by local children. C. the children quickly adjust to the local language. D. the children reject their own language. 3. English-speaking children have problems in secondary school because A. they find the lessons too easy. B. they find it difficult to learn English as a foreign language. C. all their lessons have been taught in French hitherto. D. they don’t know any English grammar. 4. Reading levels can only be maintained A. at the expense of homework in the second language. B. if reading is also practiced outside of school hours. C. by children who write fluently in English. D. by learning to read at an early age. 5. Michael suggests that bilingual children A. should be forced to conform to English language teaching methods. B. should have their language ability nurtured. C. should be segregated from other pupils. D. shouldn’t be made to feel unusual. Your answers: 1. 2. 3. 4. 5. 1


Questions 2. Listen to the conversation between Helen and Tony below and decide if each statement is TRUE (T) or FALSE (F). (10p.) STATEMENT

TRUE

FALSE

1. Both of them believe that being contactable all the time is not

FF IC IA L

necessarily a good idea.

2. Helen thinks e-mails encourage laziness of thought.

3. Tony supposes that popular music isn’t as good as it used to be because of technology.

4. Only Helen shares the idea that watching sport on TV has

O

been made more interesting by modern technology.

5. Both of them believe that some innovations of modern

3.

4.

5.

H

2.

Ơ

Your answers: 1.

N

technology will disappear in the future.

D

ẠY

M

Q

U

Y

N

Questions 3. You will hear a talk about Phyllis Pearsall, the creator of the London mapbook - “the A-Z”. Complete the sentences with NO MORE THAN TWO WORDS for each gap. The audio will be played twice. (20p) 1. Before beginning the map project, Phyllis worked as a ________________. 2. Before “the A-Z”, most maps of London concentrated on ________________ features. 3. Phyllis covered a total of ________________ streets during the project. 4. Phyllis’s father is described as being ________________ of the project. 5. Unlike other maps, “the A-Z” is not ________________ in basis. 6-7. Phyllis is described as being both ________________ and ________________ in her sketching. 8. When cataloguing the streets she’d sketched, Phyllis used a system of cards in ________________. 9. The omission of Trafalgar Square from the index was noticed by the book’s ________________. 10. The first type of shop to sell “the A-Z” maps was a ________________. Your answers: 1.

2.

3.

4.

5.

6.

7.

8.

9.

10.

2


PART TWO. LEXICO – GRAMMAR (40p.) Question 1. Choose the word or phrase which best completes each sentence. Write your answers (A, B, C, or D) in the space provided under this part. (20p.)

D

ẠY

M

Q

U

Y

N

H

Ơ

N

O

FF IC IA L

1. The old lady _______ at them kindly when she saw them enter her shop. A. glared B. grinned C. grimaced D. beamed 2. The railway track has been constructed in such a way as to allow space for _______ on a hot day. A. extension B. expropriation C. expansion D. explosion 3. According to Burgess, a novelist should not _______, for sermonizing has no place in good fiction. A. offen B. preach C. distort D. invent 4. At the moment the ruling party is on the _______ of a dilemma. A. hooves B. points C. top D. horns 5. We couldn’t stay long, so we only wished Mark many happy _______ for his birthday and hurried to the airport. A. days B. returns C. moments D. regards 6. Choose the word which is CLOSEST in meaning to the underlined word in the following sentence: “Once you have started something, you ought to see it through to the end.” A. be persevere with B. be subjected to C. be conspicuous about D. be stuck with 7. Choose the word which is OPPOSITE in meaning to the underlined word in the following sentence: “The senator chose to incur dislike rather than compromise her principles to win favor with the public.” A. dissent B. negotiate C. articulate D. take away 8. Close your eyes and try to _______ up a picture of a place where you feel at peace. A. store B. eat C. stick D. conjure 9. Martin: “Hey, can you pick me up at 3 p.m?” Peter: “Sorry, I can’t. I don’t have my _______ at the moment!” A. vehicles B. trail C. wheels D. means 10. Please don’t _______ when you talk to me. Open your mouth! A. murmur B. mumble C. stumble D. stutter 11. My teacher _______ for my good marks in the Math test. A. promised me the moon B. coincided with me C. gave me a pat on the back D. showed no sign of remorse 12. Advertisers often aim their campaigns at young people as they have considerable spending _______. 3


ẠY

M

Q

U

Y

N

H

Ơ

N

O

FF IC IA L

A. power B. force C. energy D. capacity 13. Jennifer: “Hey, Tommy. What are you guys doing?” Tommy: “_______” A. We’re just chilling out. Do you want to come around? B. I don’t think it’s a question I can discuss. C. We’re talking to you, Jennifer! D. I’m not sure if we’re doing something much. 14. We sent him to the best school in England and hired the best teachers, but it was all _______ as he had no will to learn at all. A. vain B. incurable C. invalid D. futile 15. I was _______ in an autumn month 18 years ago. A. taken up B. put forward C. given away D. brought forth 16. Choose the word which is CLOSEST in meaning to the underlined word in the following sentence: “The construction of mammoth shopping malls has contributed to the decline of small stores in neighboring towns.” A. modern B. gigantic C. numerous D. separate 17. Choose the word which is OPPOSITE in meaning to the underlined word in the following sentence: “An experiment can be marred by poor planning.” A. disclosed B. qualified C. constructed D. spoiled 18. If I _______ home a bit later, I would never have had a chance to pass the door mat. A. have come B. were to have come C. came D. could have come 19. A trap _______ disguise is what has come to be called a Trojan House. A. offered a gift of B. offers a gift in C. offering a gift to D. offered as a gift in 20. Peter: “Hey, do you want to go out tonight?” Mary: “Sorry I can’t. _______ and I will have to wake up early tomorrow.” A. I’m beat B. I’m cool C. I’m in the cloud D. I’m socked off Your answers 2

3

4

5

6

7

8

9

10

11.

12.

13.

14.

15.

16.

17.

18.

19.

20.

D

1

4


Question 2. There are FIVE mistakes in this paragraph. Write them down & give the correction. Write your answers in the space provided. (5p.)

Mistake

Correction

N

Line

O

FF IC IA L

Line 1 Anna and Chris made me at ease the first day in their polishing 2 living room—though I was not sure why these people would bother putting 3 themselves up for me at all. And when they kept inviting me back for 4 dinner parties and extending their hospitality, I wondered if maybe they 5 were bored, or if their ignorance of American types was so that they failed 6 to see that I was not at all of their social class: I kept expecting some crude 7 regional expression to betray me; and, once thought of it in those terms, I 8 knew I would have to make sure they saw that side of me—to do less 9 would be like trying to “pass”. Yet whichever I said seemed to make no 10 difference in their acceptance. Your answers

Ơ

1. 2.

H

3.

N

4.

U

Y

5.

Q

Question 3. Fill in each gap with ONE preposition to finish the following sentences. Write your answers in the space provided. (10p.)

M

1. No witnesses to the murder have come _______ and the police are struggling to solve the crime.

2. When they first met, Kate was quite taken _______ Jim. 3. Mrs. Chapman is always polite and considerate _______ her employees.

ẠY

4. Despite all the interruptions, he pressed _______ with his work.

D

5. Why don’t we have a night out? It would take your mind _______ your work. Your answers 1.

2.

5

3.

4.

5.


Question 4. Use the correct form of each bracketed word in the numbered space. Write your answers in the space provided. (10p.)

FF IC IA L

When (1-MUSE) on cities over time and in our time, from the first (whenever it was) to today, we must always remember that cities are (2-FACT). Forests, jungles, deserts, plains, oceans – the organic environment is born and dies and is (3-BEAR) endlessly, beautifully, and completely without moral constraint or (4-ETHIC) control. But cities, despite the metaphors that we apply to them from biology or nature (“The city dies when industry flees”; “The neighborhoods are the vital cells of the urban (5-ORGANIC)”), despite the anthropomorphic or (6-SENTIMENT) devices we use to describe cities – are artificial. Nature has never made a city, and what Nature makes that may seem like a city – an anthill, for instance – only seems like one. It is not a city.

H

Ơ

N

O

Human (7-BE) made and make cities, and it is they, only, that kill cities or let them die. And they do both – make cities and (8-MAKE) cities, by the same means: by (9-ACT) of choice. We enjoy deluding ourselves in this as in other things. We enjoy believing that there are forces out there completely (10-DISPOSE) our fate, natural forces, or forces so strong and overwhelming that send cities through organic and biological phases of birth, growth, and decay.

N

Your answers 1.

6.

Y

2.

U

3.

Q

4.

D

ẠY

M

5.

6

7. 8. 9.

10.


PART THREE: READING (60p.)

D

ẠY

M

Q

U

Y

N

H

Ơ

N

O

FF IC IA L

Question 1. Read the text below and decide which answer (A, B, C or D) best fits each gap. Write your answers in the space provided. (15p.) One of the hazards that electronic media like the television, radio or computers (1)…. these days is the (2)…. in book reading.The concern (3)…. mainly to the younger generations who are strongly (4)…. by the glamour of the silver screen and, consequently, don’t (5)…. the importance of acquiring first-hand information from books. To (6)…. reading for pleasure and to propagate a wide array of publications like encyclopedias, (7)…. books, manuals or fiction, radical solutions should be applied. Firstly, more (8)…. ought to be put on the educational (9)…. Youngsters should be made to feel comfortable while reading either for information or self-satisfaction in public places like airports, buses or on the beach. Secondly, libraries must be subsidized more accurately in order to provide the potential reader with (10)…. choice of publications and to become more publicly active so as to put books at people’s (11)…. rather than keep them under lock and key. Fund collecting actions organized by libraries might also (12)…. the public awareness of the advantages of becoming (13)…. in a good book. Finally, the mass media themselves might contribute substantially by recommending of purchase or valuable best-sellers and inspiring their viewers to (14)…. their knowledge and erudition, and thus help them to (15)…. the habit of spontaneous everyday reading. 1. A. denote B. play C. arise D. pose 2. A. rarity B. decline C. shortage D. deficiency 3. A. indicates B. affects C. embodies D. applies 4. A. tempted B. exposed C. submitted D. involved 5. A. observe B. recognize C. view D. distinguish 6. A. incite B. revert C. instill D. encourage 7. A. referral B. referable C. referee D. reference 8. A. relevance B. persistence C. focus D. emphasis 9. A. factor B. ground C. matter D. point 10. A. prolific B. ample C. lavish D. lush 11. A. availability B. usage C. benefit D. disposal 12. A. inflate B. amplify C. raise D. expand 13. A. occupied B. inhaled C. engrossed D. incorporated 14. A. enrich B. magnify C. arouse D. elaborate 15. A. grow B. evolve C. proceed D. develop Your answers: 1. 2. 3. 4. 5. 6. 7. 8. 9.

10.

7

11.

12.

13.

14.

15.


FF IC IA L

Question 2. Fill each of the numbered blanks in the following passage with one suitable word. Write your answers in the space provided. (15p.) It is because of the centrality of the city that the financial markets have (1)_______ put. It had been (2) _______ forecast that they would move out en masse, financial work (3) _______ among the most quantitative and computerized of functions. A lot of the back-office work has been relocated. The main business, (4) _______, is not record keeping and support services; it is people sizing (5) _______ other people, and the center is the place for that.

2.

3.

6.

7.

8.

N

H

Your answers: 1.

Ơ

N

O

The problems, of course, are immense. To be an optimist (6) _______ the city, one must believe that it will lurch from crisis to crisis but somehow survive. Utopia is (7) _______ in sight and probably never will be. The city is (8) _______ mixed up for that. Its strengths and its ills are inextricably bound (9) _______. The same concentration that makes the center efficient is the cause of its crowding and the destruction of its sun and its light and its scale. Many of the city’s problems, (10) _______, are external in origin – for example, the cruel demographics of peripheral growth, which are difficult enough to forecast, let alone do anything about. 5.

9.

10.

M

Q

U

Y

Question 3. Read the following passage and choose the best answer A, B, C or D for each question. Write your answers in the space provided. (15p.) Among the Plains Indians, two separate strains of decorative art evolved: the figurative, representational art created by the men of the tribe, and the geometric, abstract art crafted by the women. According to Dunn and Highwater, the artist’s sex governed both the kind of article to be decorated and the style to be Line 5 followed in its ornamentation. Thus, the decorative works created by tribesmen consistently depict living creatures (men, horses, buffalo) or magical beings (ghosts and other supernatural life-forms). Those created by women, however, are clearly nonrepresentational: no figures of men or animals appear in this classically geometric art. Art historians theorize that this abstract, geometric art, traditionally the Line 10 prerogative of the women, predates the figurative art of the men. Descending from those aspects of Woodland culture that gave rise to weaving, quillwork, and beadwork, it is a utilitarian art, intended for the embellishment of ordinary, serviceable objects such as parfleche boxes (cases made of rawhide), saddlebags, Line 15 and hide robes. The abstract designs combine classical geometric figures into formal patterns: a ring of narrow isosceles triangles arranged on the background of

ẠY D

4.

8


M

Q

Line 40

U

Y

N

Line 35

H

Ơ

N

Line 30

FF IC IA L

Line 25

O

Line 20

a large central circle creates the well-known “feather and circle” pattern. Created in bold primary colors (red, yellow, blue), sometimes black or green, and often outlined in dark paint or glue size, these nonrepresentational designs are nonetheless intricately detailed. Although the abstract decorations crafted by the women are visually striking, they pale in significance when compared to the narrative compositions created by the men. Created to tell a story, these works were generally heroic in nature, and were intended to commemorate a bold and courageous exploit or a spiritual awakening. Unlike realistic portraits, the artworks emphasized action, not physical likeness. Highwater describes their making as follows: “These representational works were generally drafted by a group of men—often the individuals who had performed the deeds being recorded—who drew on untailored hide robes and tepee liners made of skins. The paintings usually filled the entire field; often they were conceived at different times as separate pictorial vignettes documenting specific actions. In relationship to each other, these vignettes suggest a narrative.” The tribesmen’s narrative artwork depicted not only warlike deeds but also mystic dreams and vision quests. Part of the young male’s rite of passage into tribal adulthood involved his discovering his own personal totem or symbolic guardian. By fasting or by consuming hallucinatory substances, the youth opened himself to the revelation of his “mystery object,” a symbol that could protect him from both natural and supernatural dangers. What had been in the early 1700s a highly individualistic, personal iconography changed into something very different by the early nineteenth century. As Anglos came west in ever greater numbers, they brought with them new materials and new ideas. Just as European glass beads came to replace native porcupine quills in the women’s applied designs, cloth eventually became used as a substitute for animal hides. The emphasis of Plains artwork shifted as well: tribespeople came to create works that celebrated the solidarity of Indians as a group rather than their prowess as individuals.

Line 45

D

ẠY

1. Which of the following titles best summarizes the content of the passage? A. The Ongoing Influence of Plains Indian Art B. Male and Female in Tribal Life C. Indian Art as Narrative and Dream D. Design Specialization in Plains Art 2. The author cites examples of the work of Plains artists primarily to A. show the differences between male and female decorative styles B. emphasize the functional role of art in Indian life C. describe the techniques employed in the creation of particular works D. illustrate the changes made by Anglo influence on Plains art 3. The word “strains” in line 1 means 9


M

Q

U

Y

N

H

Ơ

N

O

FF IC IA L

A. tunes B. pressures C. varieties D. injuries 4. In lines 12 and 13, weaving, quillwork, and beadwork are presented as examples of A. male-dominated decorative arts B. uninspired products of artisans C. geometrically based crafts D. unusual applications of artistic theories 5. With which of the following statements regarding male Plains artists prior to 1800 would the author most likely agree? I. They tended to work collaboratively on projects. II. They believed art had power to ward off danger. III. They derived their designs from classical forms. A. I only B. III only C. I and II only D. II and III only 6. As used in line 27, “drafted” most nearly means A. selected B. recruited C. endorsed D. sketched 7. According to the passage, dream visions were important to the Plains artist because they A. revealed the true form of his spiritual guardian B. suggested the techniques and methods of his art C. determined his individual aesthetic philosophy D. expressed his sense of tribal solidarity 8. In its narrative aspect, Plains art resembles LEAST A. a cartoon strip made up of several panels B. a portrait bust of a chieftain in full headdress C. an epic recounting the adventures of a legendary hero D. a chapter from the autobiography of a prominent leader 9. According to lines 40–45, the impact of the Anglo presence on Plains art can be seen in the A. growth of importance of geometric patterning B. dearth of hides available to Plains Indian artists C. shift from depicting individuals to depicting the community D. emphasis on dream visions as appropriate subject matter for narrative art 10. As used in line 43, “substitute” most nearly means A. surrogate B. back-up C. relay D. fill-in

D

ẠY

Your answers: 1. 2.

10

3.

4.

5.

6.

7.

8.

9.

10.


Question 4. There are six sections A-F in the following passage. Choose the correct heading for each section from the list of headings below. Write the correct number i-x in the answer boxes 1-6. Then finish the chart which follows the passage. (15p.)

i. Locations and features of different seaweeds. iii. Use of seaweeds in Japan. v. Nutritious value of seaweeds. vii. Where to find red seaweeds ix. Mystery solved

ii. Various products of seaweeds. iv. Seaweed species around the globe. vi. Why it doesn’t dry or sink viii. Underuse of native species x. How seaweeds reproduce and grow

H

Ơ

N

O

(1) Section A: __________ (2) Section B: __________ (3) Section C: __________ (4) Section D: __________ (5) Section E: __________ (6) Section F: __________

FF IC IA L

List of Headings

N

NEW ZEALAND SEAWEED

D

ẠY

M

Q

U

Y

Section A Seaweed is a particularly nutritious food, which absorbs and concentrates traces of a wide variety of minerals necessary to the body's health. Many elements may occur in seaweed aluminium, barium, calcium, chlorine, copper, iodine and iron, to name but a few - traces normally produced by erosion and carried to the seaweed beds by river and sea currents. Seaweeds are also rich in vitamins: indeed. Eskimos obtain a high proportion of their bodily requirements of vitamin C from the seaweeds they eat. The nutritive value of seaweed has long been recognized. For instance, there is a remarkably low incidence of goiter amongst the Japanese, and for that matter, amongst our own Maori people, who have always eaten seaweeds, and this may well be attributed to the high iodine content of this food. Research into old Maori eating customs shows that jellies were made using seaweeds, fresh fruit and nuts, fuchsia and tutu berries, cape gooseberries, and many other fruits which either grew here naturally or were sown from seeds brought by settlers and explorers. Section B New Zealand lays claim to approximately 700 species of seaweed, some of which have no representation outside this country. Of several species grown worldwide, New Zealand also has a particularly large share. For example, it is estimated that New Zealand has some 30 11


FF IC IA L

species of Gigartina, a close relative of carrageen or Irish moss. These are often referred to as the New Zealand carrageens. The gel-forming substance called agar which can be extracted from this species gives them great commercial application in sea meal, from which sea meal custard is made, and in cough mixtures, confectionery, cosmetics, the canning, paint and leather industries, the manufacture of duplicating pads, and in toothpastes. In fact, during World War II, New Zealand Gigartina were sent to Australia to be used in toothpaste.

H

Ơ

N

O

Section C Yet although New Zealand has so much of the commercially profitable red seaweeds, several of which are a source of agar (Pterocladia, Gelidium, Chondrus, Gigartina), before 1940 relatively little use was made of them. New Zealand used to import the Northern Hemisphere lrish moss (Chondrus crispus) from England and ready-made agar from Japan. Although distribution of the Gigartina is confined to certain areas according to species, it is only on the east coast of the North Island that its occurrence is rare. And even then, the east coast and the area around Hokiangna have a considerable supply of the two species of Pterocladia from which agar is also available. Happily, New Zealand-made agar is now obtainable in health food shops.

D

ẠY

M

Q

U

Y

N

Section D Seaweeds are divided into three classes determined by colour—red, brown and green—and each tends to live in a specific location. However, except for the unmistakable sea lettuce [Ulva), few are totally one colour; and especially when dry, some species can change colour quite significantly—a brown one may turn quite black, or a red one appear black, brown, pink or purple. Identification is nevertheless facilitated by the fact that the factors which determine where a seaweed will grow are quite precise, and they tend therefore to occur in very well-defined zones. Although there are exceptions, the green seaweeds are mainly shallow-water algae; the browns belong to medium depths, and the reds are plants of the deeper water. Flat rock surfaces near mid-level tides are the most usual habitat of sea-bombs, Venus' necklace and most brown seaweeds. This is also the location of the purple laver or Maori karengo, which looks rather like a reddish-purple lettuce. Deep-water rocks on open coasts, exposed only at very low tide, are usually the site of bull kelp, strapweeds and similar tough specimens. Those species able to resist long periods of exposure to sun and air are usually found i on the upper shore, while those less able to stand such exposure occur nearer to or below the lowwater mark. Radiation from the sun, the temperature level, and the length of time immersed all play a part in the zoning of seaweeds.

12


FF IC IA L

Section E Propagation of seaweeds occurs by spores, or by fertilization of egg cells. None have roots in the usual sense; few have leaves, and none have flowers, fruits or seeds. The plants absorb their nourishment through their fronds when they are surrounded by water: the base or "holdfast" of seaweeds is purely an attaching organ, not an absorbing one.

Ơ

N

O

Section F Some of the large seaweeds maintain buoyancy with air-filled floats; others, such as bull kelp, have large cells filled with air. Some, which spend a good part of their time exposed to the air, often reduce dehydration either by having swollen stems that contain water, or they may (like Venus' necklace) have swollen nodules, or they may have distinctive shape like a sea-bomb. Others, like the sea cactus, are filled with slimy fluid or have coating of mucilage on the surface. In some of the larger kelps, this coating is not only to keep the plant moist but also to protect it from the violent action of waves.

H

Complete the flow chart below. Choose NO MORE THAN THREE WORDS from the passage for each answer. Write your answers in the answer boxes 7-10.

N

Gigartina

↓ made into

(8) _______________________

M

Q

U

Y

(also called as (7)_______________________)

medicine (e.g. (10)_______________________)

canned or bottled food

toothpaste

(9) _______________________

others

ẠY

Your answers

D

1.

7.

13

2.

3. 8.

4. 9.

5.

6. 10.


PART IV: WRITING (60p.)

Example:

We couldn't find the cat anywhere.

FF IC IA L

Question 1. For each of the sentence below, write a new sentence as similar as possible in meaning to the original sentence but using the word given. This word must not be altered in any way. (5p.)

(NOWHERE )

Answer:

The cat was nowhere to be found.

O

1. It was wrong of me to get angry in front of the children.

(LOST)

N

_________________________________________________________________

(GAINED)

H

Ơ

2. It’s possible that the burglars got into the building by forcing open a fire exit.

N

_________________________________________________________________

Y

3. Nobody can predict the real consequences of global warming. (SAYING)

U

_________________________________________________________________

Q

4. Even if we drive very fast, we’ll never get to the airport on time. (HOW)

M

_________________________________________________________________

D

ẠY

5. He answered the judge’s question as accurately as he could. (ACCURATE) _________________________________________________________________

14


Ơ

N

O

FF IC IA L

Question 2. The graph shows the number of cases of X disease in Someland between 1960 and 1995. Summarise the information by selecting and reporting the main features, and make comparisons where relevant. You should write about 150 words. (20p.)

______________________________________________________________________

H

______________________________________________________________________

N

______________________________________________________________________

Y

______________________________________________________________________

U

______________________________________________________________________

Q

______________________________________________________________________ ______________________________________________________________________

M

______________________________________________________________________

______________________________________________________________________ ______________________________________________________________________

ẠY

______________________________________________________________________ ______________________________________________________________________

D

______________________________________________________________________ ______________________________________________________________________ ______________________________________________________________________

15


Question 3. Give your response to the following statement. You should write about 250 words (35p.) Many people believe that social networking sites (such as Facebook) have had a huge negative impact on both individuals and society.

FF IC IA L

______________________________________________________________________ ______________________________________________________________________ ______________________________________________________________________ ______________________________________________________________________

______________________________________________________________________

O

______________________________________________________________________ ______________________________________________________________________

N

______________________________________________________________________

Ơ

______________________________________________________________________

H

______________________________________________________________________

N

______________________________________________________________________ ______________________________________________________________________

Y

______________________________________________________________________

U

______________________________________________________________________

Q

______________________________________________________________________ ______________________________________________________________________

M

______________________________________________________________________

______________________________________________________________________ ______________________________________________________________________

D

ẠY

______________________________________________________________________

16

-THE END-


BIỂU ĐIỂM VÀ ĐÁP ÁN CHẤM PART ONE. LISTENING (40p.) Questions 1 (10p.) 2p for each correct answer

FF IC IA L

1. A 2. C 3. B 4. B 5. B Questions 2 (10p.) 2p for each correct answer

O

1. T 2. F

N

3. F

Ơ

4. T

H

5. T

N

Questions 3 (20p.) 2p for each correct answer

3. 23000

Q

4. unsupportive

U

2. geographical

Y

1. portrait artist

M

5. mathematical 6. efficient

Q6 and Q7 in either order

7. observant

8. shoeboxes/shoe boxes

ẠY

9. printer(s)

D

10. newsagent

17


PART TWO. LEXICO – GRAMMAR (40p.) Question 1. Choose the word or phrase which best completes each sentence. Write your answers (A, B, C, or D) in the space provided under this part. (20p.) 1p for each correct answer. 2. C

3. B

4. D

5. B

6. A

7. A

11. C

12. A

13. A

14. D

15. D

16. B

17. C

8. D

9. C

10. B

18. B

19. D

20. A

FF IC IA L

1. D

Question 2. There are FIVE mistakes in this paragraph. Write them down & give the correction. Write your answers in the space provided. (5p.) 1p for each correct answer.

2.

3

up

3.

5

so

4.

7

thought

5.

9

whichever

polished

O

polishing

out

N

1

Ơ

1.

Correction

H

Mistake

such thinking whatever

N

Line

Q

forward with towards on off

M

1. 2. 3. 4. 5.

U

Y

Question 3. Fill in each gap with ONE preposition to finish the following sentences. Write your answers in the space provided. (10p.) 2p for each correct answer.

D

ẠY

Question 4. Use the correct form of each bracketed word in the numbered space. Write your answers in the space provided. (10p.) 1p for each correct answer. 1. musing 2. artifacts 3. reborn 4. ethical 5. organism 6. sentimental 7. beings 8. unmake 18


9. acts 10. predisposing PART THREE: READING (60p.)

FF IC IA L

Question 1. Read the following passage and choose the best answer A, B, C or D for each question. (15p.) 1p for each correct answer. 1. D

2. B

3. D

4. A

5. B

6. D

7. D

8. D

9. A

10. B

11. D

12. C

13. C

14. A

15. D

2. widely

3. being

6. about

7. nowhere

8. too

4. however

N

1. stayed

O

Question 2. Fill each of the numbered blanks in the following passage with one suitable word. Write your answers in the space provided. (15p.) 1.5p for each correct answer.

Ơ

9. together

5. up 10. furthermore/

H

moreover

2. A

3. C

4. C

5. C

6. D

7. A

8. B

9. C

10. A

Q

U

1. D

Y

N

Question 3. Read the following passage and choose the best answer A, B, C or D for each question. Write your answers in the space provided. (15p.) 1.5p for each correct answer.

M

Question 4. There are six sections A-F in the following passage. Choose the correct heading for each section from the list of headings below. Write the correct number i-x in the answer boxes 1-6. Then finish the chart which follows the passage. (15p.) 1.5p for each correct answer. 1. v

2. ii

Zealand 8. agar

4. i 9. seameal

5. x

6. vi 10. cough mixture

D

ẠY

7. New carrageen

3. viii

PART IV: WRITING (60p.) Question 1: Finish each of the following sentences in such a way that it is as similar as possible in meaning to the sentence printed before it. (5p.) 1p for each correct answer. 1. I should not have lost my temper in front of the children. 19


2. The burglars may have gained access/entry to the building by forcing open a fire exit. 3. There is no saying what the real consequences of global warming are. 4. No matter how fast we drive, we will never get to the airport on time.

FF IC IA L

5. He gave as accurate an answer to the judge’s question as he could.

Question 2. The graph shows the number of cases of X disease in Someland between 1960 and 1995. Summarise the information by selecting and reporting the main features, and make comparisons where relevant. You should write about 150 words. (20p.) The mark is based on the following marking scheme:

O

1. Task fulfillment (5p):

+ clearly presents a fully developed response

Ơ

2. Coherence and Cohesion (5p):

N

+ fully satisfies all the requirements of the task ƒ

H

+ uses cohesion in such a way that it attracts no attention ƒ

N

+ skillfully manages paragraphing 3. Lexical Resources (5p):

U

Y

+ uses a wide range of vocabulary with very natural and sophisticated control of lexical features;

Q

+ rare minor errors occur only as ‘slips’ 4. Grammatical Range and Accuracy (5p):

M

+ uses a wide range of structures with full flexibility and accuracy;

+ rare minor errors occur only as ‘slips’

ẠY

Question 3. Essay writing

D

Give your response to the following statement. You should write about 250 words (35p.) Many people believe that social networking sites such as Facebook have had a huge negative impact on both individuals and society.

The mark is based on the following scheme: 1. Content: (15p.) a provision of all main ideas and details as appropriate.

20


2. Language: (10p.) a variety of vocabulary and structures appropriate to the level of English language gifted upper-secondary school students

D

ẠY

M

Q

U

Y

N

H

Ơ

N

O

FF IC IA L

3. Presentation: (10p.) coherence, cohesion, and style appropriate to the level of English language gifted upper-secondary school students

21


Turn static files into dynamic content formats.

Create a flipbook
Issuu converts static files into: digital portfolios, online yearbooks, online catalogs, digital photo albums and more. Sign up and create your flipbook.